Download as pdf or txt
Download as pdf or txt
You are on page 1of 305

DEJA

REVIEW™
Internal Medicine

NOTICE

Medicine is an ever-changing science. As new research and clinical experience broaden our knowledge, changes
in treatment and drug therapy are required. The authors and the publisher of this work have checked with sources
believed to be reliable in their efforts to provide information that is complete and generally in accord with the
standards accepted at the time of publication. However, in view of the possibility of human error or changes in
medical sciences, neither the authors nor the publisher nor any other party who has been involved in the
preparation or publication of this work warrants that the information contained herein is in every respect accurate
or complete, and they disclaim all responsibility for any errors or omissions or for the results obtained from use of
the information contained in this work. Readers are encouraged to confirm the information contained herein with
other sources. For example and in particular, readers are advised to check the product information sheet included
in the package of each drug they plan to administer to be certain that the information contained in this work is
accurate and that changes have not been made in the recommended dose or in the contraindications for
administration. This recommendation is of particular importance in connection with new or infrequently used
drugs.
DEJA REVIEW™
Internal Medicine

Second Edition

Sarvenaz S. Saadat Mobasser, MD



School of Medicine
University of California, Irvine
Irvine, California
Residency in Family Medicine
Kaiser Permanente
Woodland Hills Medical Center
Woodland Hills, California


Copyright © 2011, 2008 by The McGraw-Hill Companies, Inc. All rights reserved. Except as permitted under the
United States Copyright Act of 1976, no part of this publication may be reproduced or distributed in any form or
by any means, or stored in a database or retrieval system, without the prior written permission of the publisher.

ISBN: 978-0-07-173633-6

MHID: 0-07-173633-6

The material in this eBook also appears in the print version of this title: ISBN: 978-0-07-171517-1, MHID: 0-07-
171517-7.

All trademarks are trademarks of their respective owners. Rather than put a trademark symbol after every
occurrence of a trademarked name, we use names in an editorial fashion only, and to the benefit of the trademark
owner, with no intention of infringement of the trademark. Where such designations appear in this book, they
have been printed with initial caps.

McGraw-Hill eBooks are available at special quantity discounts to use as premiums and sales promotions, or for
use in corporate training programs. To contact a representative please e-mail us at [email protected].

TERMS OF USE

This is a copyrighted work and The McGraw-Hill Companies, Inc. (“McGraw-Hill”) and its licensors reserve all
rights in and to the work. Use of this work is subject to these terms. Except as permitted under the Copyright Act
of 1976 and the right to store and retrieve one copy of the work, you may not decompile, disassemble, reverse
engineer, reproduce, modify, create derivative works based upon, transmit, distribute, disseminate, sell, publish or
sublicense the work or any part of it without McGraw-Hill’s prior consent. You may use the work for your own
noncommercial and personal use; any other use of the work is strictly prohibited. Your right to use the work may
be terminated if you fail to comply with these terms.

THE WORK IS PROVIDED “AS IS.” McGRAW-HILL AND ITS LICENSORS MAKE NO GUARANTEES
OR WARRANTIES AS TO THE ACCURACY, ADEQUACY OR COMPLETENESS OF OR RESULTS TO
BE OBTAINED FROM USING THE WORK, INCLUDING ANY INFORMATION THAT CAN BE
ACCESSED THROUGH THE WORK VIA HYPERLINK OR OTHERWISE, AND EXPRESSLY DISCLAIM
ANY WARRANTY, EXPRESS OR IMPLIED, INCLUDING BUT NOT LIMITED TO IMPLIED
WARRANTIES OF MERCHANTABILITY OR FITNESS FOR A PARTICULAR PURPOSE. McGraw-Hill
and its licensors do not warrant or guarantee that the functions contained in the work will meet your requirements
or that its operation will be uninterrupted or error free. Neither McGraw-Hill nor its licensors shall be liable to
you or anyone else for any inaccuracy, error or omission, regardless of cause, in the work or for any damages
resulting there from. McGraw-Hill has no responsibility for the content of any information accessed through the
work. Under no circumstances shall McGraw-Hill and/or its licensors be liable for any indirect, incidental,
special, punitive, consequential or similar damages that result from the use of or inability to use the work, even if
any of them has been advised of the possibility of such damages. This limitation of liability shall apply to any
claim or cause whatsoever whether such claim or cause arises in contract, tort or otherwise.
To my beautiful daughters Chaya and Charlotte:
You are the inspiration for everything I do.
You put the twinkle in my eyes.
Always reach for the moon and the stars!
With all my love,
Mommy
Contents

Reviewers

Acknowledgments

Preface

Introduction
Important Lab Values
Abbreviations You Should Know

Chapter 1 THE BASICS


Quick Radiology
Quick EKG Interpretation
Preventative Medicine
Preventative Screening
Fluids and Electrolytes
Nutrition
Blood Products and Transfusions

Chapter 2 CARDIOLOGY
Hypertension
Hyperlipidemia
Coronary Artery Disease
Arrhythmias
Congestive Heart Failure
Valvular Heart Diseases
Cardiomyopathy
Endocarditis
Rheumatic Fever
Pericarditis
Myocarditis
Cardiac Tamponade
Abdominal Aortic Aneurysm
Clinical Vignettes

Chapter 3 PULMONOLOGY
Lung Volumes
Hypoxia
Obstructive Pulmonary Diseases
Restrictive Lung Disease
Pleural Effusion
Cough

Acute Respiratory Distress Syndrome


Pulmonary Embolism
Pneumothorax
Hemoptysis
Lung Cancer
Pneumonia
Tuberculosis
Clinical Vignettes

Chapter 4 NEUROLOGY
Cerebrovascular Accidents
Seizure Disorders
Meningitis
Brain Tumors
Demyelinating Diseases
Cognitive Disorders
Headache
Intracranial Bleeding
Vertigo
Clinical Vignettes

Chapter 5 GASTROENTEROLOGY
Esophageal Disorders
Gastroesophageal Reflux Disease
Gastritis
Peptic Ulcers
GI Bleed
Colon
Inflammatory Bowel
Diarrhea
Malabsorption Disorders
Pancreas
Biliary Tract
Liver
Clinical Vignettes

Chapter 6 HEMATOLOGY-ONCOLOGY
Anemia
Microcytic Anemias
Macrocytic Anemias
Normocytic Anemia
Coagulopathies
Leukemias
Lymphoma
Myeloproliferative Diseases
Clinical Vignettes

Chapter 7 RHEUMATOLOGY
Arthropathies
Systemic Disorders
Muscle Disorders
Vasculitis
Clinical Vignettes

Chapter 8 NEPHROLOGY
Acute Renal Failure
Chronic Renal Failure
Glomerulonephropathies
Urinary Tract
Acid-Base Disorders
Renal Artery Stenosis
Clinical Vignettes

Chapter 9 ENDOCRINOLOGY
Diabetes
Pituitary
Thyroid
Parathyroid
Adrenals
Bones
Clinical Vignettes

Chapter 10 INFECTIOUS DISEASE


HIV/AIDS
Sexually Transmitted Diseases
Sepsis
Osteomyelitis
Clinical Vignettes

Chapter 11 DERMATOLOGY
Terminology
Skin Cancers
Psoriasis
Blistering Diseases
Vector-Borne Diseases
Fungal Infections
Bacterial and Viral Infections
Pigmentary Disorders
Hypersensitivity Reactions
Clinical Vignettes

Chapter 12 CLINICAL VIGNETTES REVIEW

Bibliography

Index
Reviewers

Edward R. Gould
Fourth Year Medical Student
SUNY Upstate Medical University
Class of 2009

Michael Sidholm, MD
PGY-1 Internal Medicine
Ross University
School of Medicine
Class of 2008

Robert Nastasi, MS
PGY-1
SUNY Upstate Medical University
Class of 2008

Vivek Punjabi, MD
PGY-1
UMDNJ
Class of 2008
Acknowledgments

The author would like to acknowledge the following individuals for their work on the first edition:

Image Contributors:
Noah Craft, MD, PhD, DTM&H
William Herring, MD, FACR
Henry J. Feldman, MD

Reviewers:
Daniel Behroozan, MD
Paul Bellamy, MD
Jia-ling Chou, MD
Afshin Khatibi, MD
Rashmi Nadig
Pamela Nagami, MD
Braden Nago, MD
Frederick Ziel, MD
Preface

The principles learned in internal medicine are the fundamental core principles applied in clinical medicine as
well as the largest proportion of questions posed on the USMLE Step 2CK exam. In order to do well both on the
wards and on the Step 2CK exam, you must have a solid foundation in these principles. This guide has been
written as a high-yield resource to endorse the rapid recall of the essential facts in a well-organized and efficient
manner.

ORGANIZATION

All concepts are presented in a question and answer format that covers the key facts on hundreds of commonly
tested internal medicine topics that may appear on the USMLE Step 2CK exam. The material is divided into
chapters organized by internal medicine subcategories, along with vignettes at the end of each chapter that
incorporate the material with their clinical presentation and relevance.

This question and answer format has several advantages:

It provides a rapid, straightforward way for you to assess your strengths and weaknesses.
It allows you to efficiently review and commit to memory a large body of information.
The clinical vignettes incorporated expose you to the prototypical presentation of diseases classically
tested on USMLE Step 2CK.
It serves as a quick, last-minute review of high-yield facts.
Compact, condensed design of the book is conducive to studying on the go.

HOW TO USE THIS BOOK

This text is intended to be used not only to study for the USMLE Step 2CK exam but is also an essential tool
while on the internal medicine and medicine subspecialty rotations, and during medical school. Remember, this
text is not intended to replace comprehensive textbooks, course packets, or lectures. It is simply intended to serve
as a supplement to your studies during your internal medicine clinical rotation and throughout your preparation
for Step 2CK. We encourage you to begin using this book early in your third year to reinforce topics you
encounter while on the wards. Also, it is recommended that you cover up the answers (rather than just reading
both the questions and the answers) and quiz yourself or even your classmates. Carry the book in your white coat
pocket so that you can easily access study material during down time. However you choose to study, we hope you
find this resource helpful throughout your clinical years and during your preparation for USMLE Step 2CK. Best
of Luck!

Sarvenaz S. Saadat Mobasser, MD


Introduction

Important Lab Values


Blood Test Value
Albumin, serum 3.2-5.5 g/dL
Alkaline phosphatase 26-110 IU/L
Ammonia, plasma 17-60 μmol/L
Amylase, serum 25-125 IU/L
Bilirubin
Direct 0-0.2 mg/dL
Total 0-1.4 mg/dL
Calcium 9-10.6 mg/dL
Chloride 101-111 mEq/L
CO2 25-34 mEq/L
Cortisol, AM 6-28 μg/dL
Cortisol, PM 3-16 μg/dL
CPK 22-269 U/L
Creatinine 0.5-1.3 mg/dL
D-Dimer <0.5 μg/mL
ESR, female 0-20 mm/h
ESR, male 0-15 mm/h
Ferritin, female 10-1107 ng/mL
Ferritin, male 23-233 ng/mL
Folate 3-18.2 ng/mL
Glucose 70-115
Hematocrit, female 34%-44%
Hematocrit, male 39.5%-50%
Hemoglobin, female 11.5-15 g/dL
Hemoglobin, male 13.5-16.9 g/dL
Iron, female 37-170 μg/dL
Iron, male 49-181 μg/dL
Lactate dehydrogenase (LDH) 91-180 IU/L
Lipase 4-24 IU/L
Magnesium 1.8-2.5 mg/dL
Osmolality, serum 278-305 mOsm/kg
Osmolality, urine 50-1200 mOsm/kg
Phosphorus 2.5-4.6 mg/dL
Platelets 150-450,000
Potassium 3.3-4.8 mEq/L
Prealbumin 18-45 mg/dL
Protein, total 6.7-8.2 g/dL
PSA, age age 0-39 0-1.4 ng/mL
PSA, age age 40+ 0-2.8 ng/mL
Reticulocyte count 0.5%-1.5%
SGOT 10-42 U/L
SGPT <60 U/L
Sodium 135-145 mEq/L
T3 uptake 25%-38%
T4 total 0.7-2.1 ng/dL
Transferrin 212-360 mg/dL
TSH 0.5-5.0 μIU/mL
Uric acid 2.6-7.2 mg/dL
WBC 4500-10,500

Writing Notes

Daily progress note: This should be in SOAP format.

Subjective: In this area you should report any overnight events, how the patient is feeling today, any complaints
or problems the patient may be experiencing, and pertinent positives and negatives.

Objective: Any physical findings are reported in this section.

Vitals: temperature, max temperature, blood pressure, pulse, respiratory rate, oxygen saturation

Glucose (if patient is diabetic): Ins and Outs (Ins = IV fluids + po intake + any parenteral intake or blood products
over 24 hours and Outs = urine output + stool + other [NG tube, chest tube, drains, emesis])

Physical examination:

General: Patient’s general appearance


HEENT (head, eyes, ears, nose, throat)
Cardiovascular
Pulmonary
Abdomen
Extremities
Neurologic

Labs: Laboratory tests are reported here.


Chemistry 7.

Meds: Some people include a list of all the medication the patient is currently using. Assessment and plan: Write
a summary of the patient, their problem(s) and possible differentials. Then write the plan for each problem.

Sign your note

Example

S: Patient has no complaints today. She is no longer short of breath and was able to ambulate yesterday.

O: T: 36.8°C, Tmax 37°C, P: 70-85, BP: 128-148/68-80, RR: 20, O2 sat: 95-100%, I/O: 1500/2000

GEN: NAD (no apparent distress)

HEENT: PERRLA (pupils are equally round and reactive to light accommodation), EOMI (extraocular muscles
are intact), NCAT (normocephalic atraumatic)

CV: RRR no M/R/G (regular rate and rhythm with no murmurs, rubs, or gallops)

Pulm: CTA B (clear to auscultation bilaterally); no R/R/W (no rhonchi, rales, or wheezes)

Abd: S (soft)/NT (nontender)/ND (nondistended)/NABS (normal abdominal bowel sounds)

Ext: no C (clubbing)/C (cyanosis)/E (edema); no calf tenderness


Meds: Aspirin 81 mg daily

Albuterol nebs q4h

A/P: 35 y/o female with asthma exacerbation now improved and at baseline
1. Asthma: Patient improved with steroids and albuterol/atrovent treatments. Patient will be sent home with a
medrol pack and albuterol inhaler. Patient will also be sent home with a steroid inhaler.
2. Disposition: Patient will be discharged home today with follow-up in 1 week.

Greta Student, MS III

History and Physical Examination

Chief complaint (CC): Main problem that the patient is here for (eg, shortness of breath)

History of present illness (HPI): Include a chronologic history of the patient’s problems and prior treatments for
this problem as well as any other history that is pertinent. Describe symptoms in terms of onset, duration, quality
of discomfort, setting, instigating and relieving factors.

Past medical history (PMH): Include the patient’s medical history and be sure to ask about heart disease,
hypertension, diabetes, cancer, and any other pertinent history. The patient’s medication list can often serve as a
clue since patients will sometimes forget to mention medical problems that they have.

Surgical history (SH): Include all operations a patient has as well as when and why. Medication: List all the
patient’s medications as well as doses and frequency with which they are taken. Also ask the patient about any
possible over-the-counter medications and alternative meds.

Allergies: Name all drugs the patient is allergic to and what happened when they took the drug.

NKDA means “no known drug allergies”

Family history (FH): This should include the health, medical problems of the patient’s family including parents,
grandparents, siblings, and often, aunts, uncles, and cousins. Be sure to ask about heart disease, diabetes,
hypertension, hyperlipidemia, and cancer.

Social history (SH): This section includes the patient’s marital status, occupation, exercise history, sexual
history, diet, and tobacco use, drug use, and alcohol use.

Review of systems (ROS): Report all the pertinent positive and negative signs and symptoms that the patient
reports (eg, the patient denies any nausea, vomiting, diarrhea, chest pain, cough, travel history …)

Physical examination: Include all pertinent organs and systems:

Vital signs: Tmax, BP, HR, RR, O2 sat, Ins/Outs

General:

HEENT:

Neck:

Cardiovascular:

Pulmonary:

Abdominal:

Genitourinary:

Back:

Extremities:

Neurologic:

Labs and studies: Include all labs and studies that you have results for.
Assesment and plan: Write a summary of the patient’s problems and differential diagnoses as well as a plan for
each problem.

Sign your name at the bottom

Procedure Note

Whenever a procedure is done, a procedure note must be written in the chart. Always remember to get consent
from the patient before a procedure is done. Below is an example.

Procedure Note:

Procedure: Biopsy of left lower abdominal macule.

Indications: Rule out melanoma.

Consent: The risks, benefits, and possible side effects of the procedure including but not exclusive of pain,
bleeding, infection, and scar were explained to the patient who understands and wishes to have the procedure
done.

Preparation: The area was prepped and draped in a sterile fashion.

Anesthesia: The area was anesthetized with 10 cc of 2% lidocaine solution using a 30-gauge needle.
Procedure: A wide excision (1 cm on each side) of the macule was done using a number-15 blade. There was
minimal bleeding. The site of the excision was closed using 4-0 nylon sutures and the specimen was sent to
pathology for examination.

Complications: The patient tolerated the procedure with no complications.

Greta Student, MS III

How to Write a Prescription

Example

How to Admit a Patient

Admission Orders

Admit to:

Floor:
Service:

Medical student name:

Resident name:

Attending name:

Diagnosis:

Primary diagnosis:

Other diagnoses:

Condition:

Good, stable, fair, guarded, critical

Vitals:

Per routine (usually q2h in ICU and q4h on the floor)

q shift

q __ h

Activity:

Ad lib

Bed rest

To chair

Ambulate bid

Bathroom privileges

Fall risk

Nursing:

Neuro check q__h

Weigh daily

Pulse oximetry

Wound care

CALL MD for systolic blood pressure (SBP) >165 or <110; diastolic BP >100 or <60;

Pulse >100, T >38.5°C

Etc
Diet:

Regular

Diabetic

Low sodium

Low fat

Clear liquid

Soft

npo (nothing by mouth)

Ins and Outs: strict, per routine

IV fluids: eg, D51/2NS @ 100 cc/h

Drains: Foley, NG tube to suction, chest tube to suction

Medication:

Medication name, dose, route, frequency

Home medication should be written out

Antibiotics

Etc

Special: These are things you will usually need to think about.

DVT prophylaxis

Pain medications

Antiemetics

Antipyretics

Allergies:

NKDA (no known drug allergies)

Penicillin

Sulfa

Etc

Labs/studies:

CBC, electrolytes, BUN, Cr, ECG, radiology studies; other labs


Example

Admit to 2 North, Internal Medicine, Medical Student: Stew Dent;

Resident:

Dr. Smith; Attending: Dr. Bay

Diagnosis: Pneumonia

Condition: Fair

Vitals: Per routine

Activity: Bathroom privileges

Nursing: Pulse oximetry; call MD for systolic blood pressure (SBP) > 165 or < 110; diastolic BP > 100 or < 60;
Pulse > 100, Temp > 38.5; Pulse ox < 90%

Diet: Regular

Ins and outs: Strict

IV fluids: D5NS@120 cc/h

Meds: Ceftriaxone 2 g IV q24 hours

Azithromycin 500 mg IV q24 hours

Tylenol 650 mg po q6 hours prn mild pain or Temp > 38.5

Special: Sequential compression stockings

Allergies: NKDA

Labs/studies: PA and lateral CXR; sputum culture/Gram stain; CBC; electrolytes; BUN; Cr

Abbreviations You Should Know

AAA
abdominal aortic aneurysm

AAS
acute abdominal series

abd
abdomen

Abx
antibiotics
ac
before meals

ACLS
advanced cardiac life support

ACTH
adrenocorticotropic hormone

ADA
American Diabetes Association

ADH
antidiuretic hormone

ADL
activities of daily living

AFB
acid fast bacillus

AFP
alpha feto protein

AI
aortic insufficiency

AKA
above knee amputation

alk phos
alkaline phosphatase

ALL
acute lymphocytic leukemia

ALS
amytrophic lateral sclerosis

AMA
against medical advice

AMI
acute myocardial infarction

AML
acute myelogenous leukemia

ANA
antinuclear antibody

ant
anterior
AP
anteroposterior

APTT
activated partial thromboplastin time

AR
aortic regurgitation

ARDS
acute respiratory distress syndrome

ARF
acute renal failure

AS
aortic stenosis

ASA
aspirin

ASD
atrial septal defect

ASO
antistreptolysin O

ATN
acute tubular necrosis

AV
arteriovenous

AVN
atrioventricular node

B
bilateral

BBB
bundle branch block

BE
barium enema

BIB
brought in by

bid
two times per day

BKA
below knee amputation
BM
bowel movement; bone marrow

BPH
benign prostatic hypertrophy

BRBPR
bright red blood per rectum

BRP
bathroom privileges

BS
blood sugar; breath sounds

BUN
blood urea nitrogen

Bx
biopsy

c
with

Ca
calcium

CA
cancer, carcinoma

CABG
coronary artery bypass graft

CAD
coronary artery disease

cath
catheter

CBC
complete blood count

CBG
capillary blood gas

CC
chief complaint

CEA
carcinoembryonic antigen

CF
cystic fibrosis
CHF
congestive heart failure

CK-MB
creatinine kinase-myocardial band

CLL
chronic lymphocytic leukemia

CML
chronic myelogenous leukemia

CMV
cytomegalovirus

CN
cranial nerves

CNS
central nervous system

CO
cardiac output

c/o
complains of

COPD
chronic obstructive pulmonary disease

CP
chest pain

CPAP
continuous positive airway pressure

CPK
creatinine phosphokinase

CPR
cardiopulmonary resuscitation

CRF
chronic renal failure

C and S
culture and sensitivity

CSF
cerebrospinal fluid

CT
computerized tomography
CTAB
clear to auscultation bilaterally

CV
cardiovascular

CVA
cerebrovascular accident

CVAT
costovertebral angle tenderness

CVP
central venous pressure

CXR
chest x-ray

D51/2NS
5% dextrose in half normal saline

D5W
5% dextrose in water

DA
dopamine

D/C
discharge, discontinue

Ddx
differential diagnosis

DI
diabetes insipidus

DIC
disseminated intravascular coagulation

DIP
distal interphalangeal joint

DJD
degenerative joint disease

DKA
diabetic ketoacidosis

DM
diabetes mellitus

DNR
do not resuscitate
DOA
dead on arrival

DOE
dyspnea on exertion

DT
delirium tremens

DTR
deep tendon reflexes

DVT
deep vein thrombosis

Dx
diagnosis

EBL
estimated blood loss

ECT
electroconvulsive therapy

EEG
electroencephalogram

EGD
esophagogastroduodenoscopy

EKG
electrocardiogram

EMG
electromyelogram

ENT
ears, nose, and throat

EOMI
extraocular muscles intact

ERCP
endoscopic retrograde cholangiopancreatography

ESR
erythrocyte sedimentation rate

ETOH
alcohol, ethanol

ETT
endotracheal tube
FB
foreign body

FBS
fasting blood sugar

f/c
fever and chills

FEV1
forced expiratory volume in 1 second

FFP
fresh frozen plasma

FH
family history

FRC
functional residual capacity

FTA-ABS
fluorescent treponemal antibody absorption (syphilis)

FTT
failure to thrive

f/u
follow-up

FUO
fever of unknown origin

FVC
forced vital capacity

fx
fracture

GC
gonococcus, gonorrhea

GERD
gastroesophageal reflux disease

GI
gastrointestinal

GU
genitourinary

HA
headache
HBsAg
hepatitis B surface antigen

HBV
hepatitis B virus

Hct
hematocrit

HDL
high-density lipoprotein

HEENT
head, eyes, ears, nose, throat

Hgb
hemoglobin

HIV
human immunodeficiency virus

HLA
histocompatablility locus antigen

h/o
history of

HO
house officer

HOB
head of bed

HPI
history of present illness

HSM
hepatosplenomegaly

HTN
hypertension

Hx
history

ICU
intensive care unit

I&D
incision and drainage

IDDM
insulin-dependent diabetes mellitus
Ig
immunoglobulin

IM
intramuscular

INH
isoniazid

I&O
intake and output

ITP
idopathic thrombocytopenic purpura

IVF
intravenous fluids

IVP
intravenous pyelogram

JVD
jugular venous distention

KUB
kidney ureter, bladder x-ray

LAD
left axis deviation (lymphadenopathy)

LAE
left atrial enlargement

LAP
left atrial pressure

LCM
left costal margin

LDH
lactate dehydrogenase

LLE
left lower extremity

LLL
left lower lobe

LLQ
left lower quadrant

LMN
lower motor neuron
LOC
loss of consciousness

LP
lumbar puncture

LR
lactated ringers

LUE
left upper extremity

LUL
left upper lobe

LUQ
left upper quadrant

LVH
left ventricular hypertrophy

m
murmur

MAO
monoamine oxidase inhibitor

MAP
mean arterial pressure

MCH
mean cell hemoglobin

MCHC
mean cell hemoglobin concentration

MCP
metacarpophalangeal joint

MCV
mean corpuscular volume

MEN
multiple endocrine neoplasia

MI
myocardial infarction

MRSA
methicillin-resistant Staphylococcus aureus

MS
mitral stenosis, multiple sclerosis
MVA
motor vehicle accident

MVI
multivitamin

NAD
no apparent distress

ND
nondistended

NG
nasogastric tube

NIDDM
non-insulin-dependant diabetes mellitus

NKDA
no known drug allergies

npo
nothing by mouth

NS
normal saline

NSAID
nonsteroidal anti-inflammatory drug

NSR
normal sinus rhythm

NT
nontender

N/V
nausea and vomiting

OB
occult blood

OOB
out of bed

OR
operating room

PAC
premature atrial contraction

PAT
paroxysmal atrial tachycardia
PCWP
pulmonary capillary wedge pressure

PDA
patent ductus arteriosus

PE
pulmonary embolism

PEEP
positive end-expiratory pressure

PERRLA
pupils equally round and reactive to light

PFT
pulmonary function test

PMD
primary medical doctor

PMH
past medical history

PMI
point of maximal impulse

PMN
polymorphonuclear cell

PM&R
physical medicine and rehabilitation

PND
paroxysmal nocturnal dyspnea

po
by mouth

POD
post operative day

PR
per rectum

PRBC
packed red blood cells

PT
physical therapy, prothrombin time

pt
patient
PTCA
percutaneous transluminal coronary angioplasty

PTH
parathyroid hormone

PTT
partial thromboplastin time

PUD
peptic ulcer disease

PVC
premature ventricular contraction

PVD
peripheral vascular disease

qAC
before each meal

qd
daily

qid
four times per day

qod
every other day

q4h
every 4 hours

RA
rheumatoid arthritis

RAD
right axis deviation

RAE
right atrial enlargement

RBC
red blood cells

RDW
red cell distribution width

RHD
rheumatic heart disease

RLE
right lower extremity
RLL
right lower lobe

RLQ
right lower quadrant

RML
right middle lobe

r/o
rule out

ROM
range of motion

ROS
review of systems

RR
respiratory rate

RRR
regular rate and rhythm

RT
respiratory therapy

RTA
renal tubular acidosis

RTC
return to clinic

RUE
right upper extremity

RUL
right upper lobe

RUQ
right upper quadrant

RVH
right ventricular hypertrophy

s
without

SBE
subacute bacterial endocarditis

SBO
small bowel obstruction
SBP
subacute bacterial peritonitis

SEM
systolic ejection murmur

SGOT
serum glutamic-oxaloacetic transaminase

SGPT
serum glutamic-pyruvic transaminase

SIADH
syndrome of inappropriate antidiuretic hormone

SL
sublingual

SLE
systemic lupus erythematosus

SOB
shortness of breath

s/p
status post

stat
immediate

subQ
subcutaneous

Sx
symptoms

T
temperature

tab
tablets

TB
tuberculosis

TIA
transient ischemic attack

TIBC
total iron-binding capacity

tid
three times per day
TKO
to keep open

TLC
total lung capacity

TPN
total parenteral nutrition

TSH
thyroid-stimulating hormone

TTP
thrombotic thrombocytopenic purpura

TURP
transurethral resection of the prostate

TV
total volume

Tx
Treatment

UA
Urinalysis

UGI
upper gastrointestinal

UMN
upper motor neuron

URI
upper respiratory infection

US
Ultrasound

UTI
urinary tract infection

VC
vital capacity

VCUG
voiding cystourethrogram

VDRL
venereal disease research laboratory (syphilis test)

V/Q
ventilation perfusion scan
VSS
vital signs stable

WBC
white blood cells

WNL
within normal limits

y/o
years old

Common Formulas

Maintenance fluids per hour: 4:2:1 rule:

4 mL/kg up to 10 kg + 2 mL/kg from 11 to 30 kg + 1 mL/kg >30 kg


Example: A person weighing 100 kg should get

(4 × 10) + (2 × 20) + (1 × 70) = 40 + 40 + 70 = 150 cc/h

Maintenance fluids over 24 hours: 100:50:20 rule

100 mL/kg up to 10 kg + 50 mL/kg from 11 to 30 kg + 20 mL/kg >30 kg

Anion gap: Na − (Cl + HCO3)

Osmolality: 2Na + glucose/18 + BUN/2.8

Creatinine clearance, also known as glomerular filtration rate (GFR):

Corrected Na: Na + [(glucose − 100) × 0.016]

Corrected total calcium: [0.8 × (normal albumin − measured albumin)] + Ca

Aa gradient: [(713 × FIO2) − (PaCO2 /0.8)] − PaO2 = 150 − (PaCO2 /0.8)] − PaO2
Anion gap: Na − Cl + HCO3 (normal value is between 8 and 12 mEq/L)
MAP (mean arterial pressure): diastolic BP + [(systolic BP − diastolic BP)/3]
Cerebral perfusion pressure: MAP − ICP (intracranial pressure)

Statistics

Incidence: Number of new cases of a disease in a population in a certain period of time

Prevalence: Number of existing cases of a disease in a population at a specific time point

Sensitivity: This determines how well the test is able to detect disease.

Specificity: This determines how well the test detects the absence of disease.

Positive predictive value (PPV): Test precision or the probability that a patient truly has the disease when they
test positive. Calculation: true positive/all positive.

Negative predictive value (NPV): Probability that a patient truly does not have the disease when they test
negative. Increased sensitivity increases NPV, and the lower the prevalence of a disease, the higher the NPV.
Calculation: true negative/all negative.

True positive: Patient with disease and positive test

False positive: Patient without disease and positive test

True negative: Patient without disease and negative test

False negative: Patient with disease and negative test

Sensitivity: A/A+C
Specificity: D/B+D

PPV: A/A+B

NPV: D/D+C

Relative risk: [A/(A+B)]/[C/(C+D)]


Odds ratio: AxD/CxB

Absolute risk: (incidence of disease in exposed) – (incidence of disease in unexposed)

Number needed to treat (NNT): Number of patients that need to be treated in order to prevent one negative
outcome. Calculation: 1/absolute risk

Length time bias: Screening tests will tend to be able to detect cases of slowly progressive disease much better
than rapidly progressive diseases, just because of the nature of having a longer asymptomatic period

Lead time bias: Screening tests detects disease before symptomatic phase, increasing the time between diagnosis
and death.

Likelihood ratio: true positive/false positive

Null hypothesis: The statement that the thing being tested is not associated with the outcome

Type I error (α): probability of detecting a difference when one does not actually exist (for example concluding
that a drug works when it actually does not)

Type II error (β): Probability of not detecting a difference when on does actually exist (for example concluding
that a drug does not work when it actually does)

Power: Probability of NOT detecting a difference when one actually does not exist (eg, concluding that a drug
does NOT work and it actually does NOT work). Power is increase by larger sample size.

Calculation: 1 – type II error

p value: probability that the results of a study could happen by chance alone. Generally p < 0.05 is considered
statistically significant.

Precision (reliability): reproducibility of the results of a test

Validity (accuracy): whether the test actually correctly measures what it is trying to measure

Study Types

Randomized controlled: Subjects are blindly assigned to groups being studied. (Eg, if you are studying a
cholesterol drug, patients are randomly assigned to the treatment group and placebo group.)

Cohort study: Exposed subject are identified and followed for a certain time to study disease outcome.

Case-control study: Identify cases and non-cases and studied retrospectively to find possible risk factors.
Experimental Errors

Recall bias: Overestimation or underestimation of risk factors due to the fact that patients may not recall
accurately. Relevant to retrospective studies.

Interviewer bias: Interpretation of data being skewed due to the scientist’s personal bias. This occurs when the
study is not a blinded study.

Unacceptability bias: Patients may not report certain information because they feel ashamed or want to please the
scientist.
CHAPTER 1
The Basics

QUICK RADIOLOGY

Chest X-ray

What is the first thing that you should check when evaluating a radiographic study?
Check the name of the patient as well as the date and medical record number.

How can you determine if the chest x-ray (CXR) is adequate?


1. Penetration: Disk spaces can be seen without distinguishing the details of the spine.
2. Inspiratory effort: Diaphragm anteriorly should be below rib 7.
3. Rotation: Spinous processes of thoracic vertebrae should be midway between clavicles.

What is a posterior-anterior (PA) film?


Posterior-anterior film: The x-ray is shot from the back of the patient to the plate in front of the patient.

What is an anterior-posterior (AP) film?


Anterior-posterior film: The x-ray is shot from the front of the patient to the back of the patient.

When is an AP film appropriate?


When a patient is bed bound

How is the image altered in an AP film?


The heart appears large.

How should you approach reading a CXR?


Remember A, B, C, D plus lungs and soft tissue
Airway: Trachea should be midline.
Bones: Check for any bony defects, fractures, osteolytic lesions.
Cardiac: The heart should be less than ½ the width of the chest.
Diaphragm: There should be no blunting of the costophrenic angles.
No free air should be seen under the hemidiaphragm.
Lungs: Look for any nodules, opacification, bronchial markings.
Soft tissue: Look for any lesions, lymphadenopathy, masses.

What structure does each of the following types of infiltrates obscure?


What classic features are seen on a chest x-ray with congestive heart failure?
Cephalization of vessels; curly B lines

Name the parts of the CXR shown below?

(Reproduced, with permission, from William Herring, MD, FACR; Radiology Residency Program Director
at Albert Einstein Medical Center in Philadelphia, PA. Available at: http://www.learningradiology.com)
1. Sharp costophrenic angle
2. Right atrium
3. Hilum and main bronchus
4. Superior vena cava
5. Trachea (midline)
6. Aortic arch
7. Left atrium
8. Left ventricle

Other Radiologic Studies

What is a kidneys, ureter, and bladder (KUB)?


X-ray which looks at the kidney, ureter, and bladder

What structures do computed tomographic (CT) scans visualize best?


CT scans visualize bone best and can identify acute bleeds.

What structures does a magnetic resonance imaging (MRI) visualize best?


Soft tissue

Name the radiographic study you would use to evaluate each of the following:

Name what each of the following radiographic findings is most commonly indicative of:

QUICK EKG INTERPRETATION

Figure 1-1 Parts of the EKG. (Reproduced, with permission, from Tintinalli JE. Emergency Medicine: A
comprehensive study guide. 6th ed. New York: McGraw-Hill, 2004:181.)

Step 1: Calculate the rate (Fig 1-2).


Rate = beats per minute.
The easy way to calculate the rate is 300/(# big boxes between two QRS complexes) or 300, 150, 100, 75,
60, 50.
300/3 = 100.
In this example, the rate is about 100 beats per minute.

Figure 1-2 Rate calculation example. In this EKG, the rate is 300/3 = 100 beats/min.

Step 2: Calculate the rhythm (Fig 1-3).


Ask the question: Is there a P wave before each QRS? And, are the P waves of the same morphology? If yes,
then the rhythm is sinus.
In the example, there is a P wave of the same morphology before each QRS, which indicates that the patient
is in sinus rhythm. If there were a lack of P waves or a disorganized rhythm, a differential diagnosis, which
you will find in Chapter 2, would come into play.

Figure 1-3 Sinus rhythm.

Step 3: Determine the axis (Fig 1-4).


Rules of thumb:
If I and aVF are positive, then axis is normal.
If I is positive and aVF is negative, check lead II.
If lead II is positive, then the axis is normal.
If lead II is negative, then there is left axis deviation.
If I is negative and aVF is positive, then there is right axis deviation.

Figure 1-4

Step 4: Evaluate the intervals.


One large box = 0.20 seconds.
One small box = 0.04 seconds.
Normal measurements:
P wave <0.12 seconds
PR interval (beginning of P wave to beginning of QRS complex) 0.12-0.2 seconds
QRS interval (beginning of the Q to the end of the S wave) <0.12 seconds
QT interval (beginning of QRS to the end of the T wave) 0.33-0.47 seconds
If PR <0.12, then junctional rhythm or bypass tract
If PR >0.2, then atrioventricular (AV) block
If QRS >0.12, then either a left bundle branch block (LBBB), right bundle branch block (RBBB), or a
nonspecific conduction delay

Step 5: Check for hypertrophy.


Atrial hypertrophy
Right atrium: tall P waves in II, III, and aVF or V1 or V2
Left atrium: notched P waves in limb leads
Ventricular hypertrophy
Left ventricular hypertrophy: height of S (mm) in V1 + height of R (mm) in
V5 >35 mm
Right ventricular hypertrophy: height of R (mm)/height of S (mm) in V1 >1

Step 6: Look for ischemic changes.


ST elevation or depression
T-wave inversion
Q waves indicating old infarct
PREVENTATIVE MEDICINE

Adult Immunizations

Table 1-1 Adult Immunization Recommendations


PREVENTATIVE SCREENING

Table 1-2 Preventative Screening Recommendations

FLUIDS AND ELECTROLYTES

Fluids

What percentage of body mass is water?


50% to 70%

In what two compartments is body water stored and what is the portion in each?
Intracellular (⅔)
Extracellular (⅓)
How is extracellular fluid separated?
Intravascular (¼); extravascular or interstitial (¾)

What percentage of body mass does intracellular water account for?


40%

What percentage of body mass does extracellular water account for?


20%

What percentage of body mass does blood account for?


About 7%

What physical examination signs can be used to assess volume status?


Skin turgor, mucous membranes, pulse, urine output, acute weight change

What are signs of hypovolemia?


Tachycardia, tachypnea, dry mucous membranes, decreased urine output, decreased blood pressure,
decreased skin turgor

What is normal urine output in an adult?


Above 30 cc/h; on average 0.5-1 cc/kg/h

How do you calculate maintenance fluids per hour?


4/2/1 rule:
4 mL/kg (up to 10 kg); 2 mL/kg(from 11 to 30 kg); 1 mL/kg (>30 kg)

How do you calculate maintenance fluids per day?


100/50/20 rule:
100 mL/kg (up to 10 kg); 50 mL/kg (from 11 to 30 kg); 20 mL/kg (>30 kg)

What are each of the following IV fluids comprised of?

What are the two most commonly used maintenance fluids?


D5 ½ NS or D5 ½ NS with 20 mEq K

What type of IV fluids should be given for fluid resuscitation?


NS or lactated Ringer because they are isotonic

Electrolytes

Hyperkalemia

What is the normal range for potassium?


3.5-5.0 mEq/L

What are the causes of hyperkalemia?


Increased load vs decreased excretion:
Increased load: exogenous K+ ingestion, blood transfusion, tissue injury (rhabdomyolysis, burns), acidosis,
hypoaldosteronism
Decreased excretion: renal failure, K+-sparing diuretics

What is pseudohyperkalemia?
Elevated K+ in a blood sample due to hemolysis

What are the signs and symptoms?


Muscle weakness, paresthesias, areflexia, bradycardia, respiratory failure, EKG changes

What are the characteristic EKG findings?


Peaked T waves
Prolonged PR interval, widening of QRS, P-wave loss, (Fig 1-5)

Figure 1-5 Peaked T waves (arrow), widened QRS (double arrow), and subtle flattening of the P waves are
seen in this patient with a serum K of 7.1. (Reproduced, with permission, from Knoop KJ, Stack LB, Storrow
AB, et al. Atlas of Emergency Medicine. 3rd ed. New York: McGraw-Hill; 2010:778.)

Above what level are symptoms usually seen?


K+ >6.5.
How is hyperkalemia treated?
1. Protect cells by increasing membrane threshold: calcium gluconate (cardioprotective)
2. Drive K+ into cells: sodium bicarbonate, insulin with glucose
3. Excrete K+: kayexalate (binds K+), furosemide, dialysis

What is the mnemonic for treatment of hyperkalemia?


C BIG K Drop:
Calcium gluconate
Bicarbonate
Insulin
Glucose
Kayexalate
Dialysis
r
o
p

What acid-base disturbance can lead to hyperkalemia?


Acidosis

Hypokalemia

What are some causes of hypokalemia?


Vomiting, diarrhea, nasogastric (NG) tube suction, diuretic use (thiazides are a common culprit), insulin,
amphotericin, hypomagnesemia

What are the signs and symptoms of hypokalemia?


Nausea, vomiting, weakness, paresthesias, hyporeflexia, ileus, digoxin sensitivity, v-tach, and EKG changes

What acid-base disturbance can cause hypokalemia?


Alkalosis

What are the characteristic EKG findings of hypokalemia?


T-wave depression, U waves, prolonged QT and ST depression (Fig 1-6)

Figure 1-6 This EKG demonstrates multiple findings consistent with hypokalemia: flattened T waves (gray
arrowhead), U waves (black arrowhead), prolonged QT (QU) intervals (double arrow), and ST-segment
depression (arrow). This patient’s potassium level was 1.9. (Reproduced, with permission, from Knoop KJ,
Stack LB, Storrow AB, et al. Atlas of Emergency Medicine. 3rd ed. New York: McGraw-Hill; 2010:777.)

How is severe hypokalemia acutely treated?


IV KCl

What is the major side effect of IV potassium?


Burning sensation at IV site through which it is being administered

How can the burning be avoided when administering IV potassium?


Slow infusion usually not more than 10 mEq/h

How can mild or chronic hypokalemia be treated?


Oral KCI supplementation or potassium-rich foods

What electrolyte level should be checked in a patient with hypokalemia?


Magnesium; hypomagnesemia can precipitate hypokalemia.

What medication can be used to treat hypokalemia?


Potassium-sparing diuretic (eg, spironolactone)

Hypercalcemia

What is the normal range for calcium?


9.0-10.6 (serum calcium)

What are the causes for hypercalcemia?


Mnemonics CHIMPANZEES:
Calcium supplementation
Hyperparathyroidism
Iatrogenic
Milk alkali syndrome
Paget disease
Addison disease
Neoplasm
Zollinger-Ellison syndrome (MEN I)
Excess vitamin A
Excess vitamin D
Sarcoidosis

What is a common iatrogenic cause of hypercalcemia?


Thiazide diuretics

What are the symptoms of hypercalcemia?


Stones (kidney stones), bones (bone pain), abdominal groans (abdominal pain, nausea, constipation), and
psychiatric overtones (confusion, concentration, fatigue)

What does the EKG look like?


Prolonged PR interval, short QT interval

What is the treatment?


IV hydration, loop diuretic (furosemide)

How is it treated in refractory cases?


Calcitonin, pamidronate, etidronate, glucocorticoids, plicamycin, dialysis

Hypocalcemia

What are the causes of hypocalcemia?


Renal failure, vitamin D deficiency, pancreatitis, diuretics, hypomagnesemia, parathyroidectomy

What can cause a pseudohypocalcemia?


Hypoalbuminemia

How can the true calcium level be calculated in hypoalbuminemia?


0.8 × (4 − plasma albumin level) + calcium level = true calcium level

What are the two classic signs of hypocalcemia?


Trousseau and Chvostek signs

What is Trousseau sign?


Carpal spasm with arterial occlusion using a blood pressure cuff

What is Chvostek sign?


Facial spasm with tapping of the facial nerve

What are some other signs and symptoms of hypocalcemia?


Tetany, seizures, perioral paresthesias, altered mental status, fatigue, weakness, EKG changes, abdominal
cramping, convulsions

What is the classic EKG finding with hypocalcemia?


Prolonged QT interval

What is the treatment for acute hypocalcemia?


IV calcium gluconate

What is the treatment for chronic hypocalcemia?


Vitamin D with oral calcium tablets

Hypernatremia

What is the normal range for sodium?


135-145 mEq/L

What are the causes of hypernatremia?


Hypovolemia: decreased oral intake of water secondary to illness or altered mental status; increased water
loss such as diuresis, vomiting, diarrhea, hyperaldosteronism
Hypervolemia: hypertonic fluid administration, excess ingestion of salt, Cushing syndrome, Conn
syndrome
Isovolemia: diabetes insipidus, skin loss

How do you calculate water deficit in hypernatremia?


0.6 × weight (kg) × (measured Na/normal Na) − 1

What are the signs and symptoms of hypernatremia?


Seizure, coma, ataxia, lethargy, irritability, spasticity, edema
What is the treatment for hypernatremia?
Treatment is dependent on each of the following underlying causea of hypernatremia:
Hypovolemia: Replace fluid with isotonic saline. Replace ½ of water deficit in first 24 hours and ½ over the
next 48-72 hours.
Hypervolemia: Loop diuretics to increase sodium excretion and fluid replacement with ½ NS
Isovolemia: Fluid replacement with ½ NS (½ water deficit in first 24 hours and ½ over next 48-72 hours). If
patient has central diabetes insipidus, give vasopressin.

What is the risk of rapid correction of hypernatremia?


Cerebral edema

What is the maximum rate at which plasma osmolality can be corrected?


2 mOsm/kg/h

What is the maximum rate at which sodium concentration can be corrected safely?
1 mEq/L/h

Hyponatremia

What is the differential diagnosis of hyponatremia?


See Fig 1-7.

Figure 1-7
What is pseudohyponatremia?
There is no true sodium deficit, but appears to be because the serum is occupied by lipids or protein.

What is factitious hyponatremia?


Normal total body sodium but decreased serum sodium because of an osmotic flow of water into serum
secondary to excess glucose or mannitol in the serum

How is serum osmolality calculated?


2 × Na + blood urea nitrogen (BUN)/2.8 + glucose/18

How is hyponatremia evaluated?


See Fig 1-8.

Figure 1-8

What are the signs and symptoms of hyponatremia?


Seizure, coma, lethargy, weakness, nausea, vomiting, ileus, altered mental status

What is the treatment for hypotonic hypovolemic hyponatremia?


Correct the underlying disorder and fluid resuscitation with IV normal saline (NS).

What is the treatment for hypotonic hypervolemic hyponatremia?


Fluid restriction. Diuretics like furosemide are helpful.

What is the treatment for hypotonic isovolemic hyponatremia?


Treat the underlying cause. Fluid restriction.

What is the maximum speed at which hyponatremia can be safely corrected?


1 mEq/h

What can happen if sodium is corrected too quickly?


Central pontine myelinolysis

Hyperphosphatemia

What is the normal range of phosphate?


2.5-4.5 mg/dL

What are the most common causes of hyperphosphatemia?


Iatrogenic

What are other causes of hyperphosphatemia?


Hypoparathyroidism, hypocalcemia, renal failure, rhabdomyolysis, tumor lysis

What are the signs and symptoms of hyperphosphatemia?


Heart block, ectopic soft tissue calcification

What is the treatment for hyperphosphatemia?


Aluminum hydroxide; sevelamer hydrochloride (Renagel); calcium acetate (Phoslo); lanthanum carbonate
(Fosrenol); insulin and glucose; in severe cases—dialysis

Hypophosphatemia

What are the causes of hypophosphatemia?


Hyperparathyroidism, diuresis, decreased po (oral) intake, renal tubular acidosis, hypokalemia,
hypomagnesemia, acetazolamide, glucose, and insulin

What are the signs and symptoms of hypophosphatemia?


Proximal muscle weakness, ataxia, rhabdomyolysis, paresthesias, hemolytic anemia, seizure

What is the treatment for hypophosphatemia?


Potassium phosphate of sodium phosphate supplementation

Hypermagnesemia

What is the normal range of magnesium?


1.5-2.5 mEq/L

What are the causes of hypermagnesemia?


Iatrogenic, renal failure, tumor lysis

What are the signs and symptoms of hypermagnesemia?


Weakness, fatigue, ↓ deep tendon reflexes, hypotension, paresthesias, coma, decreased respirations

What is the treatment for hypermagnesemia?


Calcium gluconate and dialysis in refractory cases
Hypomagnesemia

What are the causes of hypomagnesemia?


Malabsorption, diarrhea, vomiting, NG tube suction, alcoholic patient, diuresis, hypokalemia or
hypocalciuria induce hypomagnesemia, insulin and glucose administration, short bowel syndrome, total
parenteral nutrition, hypercalcemia

What are the signs and symptoms of hypomagnesemia?


Weakness, hyperreflexia, seizure, altered mental status, torsades de pointes, atrial fibrillation, hypokalemia,
and hypocalcemia refractory to replacement

What EKG changes would you expect to see in a patient with hypomagnesemia?
Prolonged QT and PR intervals, flattened T waves; may see torsades de pointes

What is the treatment for hypomagnesemia?


Magnesium sulfate IV

What other electrolyte abnormalities are related to hypomagnesemia?


Hypokalemia and hypocalcemia—if magnesium is low, these electrolyte abnormalities become refractory to
treatment.

Name the electrolyte abnormality associated with the following EKG (Fig. 1-9).
Hyperkalemia with peaked T waves

Figure 1-9 Peaked T waves. (Reproduced, with permission, from Knoop KJ, Stack LB, Storrow AB, et al.
Atlas of Emergency Medicine. 3rd ed. New York: McGraw-Hill, 2010: 778. Photo contributor: R. Jason
Thurman, MD.)

NUTRITION


Name the type of diet you would order for each of the following types of patients:

Normally, what is the daily protein requirement for an adult?


1 g/kg per 24 hours

Normally, what is the daily carbohydrate requirement for an adult?


35 kcal/kg per 24 hours

How many kilocalories (kcal) in 1 g of fat?


9 kcal

How many kcal in 1 g of carbohydrate?


4 kcal

How many kcal in 1 g of protein?


4 kcal

What lab test is used to determine chronic nutritional status?


Albumin, since the half-life is about 20 days

What lab test is used to determine acute nutritional change?


Prealbumin, since the half-life is about 3 days

Name the fat-soluble vitamins.


D, E, A, K (DEAK)

Where are the fat-soluble vitamins absorbed?


In the terminal ileum

Where is vitamin B12 absorbed?


In the terminal ileum

What must bind B12 in order for it to be absorbed?


Intrinsic factor

Where is iron absorbed?


Jejunum

Where is intrinsic factor produced?


It is produced by the gastric parietal cells.

Name the effect on the body with each of the following deficiencies:

What are the vitamin K-dependent clotting factors?


Factors 2, 7, 9, 10

What is TPN?
Total parenteral nutrition

What are the indications for TPN use?


Npo for >7 days
Pancreatitis
Anorexia
Enterocutaneous fistula
Ileus that is not resolving
Burn patients
Patients unable to take food by mouth

What are the three main components of TPN?


1. Amino acids
2. Dextrose
3. Fat

What percentage of TPN is fat?


10% (20% in the form of intralipid)

What percentage of TPN calories comes from dextrose?


50% to 70%

What percentage of total calories comes from fat?


30% to 50%

What percentage of total calories comes from amino acids (or protein)?
10% to 20%

How is basal energy expenditure (BEE) calculated in a male?


66 + (13.7 × weight [kg]) + (5 × height [cm]) − (6.8 × age)

How is BEE calculated in a female?


65 + (9.6 × weight [kg]) + (1.8 × height [cm]) − (4.7 × age)

What are the complications of TPN?


Fatty liver, acalculous cholecystitis, hyperosmolality, line infection, refeeding syndrome, cholestasis

What is refeeding syndrome?


Low potassium, phosphate, and magnesium after refeeding of a patient who was previously starving

What is PPN?
Partial parenteral nutrition

When would PPN be used?


In patients who can tolerate some nutrition orally and only need some supplementation

A patient who becomes jaundiced while on TPN or PPN most likely has what condition?
Cholestasis

BLOOD PRODUCTS AND TRANSFUSIONS

What blood products are measured when checking a complete blood count (CBC)?
White blood cells, hemoglobin, hematocrit, platelets, red blood cells

Name the blood products described below:


Name the blood tests described below:

What is the problem with using FFP in patients on Coumadin?


It will reverse the anticoagulation quickly; however, it is more difficult to get the patient back to a
therapeutic level.

What else can be used to reverse anticoagulation in a patient on Coumadin?


Vitamin K, but much slower than FFP

Which foods have vitamin K?


Leafy green vegetables

What is involved in normal coagulation?


Damage to the endothelium leads to platelet binding and aggregation; coagulation factors then help lay down
fibrin to form and stabilize a clot.

What is a therapeutic INR level for a patient on Coumadin for each of the following underlying conditions:

When should you consider a blood transfusion in a normal, healthy patient?


When hemoglobin drops below 8

When should you consider a blood transfusion in a patient with coronary artery disease?
When hemoglobin drops below 10

How does 1 U of PRBC affect the hemoglobin and hematocrit?


1 U should increase the hemoglobin by 1 g/dL and hematocrit by 3%.

What is the formula for converting hematocrit to hemoglobin?


Hematocrit ÷ 3 = hemoglobin.

What study should be ordered if you are considering transfusing a patient?


Type and cross

What is a type and cross?


The patient’s RBCs are cross-matched to available donor blood for transfusion. In this process, the patient’s
serum is checked for preformed antibodies to the RBCs of the donor.

What is a type and screen?


The patient’s blood type and Rh antigen are determined and the donor’s blood is screened for common
antibodies.

What blood type is considered the universal donor?


O

What blood type is considered the universal recipient?


AB

What are the two main complications of a blood transfusion that a patient should know about before
consenting for a transfusion?
Possibility of acquiring an infectious disease and possibility of rejection

What is the most common cause of rejection during a blood transfusion?


Clerical error leading to ABO incompatibility

What are the most common signs and symptoms seen of an acute rejection?
Fever, chills, tachycardia, shock, acute renal failure

What is the treatment of a rejection to a blood transfusion?


Stop the transfusion! IV fluid resuscitation and make sure the patient has good urine output. If urine output is
not sufficient, furosemide (Lasix) can be administered.
After a transfusion, what would you expect to happen to the ionized calcium in the blood?
It decreases because of the preservative citrate used to store blood.

What is the most common transfusion-related infection?


Hepatitis

What is the risk of infection with hepatitis B from a blood transfusion?


1 in 200,000 U of blood

What is the risk of infection with hepatitis C from a blood transfusion?


1 in 2 million U of blood

What is the risk of getting infected with human immunodeficiency virus (HIV) from a blood transfusion?
1 in 2 million U of blood

How long can PRBCs be stored?


6 weeks

What is the life span of a RBC?


120 days

What is thrombocytopenia?
Platelet count <200,000

At what platelet count is there a risk for spontaneous intercranial bleeding?


Platelet count <20,000

In an actively bleeding patient or a patient who is preoperative, what should the platelet count be?
A minimum of 50,000

In what cases of thrombocytopenia are platelets not transfused?


Do not transfuse platelets in patients with thrombotic thrombocytopenic purpura (TTP), idiopathic
thrombocytopenic purpura (ITP), and disseminated intravascular coagulation (DIC), because platelet
transfusion will only perpetuate the problem. Platelets are only transfused if the patient is actively bleeding.
CHAPTER 2
Cardiology

HYPERTENSION

How is hypertension defined?


Prehypertension: 120-139/80-89
Stage 1: 140-159/90-99
Stage 2: >160/>100

What is the most common cause of hypertension?


90% is essential or idiopathic.

What are some secondary causes of hypertension?


1. Cardiovascular: coarctation of aorta, aortic regurgitation
2. Renal: renal artery stenosis, polycystic kidney disease, glomerular disease
3. Endocrine: eclampsia, pheochromocytoma, primary hyperaldosteronism (Cushing and Conn)

When would you suspect a possible secondary cause?


Resistant hypertension despite multiple medications, diagnosis of hypertension at age <25 or >55

Define hypertensive urgency.


Systolic >180, diastolic >120 with no end-organ failure

Define hypertensive emergency.


Also known as malignant hypertension; systolic >180, diastolic >120; with end-organ failure

What are the signs and symptoms of malignant hypertension?


1. Change in mental status
2. Papilledema
3. Anuria (sign of renal failure)
4. Heart failure
5. New-onset neurologic change

What is the treatment for malignant hypertension?


Nitroprusside or nitroglycerine

In malignant hypertension, by how much should the blood pressure be reduced in 1 hour?
Do not decrease by more than ¼ within 2-6 hours, otherwise the patient will be at risk for a stroke.

How do you calculate mean arterial pressure (MAP)?


(2 × diastolic + systolic)/3

What hypertensive treatment is favorable for a patient with each of the following comorbidities?

What are the relative contraindications for each of the following treatments?

What are the most common side effects for each of the following treatments?

Which three drugs are proven to reduce morbidity and mortality?


1. Beta-blockers
2. Thiazide diuretics
3. ACE inhibitors

HYPERLIPIDEMIA

When should a patient with no family history be screened for hyperlipidemia?


Men aged 35; women aged 45

How often should a patient with previously normal lipids be rechecked for hyperlipidemia?
Every 5 years

What should the low-density lipoprotein (LDL) level be in a patient with no or one risk factor(s) for
coronary artery disease (CAD)?
<160

What is the goal LDL for a patient with known CAD or CAD equivalents?
<70

What is the goal LDL for patient with no known CAD but with two or more risk factors?
<130

What are some examples of CAD equivalents?


Diabetes, peripheral artery disease, abdominal aortic aneurysm, more than a 20% 10-year risk of developing
coronary artery disease

What is a protective factor in terms of hyperlipidemia?


High-density lipoprotein (HDL) >60

What is the mechanism for each of the following lipid-lowering agents?

What should you be concerned about in a patient on a statin complaining of muscle pain?
Rhabdomyolysis

How can you test for this?


Check creatine kinase (CK)

What side effect of statins should you screen for?


Elevation in alanine aminotransferase (ALT)

CORONARY ARTERY DISEASE

What is CAD?
Atherosclerosis leading to angina or MI

What are the risk factors for CAD?


Major
Family (hx) (MI before age 55 in a male or 65 in a female)
Diabetes
Smoking
Hyperlipidemia
Hypertension
Age: male >45, female >55
Minor
Obesity
Male sex
Postmenopausal female
Elevated plasma homocysteine

What lifelong treatment has been shown to decrease mortality in a patient with CAD?
Aspirin, beta-blocker, statin, ACE inhibitor

What is stable angina?


Substernal chest pain (may radiate as well to arms, jaw, and so forth) due to ischemia that occurs both
predictably and reproducibly at a certain level of exertion and relieved with rest/nitrates

What are some classic electrocardio-graphic (EKG) findings in a patient with angina?
ST depression or T-wave inversion

What is the treatment for acute angina?


Sublingual nitroglycerin up to three doses

What is the long-term treatment for angina?


Nitrates, aspirin, beta-blocker, statin, smoking cessation

What is unstable angina?


Angina occurring more frequently, unrelieved by nitroglycerin, or occurs at rest

How do you evaluate a patient with unstable angina?


EKG, cardiac enzymes, and, once stable, a cardiac stress test to risk stratify; angiography may be necessary

How should a patient with unstable angina be treated?


Hospitalization and treatment with nitroglycerin, aspirin, beta-blocker, ACE inhibitor, statin, heparin drip or
Lovenox while on a cardiac monitor

When is a coronary artery bypass graft (CABG) indicated?


Failure of medical treatment with severe three-vessel disease; multiple vessel disease in a diabetic patient; or
>50% stenosis of the left main artery; proximal significant left anterior descending (LAD) coronary artery
stenosis with left ventricular (LV) dysfunction

What is Prinzmetal angina?


Angina due to coronary vasospasm that is usually nonexertional but can be exertional. Angiography is
normal in these patients.

What is an MI?
Myocardial necrosis caused by ischemia

What are the classic symptoms of an MI?


Crushing, substernal chest pain described as chest tightness or pressure. It can radiate to the left arm, neck,
or jaw and can be associated with concomitant diaphoresis, shortness of breath, nausea, and vomiting.

What patients can present with nonclassic symptoms?


Diabetics and the elderly

What are the classic EKG changes associated with an MI?


ST elevation (Fig 2-1A) or depression, new left bundle branch block (LBBB), T-wave changes (Fig 2-1B)

Figure 2-1A

Figure 2-1B

What are defining factors for an MI?


Two of the following three being true:
1. New left bundle branch block
2. Chest pain >20 minutes
3. Elevated cardiac markers

What are the three different cardiac enzymes tested in a patient with chest pain?
Troponin, creatine kinase (CPK), and CK-MB (creatine kinase-MB)
How do the three cardiac enzymes differ in terms of elapsed time since an MI?
See Table 2-1.

Table 2-1 Cardiac Enzyme Elevation in an Acute Myocardial Infarction

How often should the cardiac enzymes be done?


Repeat every 6-8 hours for a 24-hour period.

What is the mnemonic for emergent treatment of an MI?


Be MONA:
Beta-blocker
Morphine
Oxygen
Nitroglycerin
Aspirin

When is thrombolysis indicated?


In an ST-elevation MI, within 12 hours of onset of chest pain

What are contraindications to thrombolytics?


Previous cerebral hemorrhage, known cerebral aneurysm or arteriovenous malformation (AVM), known
intracranial neoplasm, ischemic stroke in the last 3 months, aortic dissection, active bleeding, significant
closed head or facial trauma

What is a contraindication to the use of streptokinase specifically?


Cannot be used more than once in a 6-month period because of its immunogenicity

What are some possible post-MI complications?


New arrhythmias; Dressler syndrome; papillary muscle rupture; thromboembolism; CHF, ventricular septal
defect (VSD), myocardial rupture

What is Dressler syndrome?


An autoimmune process with the features of fever; pericarditis; elevated erythrocyte sedimentation rate
(ESR) that occurs 2-4 weeks after an MI

What is the treatment of Dressler syndrome?


Nonsteroidal anti-inflammatory drugs (NSAIDs) and aspirin

What physical examination finding is indicative of a papillary muscle rupture?


New mitral regurgitation

ARRHYTHMIAS

Define each of the following types of heart block:


See Fig 2-2A-D.

What is the treatment for each of the following types of heart block?

Name some medications that can lead to second-degree heart block?


Digoxin, beta-blockers, calcium channel blockers

What is the most common chronic arrhythmia?


Atrial fibrillation

Figure 2-2A-D (A) The PR interval is fixed (double arrows) and is >0.2 seconds, or five small blocks. (B)
The PR interval gradually increases (double arrows) until a P wave is not followed by a QRS and a beat is
“dropped” (brackets). The process then recurs. P waves occur at regular intervals, though they may be
hidden by T waves. (C) The PR interval is constant (double arrows) until the dropped beat (brackets). (D)
The P-P interval is uniform (lower double arrows) and the R-R interval is uniform (upper double arrows),
but the P waves and QRS complexes are disassociated. (Reproduced, with permission, from Knoop KJ, Stack
LB, Storrow AB, et al. Atlas of Emergency Medicine. 3rd ed. New York: McGraw-Hill; 2010:747-750.)

What is atrial fibrillation?


Irregularly irregular rhythm caused by disorganized electric activity of the atrium (Fig 2-3)

Figure 2-3

What is the mnemonic for some etiologies of atrial fibrillation?


PIRATES:
Pulmonary disease
Ischemia
Rheumatic heart disease
Anemia/Atrial myxoma
Thyroid
Ethanol
Surgery, Sepsis

What is the most common underlying cause of atrial fibrillation?


Chronic hypertension

What are some symptoms that patients with atrial fibrillation complain of?
Fatigue, light-headedness, palpitations

What is the major complication of atrial fibrillation if left untreated?


Embolization which often can lead to stroke

What are the treatments of atrial fibrillation?


Rate control with beta-blocker, calcium channel blocker (diltiazem), digoxin
Antiarrhythmic agents (if failure to rate control or symptomatic despite rate control)
Anticoagulation with Coumadin
In an unstable patient, synchronized cardioversion

What is atrial flutter?


Macroreentrant arrhythmia; atrial rates are typically between approximately 240 and 400 beats/min

What is the classic EKG pattern described in atrial flutter?


“Saw tooth” (Fig 2-4)

Figure 2-4

What is multifocal atrial tachycardia (MAT)?


Irregularly irregular rhythm caused by at least three sites of competing atrial activity

What is the classic EKG finding in MAT?


At least three different P-wave morphologies in the same lead (Fig. 2-5)

Figure 2-5

What medical condition is associated with MAT?


COPD

What is the treatment for MAT?


Treat the underlying cause.

What is a premature ventricular contraction (PVC)?


Ectopic beats of ventricular origin

What is the typical EKG finding in PVCs?


Wide QRS with no P wave

What is ventricular tachycardia (VT)?


More than three consecutive PVCs; sustained VT must last >30 seconds (Fig 2-6)

Figure 2-6 Ventricular tachycardia with capture beat. (Reproduced, with permission, from Knoop KJ, Stack
LB, Storrow AB, et al. Atlas of Emergency Medicine. 3rd ed. New York: McGraw-Hill; 2010:765. Photo
contributor: James V. Ritchie, MD.)

What is the possible complication of VT?


Ventricular fibrillation or cardiac arrest/hemodynamic collapse

What is the treatment for VT?


If the patient is asymptomatic and not hypotensive, treat with lidocaine or amiodarone; if the patient is
hypotensive or pulseless, treatment is defibrillation.

What is ventricular fibrillation?


Disorganized electric activity of the ventricle (Fig 2-7)

Figure 2-7

What is the treatment for ventricular fibrillation?


Emergent cardioversion

What is torsades de pointes?


Prolonged VT with rotation around the axis in a patient with a prolonged QT interval at baseline

What are the underlying causes of torsades de pointes?


Quinine, procainamide, intracranial bleed, tricyclics, phenothiazines, electrolyte abnormalities such as
hypomagnesemia, hypokalemia, hypocalcemia

What are the classic EKG findings in Wolff-Parkinson-White (WPW) syndrome?


“Delta” waves and short PR interval (Fig 2-8)

Figure 2-8 The PR interval is shortened (double arrow) and a delta wave (upsloping initial QRS segment) is
seen (arrow, shaded area). (Reproduced, with permission, from Knoop KJ, Stack LB, Storrow AB, et al. Atlas
of Emergency Medicine. 3rd ed. New York: McGraw-Hill; 2010:775.)

What medications are contraindicated in WPW?


ABCD:
Adenosine
Beta-blocker
Calcium channel blocker
Digoxin

What is the primary medical treatment for WPW?


Procainamide

What would be the first-line treatment in a WPW patient with hypotension, tachycardia, and evidence of
hypoperfusion?
Synchronized cardioversion because this patient is unstable

How does digoxin toxicity present?


Supraventricular tachycardia (SVT) with atrioventricular (AV) block and yellow vision

CONGESTIVE HEART FAILURE

What is the definition of CHF?


Inability of the heart to pump enough blood to meet systemic demands. Left-sided heart failure (LHF) leads
to pulmonary vascular congestion while right-sided heart failure (RHF) causes systemic venous congestion.

What are the underlying causes of CHF?


Myocardial ischemia, anemia, pulmonary embolism, endocarditis, cardiomyopathy, hypertension,
pericarditis, cardiac dysrhythmias, thyrotoxicosis, diabetes, rheumatic heart disease, CAD
What is the most common cause of RHF?
Left heart failure

What are the symptoms of RHF?


Hepatomegaly, jugular venous distension (JVD), ascites

What are the symptoms of LHF?


Orthopnea, S3 gallop, paroxysmal nocturnal dyspnea, cough, diaphoresis, rales

What is classically seen on a chest x-ray (CXR) in a patient with CHF?


Pulmonary vascular congestion, enlarged heart, curly B lines

What are the treatments for CHF?


ACE inhibitor, diuretics, digoxin, calcium channel blocker, sodium-restricted diet, beta-blockers (but not in
acute CHF)

What is second-line treatment for CHF?


Isosorbide and hydralazine if the patient can’t tolerate an ACE inhibitor ACE inhibitors, nitroprusside

What medications decrease afterload?


Diuretics, morphine, nitroglycerine

What medications decrease preload?

Which medications have been shown to decrease mortality in CHF?


ACE inhibitor, beta-blocker, spironolactone (in class 3 and 4 heart failure)

What is initially used to treat acute pulmonary edema?


Remember the mnemonic MOrFiN
Morphine
Oxygen
r
Furosemide
i
Nitrates

Name the drug(s) used in CHF that:

VALVULAR HEART DISEASES

What does the S1 sound represent in a heart beat?


Closure of mitral and tricuspid valves

What does the S2 sound represent in a heart beat?


Closure of the aortic and pulmonic valves

What is the most common valvular heart disease found in young women?
Mitral valve prolapse

What is the underlying etiology of mitral valve prolapse?


Idiopathic; genetic transfer via autosomal dominant gene; ischemic heart disease; Marfan; myxomatous
degeneration of the mitral valve

What is the pathognomonic murmur heard in a mitral valve prolapsed?


Late systolic murmur and a mid-systolic click

Where is the murmur most audible?


Apex

What is the treatment for mitral valve prolapse?


No treatment is necessary.

What are the underlying etiologies of mitral stenosis?


Most commonly due to rheumatic heart disease

In what sex does mitral stenosis predominate?


Females

What are the signs and symptoms of mitral stenosis?


Dyspnea, orthopnea, cough, rales, hoarse voice, atrial fibrillation, hemoptysis

What is the underlying cause leading to the symptoms found in mitral stenosis?
Flow is decreased behind the mitral valve leading to left atrial enlargement and eventually heart failure.

Name the valvular heart disease associated with each of the following:

What is a mnemonic to remember diastolic murmurs?


AR/MS piTS (Aortic Regurgitation/Mitral Stenosis, Tricuspid Regurgitation)

What is a mnemonic to remember holosystolic murmurs?


MoR/TaR (Mitral Regurgitation/Tricuspid Regurgitation)

CARDIOMYOPATHY

What are the three categories of cardiomyopathy?


Remember the mnemonic HaRD
Hypertrophic
a
Restrictive
Dilated

What is the mnemonic for some etiologies of a dilated cardiomyopathy?


ABCD I
Alcohol abuse
Beriberi
Cocaine, Chagas disease, Coxsackie B
Doxorubicin
Idiopathic, Ischemic, Infectious

Name the type of cardiomyopathy associated with each of the following descriptions:

ENDOCARDITIS

What is endocarditis?
Heart valve inflammation usually due to an infective cause

Name the most common causes of the following types of endocarditis:

What type of endocarditis is seen in systemic lupus erythematosus (SLE)?


Libman-Sacks endocarditis (LSE) caused by autoantibodies damaging heart valves

How can this be distinguished on echocardiogram?


Sterile fibrofibrinous vegetations that favor the left-sided heart valves. They tend to form on the ventricular
surface of the mitral valve.

What are the signs and symptoms of endocarditis?


Fever, chills, Janeway lesion, Roth spots, Osler nodes, splinter hemorrhages, new murmur, conjunctival
hemorrhages

What are Janeway lesions?


Dark hemorrhagic peripheral macules or painless nodules usually on palms and soles (Fig 2-9)

Figure 2-9 Janeway lesions. (Reproduced, with permission, from Knoop KJ, Stack LB, Storrow AB, et al.
Atlas of Emergency Medicine. 3rd ed. New York: McGraw-Hill; 2010:374. Photo contributor: Department
of Dermatology, Wilford Hall USAF Medical Center and Brooke Army Medical Center, San Antonio, Texas.)

What are Roth spots?


Retinal hemorrhages

What are Osler nodes?


Ouch! Painful nodules on fingers and toes (Fig 2-10)

Figure 2-10 Osler nodes. (Reproduced, with permission, from Knoop KJ, Stack LB, Storrow AB, et al. Atlas
of Emergency Medicine. 3rd ed. New York: McGraw-Hill; 2010:375. Photo contributor: Armed Forces
Institute of Pathology, Bethesda, Maryland.)

What are splinter hemorrhages?


Petechiae on fingernails (Fig 2-11)

Figure 2-11 Splinter hemorrage. (Reproduced, with permission, from Knoop KJ, Stack LB, Storrow AB, et
al. Atlas of Emergency Medicine. 3rd ed. New York: McGraw-Hill; 2010:375. Photo contributor: Armed
Forces Institute of Pathology, Bethesda, Maryland.)

What is the most likely cause of right-sided endocarditis?


IVDA

What tests would you order to help diagnose endocarditis?


Three sets of blood cultures and an echocardiogram to look for vegetations

What valve is most commonly affected in endocarditis?


Mitral valve

What valve is most commonly affected in an IV drug user with infective endocarditis?
Tricuspid valve

What criteria are used to make the diagnosis of endocarditis?


Duke criteria

What are the major criteria?


1. Two positive blood cultures demonstrating the same organism
2. Positive echocardiogram

What are the minor criteria?


1. Predisposing condition such as a valvular heart abnormality, hypertrophic cardiomyopathy, congenital heart
disease
2. Documented temperature >38°C
3. Signs of embolic disease such as Janeway lesions, pulmonary emboli, cerebral emboli, hepatic or splenic
emboli
4. Immunologic signs such as Roth spot
5. One positive blood culture

Before an organism is isolated and antibiotics can be tailored, what antibiotics should be initiated in a
patient suspected to have endocarditis?
Aminoglycoside and a beta-lactam

How long should a patient with endocarditis be treated with antibiotics?


4-6 weeks

Who should be treated with prophylactic antibiotics for endocarditis before dental procedures or
gastrointestinal or genitourinary procedures?
Patients with any of the following:
History of endocarditis, prosthetic heart valves, unrepaired cyanotic heart disease, congenital cyanotic
disease repaired within the last 6 months, or cardiac transplantation with subsequent valvulopathy

What antibiotic is standard for prophylaxis prior to dental procedures?


Amoxicillin

RHEUMATIC FEVER

What infection causes rheumatic fever?


Group A streptococcal pharyngitis

Why does this infection cause rheumatic heart disease?


The antistreptococcal antibodies react with cardiac antigen.

What valve is most commonly affected in rheumatic heart disease?


Mitral valve

What serologic test could be used to confirm a prior streptococcal infection?


A positive antistreptolysino (ASO) antibody titer

What is the mnemonic for the five major criteria for rheumatic heart disease?
Jones criteria:
Joints (migratory polyarthritis)
Carditis (endocarditis, pericarditis, myocarditis)
Nodules (subcutaneous)
Erythema marginatum (serpiginous rash)
Sydenham chorea

What are the minor criteria for rheumatic heart disease?


PR-interval prolongation
Fever
Elevated ESR
Arthralgias

How should streptococcal pharyngitis be treated to prevent rheumatic heart disease?


Penicillin

PERICARDITIS

What is pericarditis?
Inflammation of the pericardium

What are some causes of pericarditis?


Infectious: viral, bacterial, fungal
Autoimmune: rheumatoid arthritis, SLE, scleroderma
Drugs HIP: hydralazine, isoniazid, procainamide (these are the same drugs that can lead to SLE-like
reaction); radiation therapy
Trauma
Post-MI
Metastatic cancer
Uremia

What is pericarditis that occurs 2-4 weeks post-MI called?


Dressler syndrome

What are the classic symptoms of pericarditis?


Pleuritic chest pain that is relieved with sitting up and leaning forward

What are the pathognomonic physical exam findings of pericarditis?


Pericardial friction rub on auscultation of the heart during expiration; pulsus paradoxus

What are the classic EKG findings associated with pericarditis?


Diffuse ST elevations and PR depressions (usually in all or almost all leads) (Fig 2-12)

Figure 2-12 Classic EKG finding assosiated with pericardits. (Reproduced, with permission, from Stead LG,
Stead SM, Kaufman MS, et al: First Aid for the Medicine Clerkship. 2nd ed. New York: McGraw-Hill;
2006:33.)

How can the diagnosis of pericarditis be confirmed?


Pericarditis is a clinical diagnosis, but an echocardiogram may show a pericardial effusion.

How is pericarditis treated?


The underlying cause should be addressed. NSAIDs to decrease inflammation; antibiotics for bacterial
causes; steroids for autoimmune etiology; pericardiocentesis would be necessary for a large pericardial
effusion

MYOCARDITIS

What is myocarditis?
Inflammation of the heart muscle

What is the most common viral cause of myocarditis?


Enterovirus infection (eg, Coxsackie B)

What are the causes of myocarditis?


Viral: Coxsackie A and B, HIV, EBV (Epstein-Barr), HBV (hepatitis B), cytomegalovirus (CMV)
Bacterial: rheumatic fever, Lyme disease, meningococcus, mycoplasma
Parasitic: Chagas disease, toxoplasmosis, trichinella
Autoimmune: SLE, Kawasaki disease
Drugs

What are the signs and symptoms of myocarditis?


Precordial chest pain with signs of CHF

What does the EKG look like in a patient with myocarditis?


Nonspecific ST changes, dysrhythmias

How can a definitive diagnosis of myocarditis be made?


Myocardial biopsy

What is the treatment for myocarditis?


Treat CHF symptoms, dysrhythmias, and the underlying etiology. Steroids are contraindicated. In some
cases, intravenous immunoglobulin (IVIG) is beneficial.

CARDIAC TAMPONADE

What is cardiac tamponade?


Pericardial fluid accumulation that causes impaired cardiac filling and thus leads to decreased cardiac output

What is Beck’s triad?


Symptoms seen in cardiac tamponade:
1. Hypotension
2. Distant heart sounds
3. JVD

What are some other classic symptoms of cardiac tamponade?


Dyspnea, tachycardia, pulsus paradoxus

What is pulsus paradoxus?


>10 mm Hg fall in blood pressure during inspiration

What is seen on EKG in a patient with pulsus paradoxus?


Electrical alternans—a beat-to-beat change in the height of the QRS complex

What study can help confirm the diagnosis of cardiac tamponade?


Echocardiogram will show a pericardial effusion.

What is the treatment for pericardial tamponade?


Pericardiocentesis vs pericardial window. IV fluids should also be given for volume expansion.

ABDOMINAL AORTIC ANEURYSM


Who should be screened for an abdominal aortic aneurysm (AAA)?


Men older than 65 with a smoking history

What is the screening tool used?


Ultrasound

How should AAA be managed?


AAA <5 cm: blood pressure control, smoking cessation, and annual imaging AAA >5 cm: elective surgical
repair

What are the typical symptoms of a ruptured aneurysm?


Severe chest pain with radiation to the back between the scapula. Often described as a tearing pain.

What is the treatment of a ruptured aneurysm?


Emergent surgery

What are the risk factors to the development of an AAA?


1. Smoking (#1 risk factor)
2. Hyperlipidemia
3. Chronic hypertension

What is the classic description of a patient with an AAA on chest x-ray?


Widened mediastinum

CLINICAL VIGNETTES

A 54-year-old male with type II diabetes comes in for a routine physical. The vitals show a BP of 138/69.
The patient currently takes aspirin, Zocor, and metformin. What type of drug would you add to this
regimen?
ACE inhibitor

A 65-year-old male with a history of long-standing hypertension, presents with sensation of


lightheadedness, shortness of breath, and palpitations. An EKG shows an irregularly, irregular rhythm.
The heart rate is 110. The patient is started on a beta-blocker and the rate is controlled. You decide to start
Coumadin. What is the goal INR?
INR between 2 and 3

A patient with a history of coronary artery disease presents to the emergency room complaining of chest
pain. His medication list includes the following drugs: Prozac, lisinopril, atenolol, Zyrtec, nitroglycerin,
aspirin, daily multivitamin. What class of medication would you suggest him to add to this regimen?
Statin

An obese 48-year-old male presents complaining of palpitations and light-headedness. An EKG


demonstrates an irregularly irregular rhythm at 120 beats/min. What is the most likely underlying cause of
this patient’s problem?
The patient has atrial fibrillation and most likely caused by hypertension.

A 36-year-old male with no past medical history complains of several days of chest pain. The chest pain is
exacerbated with inspiration and relieved with sitting up. He recently had a viral illness with fever. His
physical examination is unremarkable. His EKG shows diffuse ST-segment elevations. What is the most
likely diagnosis for this patient’s symptoms and what is the first-line treatment?
Pericarditis and NSAIDs
CHAPTER 3
Pulmonology

LUNG VOLUMES

What is the definition of each of the following? (Fig 3-1)


See Figure 3-1

Figure 3-1 Lung volumes as represented by spirogram tracing. (This figure was published in Lumb AB.
Nunn’s Applied Respiratory Physiology, 5th ed. Butterworth-Heinemann. Copyright Elsevier 2000.)

HYPOXIA

What is the mnemonic for the mechanisms of hypoxia?


CIRCULAR:
Circulatory
Increased oxygen requirement
Respiratory
Carbon monoxide poisoning
Underutilization
Low fraction of inspired oxygen (FIo2)
Anemia
Right-to-left shunt

What are the important respiratory causes of hypoxia?


Hypoventilation due to a decreased respiratory rate, decreased vital capacity, or ventilation/perfusion ratio
(V/Q) mismatch

How can respiratory hypoxia be improved?


Supplemental oxygen and by treating the underlying cause

What is a common underlying cause of decreased respiratory rate?


Drugs: opiates

What are common underlying causes of increased respiratory rate?


Infection
Trauma
What are some reasons for decreased vital capacity?
Underlying lung disease, deformities of the chest wall such as in severe scoliosis, muscle weakness

When is low FIO2 mostly a problem?


High altitudes or closed spaces with no fresh air or fire

How does diffusion impairment cause hypoxia?


With circulatory impairment such as in heart failure or anemia, there is poor perfusion and, therefore,
decreased blood transit time in the lungs causing decreased diffusion. Other reasons for diffusion impairment
would be due to underlying lung pathology causing an increased diffusion pathway.

Give an example of hypoxia caused by underutilization.


When there is impairment of cytochrome due to toxins/poisons, such as with cyanide

What are the examples of increased requirement for oxygen?


Exercise, hyperthyroidism, infection

What are some examples of causes of V/Q mismatch?


Pulmonary embolism, underlying lung disease such as in lung cancer or chronic obstructive pulmonary
disease (COPD), bronchospasm, pneumonia, pulmonary edema

Why is carbon monoxide poisoning a cause of tissue hypoxemia?


Carbon monoxide binds to hemoglobin and makes it unavailable for oxygen transport.

What is the clinical sign of carbon monoxide poisoning?


Cherry red lips and nails

What is an A-a gradient?


The difference in concentration between alveolar and arterial oxygen. It is a measure of gas exchange
efficiency in the lung. The less the gradient the greater the oxygenation.

How do you expect the PCO2, and A-a gradient to be affected in each of the following causes of hypoxia?
See Table 3-1.
1. Hypoventilation
2. Right-to-left shunt
3. Low FIO2
4. V/Q mismatch

Table 3-1 Hypoxia Etiology and Effects on PCO2 and A-a Gradient

What are the signs and symptoms of hypoxia?


Dyspnea, tachypnea, tachycardia (increased perfusion), clubbing of nails, and cyanosis of extremities
What is the treatment for most types of hypoxia?
Increased FIO2 via oxygen administration while identifying and treating the underlying cause

What type of hypoxemia does not improve with increased FIO2?


A right-to-left shunt because there is no ventilation of the abnormal alveoli and, therefore, blood does not
come in contact with oxygen

How is hypoxemia secondary to high altitude treated?


Oxygen administration can help but the body adjusts and self-corrects within several weeks.

OBSTRUCTIVE PULMONARY DISEASES

What defines chronic obstructive pulmonary disease (COPD)?


As the name implies, it is defined by chronic obstruction to expiratory airflow such that the forced expiratory
volume in 1 second/forced vital capacity (FEV1/FVC) is decreased.

What are the two main forms of COPD?


Emphysema and chronic bronchitis

What is the male-to-female ratio of emphysema?


Male:female = 10:1.

What defines emphysema?


Chronic obstructive expiratory airflow with dilation of air spaces caused by destruction of alveolar walls

What is the most common cause of emphysema?


Smoking

What type of emphysema does smoking cause?


Centrilobular, meaning that it affects the bronchioles (Hint: The “S” sound is in both smoking and
centrilobular)

What causes panacinar emphysema?


Alpha-1-antitrypsin deficiency

What is the function of alpha-1-antitrypsin in the lung?


It protects the elastin in the lungs from proteolytic enzymes.

What are the pathognomonic symptoms associated with emphysema?


Pursed lip breathing (with prolonged expiratory phase), barrel chest, hyperventilation; classically
described as a “pink puffer,” weight loss

What is seen on a chest x-ray (CXR) in a patient with emphysema?


Hyperinflation and hyperlucency of the lungs with flattening of the diaphragms; parenchymal bullae and
subpleural blebs may be present; alveolar wall destruction

What do you expect to see in arterial blood gases (ABGs) in a person with early-stage emphysema?
Low PCO2 and normal/low PO2

What is the long-term treatment for emphysema?


Smoking cessation! Home oxygen, bronchodilators, steroids; pneumococcal and flu vaccines should be
offered

What defines chronic bronchitis?


Productive cough on most days during 3 or more consecutive months for 2 or more consecutive years

What is the difference in symptomatology in chronic bronchitis vs emphysema?


Chronic bronchitis includes a persistent productive cough as well as more hypoxia than seen in emphysema,
and patients are usually overweight.

What is the pathognomonic description of a person with chronic bronchitis?


“Blue bloater” because of CO2 retention and hypoxia

What do you expect to see in an ABG in a person with chronic bronchitis?


High Pco2 and low PO2, compensated respiratory acidosis

What are the potential complications associated with chronic bronchitis?


Right heart failure (cor pulmonale), polycythemia, pneumonia, hepatomegaly

What is the treatment for chronic bronchitis?


Treatments are the same as that for emphysema and include smoking cessation, oxygen therapy,
bronchodilators, and steroids, and, also, some treatment with antibiotics in exacerbations.

What are the only treatments proven to extend life in COPD?


Oxygen therapy and smoking cessation

How is bronchiectasis defined?


Pathologic dilatation of bronchioles caused by chronic inflammation and wall structure destruction

What are some common etiologies of bronchiectasis?


Cystic fibrosis, tuberculosis (TB), lung abscess, toxin inhalation

What is the underlying pathologic problem that results because of chronic dilatation of bronchioles?
The dilated bronchioles impede mucociliary clearance, favoring mucus pooling and colonization with
bacteria and, therefore, further lung damage.

What are the most common pathogens that colonize the lung in an individual with bronchiectasis?
SHiPS:
Staphylococcus aureus
Haemophilus influenzae
i
Pseudomonas
Streptococcus pneumonia

How do you treat the organisms that most commonly infect the lung in bronchiectasis?
Third-generation cephalosporin

What are the signs and symptoms of bronchiectasis?


Halitosis, hemoptysis, chronic cough with sputum production

How can bronchiectasis be diagnosed?


High-resolution computed tomographic (CT) scan of the lungs will demonstrate bronchial dilatation as well
as destruction
What is the pathognomonic sign seen on CT in a person with bronchiectasis?
Tram track lung markings

What is the treatment for bronchiectasis?


Antibiotics for infections, bronchodilators, oxygen, mucolytics, chest PT and, sometimes, lung transplant

How is asthma defined?


Reversible obstruction of airways secondary to airway inflammation, hypersecretion and, most importantly,
bronchoconstriction that leads to a decreased peak flow and FEV1

What is intrinsic asthma associated with?


Exercise-induced or upper respiratory infection (URI)–induced asthma

What is extrinsic asthma associated with?


Asthma caused by eosinophilia or increased immunoglobulin E (IgE) levels in response to Environmental
antigens

When does asthma usually start and what is its usual course?
Asthma generally begins during childhood and usually resolves on its own by the early teenage years.

What is often the first symptom of asthma that a patient will often describe?
Nighttime cough (for some people this is the only symptom)

What are some of the major signs and symptoms of an acute asthma exacerbation?
Expiratory wheeze, shortness of breath, chest tightness, subcostal retractions, accessory muscle use,
prolonged expiratory phase

What would spirometry show in an asthmatic?


Decreased FEV1

How can it be confirmed that the wheezing is caused by asthma and not some other cause?
The wheezing resolves with bronchodilator therapy and the FEV1 will increase by 10% or more.

What would an ABG show in an asthma attack?


Hypoxia and respiratory alkalosis

What is a sign of impending respiratory failure in a case of asthma?


ABG that shows normalizing Pco2

What classic diagnosis should you think of if the complete blood count (CBC) of an asthmatic demonstrates
eosinophilia?
Churg-Strauss syndrome

What are the different categories of asthma, what are their symptoms (Sx), and what is the treatment for
each?
See Table 3-2.

What is the first-line treatment for an acute asthma exacerbation?


Oxygen, bronchodilators (includes beta-agonist and ipratropium [Atrovent]), and steroids

What is the second-line treatment for an acute asthma attack?


Subcutaneous epinephrine and MgSO4
How can mild asthma refractory to aggressive beta-agonist therapy be treated?
Add an inhaled steroid.

When is systemic corticosteroid therapy indicated in asthma?


Daily or continuous asthma that is refractory to beta-agonist and inhaled steroids

What are some alternative therapies in asthma?


Leukotriene inhibitors, methylxanthines, immunomodulators, leukotriene receptor modifiers, and cromolyn
sulfate. Also allergic desensitization in extrinsic asthma.

Table 3-2 Asthma Classification and Medical Management

RESTRICTIVE LUNG DISEASE

What is the definition of a restrictive lung disease?


Unlike obstructive lung disease, the FEV1/FVC is normal to high; it is the total lung capacity (TLC) that
decreases

What are some examples of restrictive lung diseases?


Interstitial lung diseases, spaceoccupying lesions such as tumors; pleural effusions; neuromuscular diseases
such as severe scoliosis, spinal cord trauma, and multiple sclerosis

What are some examples of interstitial lung diseases?


Anything that causes chronic lung injury such as asbestosis, acute respiratory distress syndrome (ARDS),
coal mine dust, silicosis, berylliosis, chronic lung injury because of chronic infections, radiation
What is the pathognomonic description of an interstitial lung disease?
“Honeycomb lung”

What is the most common cause of atelectasis?


A postoperative patient who is nonambulatory for a long period of time

What types of chemotherapy can cause a restrictive lung disease?


Busulfan and bleomycin

PLEURAL EFFUSION

What is a pleural effusion?


Increased fluid in the pleural space

What are the two main types of pleural effusions?


1. Exudate
2. Transudate

What are some common causes of exudative pleural effusions?


Infection such as pneumonia, malignancy, collagen vascular disease

What are some common causes of transudative pleural effusions?


Congestive heart failure (CHF), cirrhosis, nephritic syndrome

What is the underlying cause of fluid buildup in an exudate?


Increased capillary permeability

What is the underlying cause of fluid buildup in a transudate?


1. Decreased oncotic pressure
2. Increased hydrostatic pressure

How can a pleural effusion be evaluated?


Thoracentesis with analysis of cell counts, cultures, chemistries, and cytology

How can a pleural effusion be treated?


Treating the underlying cause and thoracentesis can be both diagnostic and therapeutic.

What lab tests should be sent in order to evaluate the pleural fluid?
Fluid and serum protein, glucose, lactate dehydrogenase (LDH); fluid culture and Gram stain; fluid cytology
and cell count with differential and, additionally, you can send fluid amylase, AFB, ANA, RF, pH

What defines an exudative effusion?


If any of the following is true, the fluid effusion is considered exudative. Pleural protein/serum protein >0.5
Pleural LDH/serum LDH >0.6 Pleural LDH <200

What does it signify if the pleural fluid has >10,000 WBCs with polymorphonuclear neutrophils (PMNs)?
Most likely a parapneumonic effusion

What is gross blood in the pleural fluid associated with?


Tumor or trauma
What can low glucose (glucose <60) in the pleural fluid be associated with?
Tumor, empyema, rheumatologic etiology, parapneumonic exudate

What are high amylase levels in pleural fluid associated with?


Pancreatitis but can also be malignancy, or esophageal rupture

What percentage of pleural effusions caused by malignancy will have a fluid cytology that has malignant
cells?
Only 40%

How can exudative and transudative pleural effusions be differentiated?


See Table 3-3.

Table 3-3 Exudative vs. Transudative Pleural Effusion

COUGH

What is the definition of an acute cough?


Cough that has lasted <3 weeks

What is the most common cause of an acute cough?


Postnasal drip (also very common are asthma and GERD)

What are the most common causes of postnasal drip?


Sinusitis, allergic rhinitis, seasonal or environmental allergies, flu or cold

What is the preferred method of treatment of postnasal drip caused by allergies?


Antihistamine treatment and/or nasal corticosteroid

What is the preferred method of treatment of postnasal drip caused by the cold?
Antihistamine as well as a decongestant

What is sinusitis?
A bacterial or viral infection of the sinuses

Which sinus is most commonly affected?


The maxillary sinus

What are the signs and symptoms of sinusitis?


Fever, tenderness to percussion over the sinuses, increased pain with bending forward, purulent nasal
discharge, halitosis, headache

Define acute, subacute, and chronic sinusitis?


Acute sinusitis lasts <3 weeks, subacute lasts between 21 and 60 days, and chronic sinusitis lasts >60 days.

What most commonly causes acute sinusitis?


Viruses

What are the most common pathogens involved in acute bacterial sinusitis?
Streptococcus pneumoniae, H influenzae, and Moraxella catarrhalis

What is the treatment for acute sinusitis?


Viral rhinosinusitis does not require antimicrobial treatment. Nasal corticosteroids and decongestants are
helpful. Studies have shown that steroids lead to faster symptom resolution. Bacterial causes should be
treated with amoxicillin, augmentin, or bactrim for 1-2 weeks.

What are the potential complications secondary to chronic sinusitis?


Meningitis, osteomyelitis, orbital cellulitis, cavernous sinus thrombosis, abscess

What is the classic organism causing sinusitis in a diabetic?


Aspergillus causing mucormycosis

What is the definition of a chronic cough?


A cough lasting >3 weeks

What are the three most common causes of chronic cough?


1. Postnasal drip
2. Asthma
3. Gastroesophageal reflux disease (GERD)

What medication class can cause a chronic cough?


Angiotensin–converting enzyme (ACE) inhibitors

ACUTE RESPIRATORY DISTRESS SYNDROME


What are the components of acute respiratory distress syndrome (ARDS)?
Refractory hypoxemia, decreased lung compliance, noncardiogenic pulmonary edema

What is the etiology of ARDS?


Endothelial injury secondary to aspiration, multiple transfusions, shock, sepsis, trauma

What are the criteria needed to diagnose ARDS?


1. Acute onset of respiratory distress
2. PAO2: FIO2 ratio <200 mm Hg
3. Bilateral pulmonary infiltrates on CXR
4. Normal capillary wedge pressure

What is the treatment for ARDS?


Treat the underlying disease and give adequate oxygen via mechanical ventilation

What is the overall mortality in ARDS?


About 50%

PULMONARY EMBOLISM

What is the most common etiology of a pulmonary embolism (PE)?


Dislodged deep vein thrombosis (DVT)

What are the risk factors for a DVT?


Virchow triad: stasis (usually due to immobilization), hypercoagulable state, endothelial injury

What are the risk factors for a PE?


Same risk factors as getting a DVT, as well as having a DVT, stroke, myocardial infarction (MI); recent
surgery leading to immobilization

What are some examples of hypercoagulable states?


Malignancy; protein C or protein S deficiency; antithrombin III deficiency; factor V Leiden deficiency;
hyperestrogen states such as pregnancy, oral contraceptive use, smoking

What is an important question to ask in the patient’s history?


Ask if they have had any recent travel or other immobilization. Long trips cause people to be immobile for
long periods of time and therefore have a greater risk for developing DVTs and therefore PEs.

What is the most common sign in a patient with a PE?


Sinus tachycardia

What are some of the common symptoms of PE?


Dyspnea, tachypnea, pleuritic chest pain, fever, unilaterally swollen and painful posterior lower extremity,
cough, hemoptysis

What are the classic CXR findings in a PE?


Hampton hump—wedge-shaped infarct
Westmark sign—hyperlucency in the lung region supplied by the affected artery

What is the most common EKG finding in a PE patient?


Sinus tachycardia
What is the classic EKG finding in a PE patient?
S1Q3T3—S wave in lead I, Q wave in lead III, and inverted T wave in lead III (Fig 3-2)

Figure 3-2 Pulmonary embolism S1Q3T3 pattern. (Reproduced with permission from Kaufman MS et al.
First Aid for the Medicine Clerkship. New York: McGraw Hill; 2002:75; Figure 3-1)

What is the gold standard for diagnosis of PE?


Pulmonary angiography

What are some of the diagnostic techniques used to diagnose a PE?


Spiral CT, V/Q scan, pulmonary angiography or MR angiography

What blood test can be done to rule out PE if it is not positive?


D-dimer

What diagnostic test can be done to rule out a DVT?


Duplex ultrasound

What is the algorithm used to diagnose a PE, when one is suspected?


See Fig 3-3.

Figure 3-3

What are the treatments for a PE?


Heparin or Lovenox (low-molecular-weight heparin) acutely, long-term treatment with Coumadin or inferior
vena cava (IVC) filter, or tissue-type plasminogen activator (tPA) thrombolysis in massive PE

What needs to be done if Coumadin is being started?


Heparin must be continued until Coumadin becomes therapeutic since Coumadin can cause a
hypercoagulable state.

What is the therapeutic international normalized ratio (INR)?


INR of 2-3

PNEUMOTHORAX

A person with what body habitus is most likely to have a primary spontaneous pneumothorax?
Tall and thin male

What is the most likely etiology of a primary spontaneous pneumothorax?


Rupture of subpleural blebs

What are some risk factors for having a secondary spontaneous pneumothorax?
COPD, lung cancer, pneumonia, TB, HIV, cystic fibrosis, trauma

What are the signs and symptoms of a pneumothorax?


Sudden unilateral chest pain, dyspnea, and tachypnea

What is found on physical examination in a person with a pneumothorax?


Absent breath sounds on the side of the pneumothorax and hyperresonance to percussion

What is seen on CXR in a pneumothorax?


Absent lung markings on the side of the pneumothorax

What is the treatment of a spontaneous pneumothorax?


Oxygen is the mainstay of therapy, but if the pneumothorax is symptomatic, a tube thoracostomy may be
indicated. Pleurodesis can be used to make the visceral and parietal pleura adhere to each other.

What is a tension pneumothorax?


A chest wall defect causes air to be trapped in the pleural space during expiration like a one-way valve (Fig
3-4).

Figure 3-4 Tension pneumothorax. (Reproduced, with permission, from William Herring, MD, FACR;
Radiology Residency Program Director at Albert Einstein Medical Center in Philadelphia, PA;
http://www.learningradiology.com)

How is a tension pneumothorax treated?


This is a medical emergency. Treatment includes immediate needle decompression and chest tube placement
thereafter.

HEMOPTYSIS

What are the two most common causes of hemoptysis in the United States?
1. Bronchitis
2. Bronchogenic carcinoma

What are the two most common causes of hemoptysis worldwide?


1. Tuberculosis
2. Bronchiectasis

What are some other causes of hemoptysis?


Pneumonia, Wegener granulomatosis, Goodpasture syndrome, mycetoma, pneumonitis, arteriovenous (AV)
malformation, pulmonary embolism, airway trauma, foreign body, metastatic lung tumor, bronchial
carcinoid

What are the treatments for hemoptysis?


Have the patient lie down with the bleeding side down to protect the airway, oxygen as needed and in severe
cases, bronchial artery embolization or intubation of the good lung

LUNG CANCER

What is the most common cause of cancer death in the United States?
Lung cancer

What is the most likely causative factor of lung cancer?


Smoking

What are some other causes of lung cancer?


Second-hand smoke, exposure to asbestos, nickel, arsenic, radon gas

What are the two main categories of lung cancer?


Small cell and nonsmall cell

What are the different types of lung tumors that are nonsmall cell lung cancers?
Large cell, adenocarcinoma, squamous cell, bronchoalveolar cell

Name the type of lung cancer associated with the following:


What are some signs and symptoms of lung cancer?


Chronic cough, hemoptysis, weight loss, night sweats, pneumonia (postobstructive), hoarseness,
paraneoplastic syndrome

How is lung cancer diagnosed?


Usually a nodule or mass is seen on CXR or CT of the chest and is diagnosed with a biopsy usually done via
bronchoscopy or CT-guided fine-needle aspiration

What are some of the signs and symptoms of a carcinoid tumor?


Symptoms of carcinoid syndrome due to serotonin secretion, which include flushing, asthmatic wheeze,
diarrhea

What is the diagnostic test for a carcinoid tumor?


Test for elevated urine 5-hydroxyindoleacetic acid (5-HIAA), a serotonin metabolite
How is carcinoid syndrome treated?
Serotonin antagonist

What is a Pancoast tumor?


Superior sulcus tumor

What paraneoplastic syndromes are associated with a Pancoast tumor?


Horner syndrome, superior vena cava syndrome

What is Pancoast syndrome?


Shoulder and arm pain secondary to the tumor compressing the thoracic inlet with compression of the
brachial plexus and cervical sympathetic nerves

Name the paraneoplastic syndrome associated with signs and symptoms described below:

PNEUMONIA

What are some common signs and symptoms of pneumonia (PNA)?


Cough with purulent sputum, fever, chills, pleuritic chest pain

What are some common physical examination findings in a patient with pneumonia?
Decreased breathing sounds, crackles, egophony, dullness to percussion, tactile fremitus on the side of the
pneumonia, fever

What studies should be ordered if a PNA is suspected?


CXR, CBC, sputum culture and Gram stain, blood culture (in hospitalized patient)

What do you see on a CXR in a patient with pneumonia?


Lobar consolidation (Fig 3-5)

Figure 3-5 Pneumonia. (Reproduced, with permission, from William Herring, MD, FACR; Radiology
Residency Program Director at Albert Einstein Medical Center in Philadelphia, PA;
http://www.learningradiology.com)

What would the CBC show?


Leukocytosis with a left shift

Name the most common organism causing pneumonia in each of the following cases:

What is the treatment for each of the following cases of pneumonia?

What are the most common pathogens and treatments in each of the following cases?

TUBERCULOSIS

How does TB spread?


Air droplet transmission
Who is at high risk for becoming infected with TB?
Immunocompromised, foreign-born, homeless, prisoner, low-income communities, intravenous (IV) drug
users

What are the common signs and symptoms of TB?


Productive cough, night sweats, weight loss, hemoptysis, fever, chills, chest pain

How is latent TB detected?


Positive purified protein derivative (PPD) (tuberculin) skin test

What is considered a positive PPD?


>15 mm in any person
>10 mm in immunocompromised, IV drug user, foreign-born, prisoner, nursing home resident, people who
work in the medical field (that means you), underserved community
>5 mm: HIV, abnormal CXR, close contact with someone who had TB

How is a positive PPD treated?


Isoniazid (INH) for 9 months with vitamin B6

What laboratory tests should be done when starting a patient on INH?


Liver function tests (LFTs) because of possibility of hepatotoxicity

What part of the lung does primary TB usually affect?


Lower lobes

What is the radiographic finding of healed primary TB called?


“Ghon complex,” which is a calcified nodule with calcified lymph nodes (Fig 3-6)

Figure 3-6 Tuberculosis. (Reproduced, with permission, from William Herring, MD, FACR; Radiology
Residency Program Director at Albert Einstein Medical Center in Philadelphia, PA;
http://www.learningradiology.com)
What is secondary TB?
Reactivation TB

Where is secondary TB usually found?


Lung apices

What is extrapulmonary TB?


TB that had disseminated to other organs

What is the most common extrapulmonary location for TB to spread?


Kidneys

What are other locations where extrapulmonary TB can be found?


Liver, central nervous system (CNS), vertebral bodies, psoas muscle, cervical lymph nodes, pericardium

What is TB of the vertebral bodies called?


Pott disease

What is cervical lymphadenopathy secondary to TB infection called?


Scrofula

How is active TB diagnosed?


Clinical symptoms, CXR, and sputum acid-fast stain and culture

What is seen on CXR in active TB?


Upper lobe infiltrates with scarring, cavitary lesions

What is the standard treatment for active TB?


Four-drug therapy initially for 2 months followed by 2-drug therapy (of INH and rifampin) for 4 months
(remember the mnemonic RIPE):
Rifampin
INH
Pyrozinamide
Ethambutol

CLINICAL VIGNETTES

A 63-year-old female with a history of ovarian cancer presents with severe shortness of breath and chest
pain with inspiration. She has a low-grade fever, heart rate of 125, blood pressure of 138/60, respiratory
rate of 25, and oxygen saturation of 88%. What test will confirm the diagnosis?
CT pulmonary angiogram or V/Q scan to check for a pulmonary embolism

A 30-year-old male with no significant past medical history presents to your office with fever, cough, and
shortness of breath. On examination there are decreased breath sounds in the right lower lobe. His oxygen
saturation is 92%. He has an elevated white count on CBC. What organism do you suspect?
Mycoplasma

A healthy American born 28-year-old male has started a new job at the bank that requires him to get a
PPD. He denies exposure to tuberculosis and in a recent HIV test he had was negative. His PPD comes
back at 16 mm. What is your next step?
Treat with INH and vitamin B6

A 60-year-old male with a 40 pack-year history of smoking presents with a cough. He states that the cough
has been present for many months. It is a dry cough. He denies any chest pain or fever but does state that
he often feels short of breath. A CBC comes back within normal limits. On examination he is a barrel-
chested male with decreased breath sounds throughout and distant heart sounds. CXR only demonstrates
flattened diaphragms. What is the most likely reason for this patient’s cough?
Emphysema

A tall 20-year-old male complains of sudden left-sided chest pain with shortness of breath and tachypnea.
The chest x-ray shows absent lung markings of the left side. What is the initial treatment for his condition?
Tension pneumothorax requires immediate needle decompression followed by chest tube placement
CHAPTER 4
Neurology

CEREBROVASCULAR ACCIDENTS

What is a cerebrovascular accident (CVA)?


Sudden onset of neurologic deficit that is a result of cerebrovascular disease

What does TIA stand for?


Transient ischemic attack

What is a TIA?
A neurologic deficit that lasts <24 hours and resolves completely

What is a stroke?
Focal neurologic deficit that results from infarcted cerebral tissue

What does RIND stand for?


Reversible ischemic neurologic deficit

What is RIND?
Neurologic deficits that last >24 hours and <3 weeks

What are the two greatest risk factors for a stroke?


1. Hypertension
2. Smoking

What are the two etiologies of a CVA?


Ischemic or hemorrhagic stroke

What are the two most common etiologies for ischemia?


1. Thrombotic etiology which is secondary to atherosclerosis
2. Embolic etiology which is usually either cardiac in origin or from carotid arteries

What is the most common etiology of a CVA?


Ischemia

Name the term associated with each of the following:


Describe the deficits caused by occlusion of the following arteries:


What is the first study to order if you suspect a stroke in a patient?


Computed tomography (CT) of head without contrast to rule out a bleed

What other studies can be done to further assess the patient?


Magnetic resonance imaging (MRI) to evaluate for subacute infarction; carotid Doppler ultrasound to rule
out carotid artery stenosis; echocardiogram to rule out embolic sources

What is the treatment for a TIA?


Patient should be started on aspirin.

What medication should the patient be started on if they fail aspirin?


Plavix (clopidogrel)

What other antiplatelet therapies are available other than aspirin?


Plavix (clopidogrel), ticlopidine, Aggrenox

When would you consider a carotid endarterectomy?


If the patient had carotid artery stenosis >70%

What is the treatment for a patient who had a cardioembolic stroke?


Anticoagulation with heparin or Coumadin
What treatment improves outcome in a patient who present with an embolic stroke with symptoms
beginning <3 hours ago?
Tissue plasminogen activator (tPA)

What is the major contraindication to tPA use?


Intracranial bleeding

How should hypertension be treated in a patient who acutely had a stroke?


Hypertension should not acutely be controlled tightly in order to allow for good cerebral perfusion. A 25%
reduction in mean arterial pressure in the first 24 hours is acceptable.

Other than starting medications, what other long-term interventions should be taken in a patient with a
history of stroke to prevent future infarctions?
Good diabetes control (improved HgA1c); control hypertension; smoking cessation; treat hyperlipidemia

SEIZURE DISORDERS

What is a seizure?
Excessive firing of cortical neurons leading to neurologic symptoms

What is the single most useful test to evaluate seizures?


Electroencephalogram (EEG)

What tests should be done on a patient suspected to have had a seizure?


Complete neurologic examination. Check for signs of incontinence, tongue lacerations, other injuries to the
body to distinguish from syncope. Also check the following laboratory tests: complete blood count (CBC),
electrolytes, calcium, glucose, oxygen level, liver function tests, blood urea nitrogen (BUN), creatinine (CR),
rapid plasma reagin (RPR), erythrocyte sedimentation rate (ESR), urine toxicology screen. MRI and CT can
also be done to rule out a mass.

What factors can increase the risk of having a seizure?


History of having a seizure in the past, central nervous system (CNS) tumor, CNS infection, trauma, stroke,
high fever in children, drugs

What is Todd paralysis?


Postictal state in which there are focal neurologic deficits that last 24-48 hours; usually associated with focal
seizures

Name the two types of generalized seizures.


1. Tonic-clonic seizures
2. Absence seizures

Name the seizure disorder described below:



Indicate the treatment for each of the following types of seizures:

What is the most significant side effect(s) of each of the following antiseizure medications?

When can antiseizure medication be discontinued in a patient with a history of seizures?


No seizures for 2 years

MENINGITIS

What is the most common bacterial pathogen causing meningitis in adults?


Streptococcus pneumoniae causes up to 60% of meningitis cases.

What two bacterial pathogens cause most cases of meningitis in young adults?
S pneumoniae and Neisseria meningitides

In what population does Group B Streptococcus cause meningitis?


Neonates (most common cause of meningitis in neonates)

What three bacterial pathogens most commonly cause meningitis in neonates?


1. Listeria
2. Group B Streptococcus (GBS)
3. Escherichia coli

In what adult population does Listeria cause meningitis?


Immunocompromised patients

What bacterial pathogen known to cause meningitis is now vaccinated against?


Haemophilus influenza

What is the treatment for each of the bacterial pathogens in meningitis?


See Table 4-1.

Table 4-1 Meningitis Organisms and Treatments


What are the classic symptoms of meningitis?


Fever, headache with neck stiffness, photophobia, meningismus, Kernig sign, Brudzinski sign

What is meningismus?
Patient has difficulty touching their chin to their chest.

What is Kernig sign?


Patient has pain when extending the knee with the thigh at 90°.

What is Brudzinski sign?


Neck flexion causes involuntary flexion at the hip and knees.

What test is used to diagnose meningitis?


Lumbar puncture with cerebrospinal fluid (CSF) analysis including Gram stain, cultures

What would the CSF findings be in bacterial meningitis (see Table 4-2)?
Increased protein, decreased glucose, very elevated WBCs, elevated opening pressure, and elevated number
of neutrophils

What would the CSF findings be in viral meningitis?


Normal protein and glucose, elevated WBC, normal or elevated opening pressure, increased lymphocytes

What would the CSF findings be in fungal meningitis?


Elevated protein, decreased glucose, elevated WBC, elevated opening pressure, increased lymphocytes
(Table 4-2)

Table 4-2 CSF Findings in Meningitis


What is the appropriate empiric treatment for meningitis in each of the following populations?
(see Table 4-3)
Neonates
1-3 months
Young adults
Adults
Elderly, immunocompromised

Table 4-3 Meningitis Treatment by Population

BRAIN TUMORS

What is the most common type of brain tumor?


Metastatic tumor

From what primary tumors do most metastatic brain tumors originate?


Lung cancer, breast cancer, melanoma, gastrointestinal (GI) tumors

Anatomically, where do most adult brain tumors tend to present?


Supratentorially

Anatomically, where do most childhood brain tumors tend to present?


Infratentorially

What are some common symptoms of brain tumors?


Headache (especially upon waking), vomiting, seizures, focal neurologic symptoms
How is a brain tumor diagnosed?
CT with contrast/MRI with gadolinium localizes the lesion and a biopsy is used to get the histologic class of
the tumor.

What is the most common type of primary brain neoplasm?


Astrocytoma

What is the most common type of astrocytoma?


Glioblastoma multiforme

What is the prognosis of glioblastoma multiforme?


Poor prognosis. 5-year survival is <5%.

Where do ependymomas usually arise?


In the fourth ventricle

In what population are ependymomas most common?


Children

What is the prognosis?


80% 5-year survival

What is the most common cranial nerve tumor?


Schwannoma

What cranial nerve does a schwannoma affect?


Cranial nerve VIII—vestibular division

What is the most common mesodermal tumor?


Meningioma

How are most brain tumors treated?


Surgical excision and radiation. Medulloblastomas also require chemotherapy and schwannomas are treated
with surgery alone.

DEMYELINATING DISEASES

What is the most common demyelinating disorder?


Multiple sclerosis (MS)

Who is at higher risk for developing MS?


Those with a family history of MS, those who lived up until puberty in northern latitudes or temperate
climates, female sex (incidence is 2:1 female:male)

What age is the peak age of MS presentation?


Age 20-40

What is the typical course of the disease?


Multiple progressive neurologic alterations that wax and wane and cannot be explained by a single lesion

What are some of the signs and symptoms of MS?


Limb weakness, paresthesias, optic neuritis, nystagmus, scanning speech, intranuclear ophthalmoplegia,
vertigo, diplopia

What is Lhermitte sign?


Shock-like sensation down the spine when patient flexes their neck. Also known as the “barber chair
phenomenon.”

What can be seen on MRI on a patient with MS?


MRI shows multiple, asymmetric, periventricular plaques with multiple areas of demyelination.

What does the CSF show in an MS patient?


Oligoclonal bands; elevated IgG

What is the treatment for MS?


Steroids during acute episodes and interferon-β to prolong remission

What is the other name for amyotrophic lateral sclerosis (ALS)?


Lou Gehrig disease

What is the underlying pathology in ALS?


Slowly progressive loss of upper and lower motor neurons in the CNS

What are the clinical signs and symptoms of ALS?


Asymmetric, progressive muscle weakness initially with fasciculations which present clinically as difficulty
swallowing. Patients also have upper motor neuron and lower motor neuron signs on physical examination.
They do not have bowel or bladder involvement.

Give examples of both upper and lower motor neuron signs?


Upper motor neuron signs: spastic paralysis, hyperreflexia, upgoing Babinski
Lower motor neuron signs: flaccid paralysis, fasciculations, absent Babinski

How is ALS diagnosed?


Clinically there should be a combination of upper motor neuron and lower motor neuron symptoms in three
or more extremities. An electromyogram (EMG) will show widespread denervation and fibrillation
potentials in at least three limbs.

What is the main treatment for ALS?


Supportive care

What do ALS patients ultimately die from?


Respiratory failure

What is Guillain-Barré syndrome?


An autoimmune, acute demyelinating disorder affecting the peripheral nerves (particularly motor fibers)

What bacterial infection is Guillain-Barré syndrome associated with?


Campylobacter jejuni

What often precedes Guillain-Barré syndrome?


A bacterial infection causing diarrhea, specifically with Campylobacter, viral infection, or vaccination

Clinically, how does Guillain-Barré syndrome present?


Symmetric ascending paralysis. Symptoms usually begin with distal weakness and progress to proximal
weakness with hyporeflexia and facial diplegia. It can eventually progress to paralysis of the diaphragm,
leading to respiratory failure.

What tests would you do to diagnose Guillain-Barré syndrome?


Lumbar puncture and EMG

What would you see in the CSF after a lumbar puncture?


↑↑protein; normal cell count—this is known as albuminocytologic dissociation

What interventions should be undertaken in a patient with Guillain-Barré syndrome?


Monitor respiratory function very closely and intubate if needed. Medical treatment includes
plasmapheresis and intravenous immunoglobulin (IVIG).

What is the prognosis?


Good prognosis

COGNITIVE DISORDERS

What is dementia?
A syndrome of global intellectual and cognitive deficits which are constant and progressive. Patients have no
sensory abnormalities (no auditory or visual hallucinations)

What specific type of cognitive deficit is dementia usually characterized by?


Memory loss (Remember: dememtia)

What types of cognitive impairments characterize dementia?


Impairments in memory, abstract thought, planning and organization as well as aphasia, apraxia, and agnosia

What are the causes of dementia?


Alzheimer; Parkinson, Huntington; seizure disorder; stroke; B12 deficiency; thiamine deficiency; folate
deficiency; alcoholism; head trauma (especially repetitive); neurotoxins; CNS infections such as syphilis;
CNS malignancies; normal pressure hydrocephalus
Degenerative disorders (Alzheimer, Parkinson, Huntington) (Remember the mnemonic: DEMENTIAS)
Electrolyte imbalances/Endocrine
Mass effect
Epilepsy
Neurotoxins
Trauma
Infection
Alzheimer is most common (70%-80%)
Stroke

What tests would you order if you suspected dementia in a patient?


Head CT, CBC, electrolytes, BUN, creatinine, AST, ALT, B12, folate, rapid plasma region (RPR), thyroid-
stimulating hormone (TSH), urine toxicology screen

What medical problem can mimic dementia?


Depression can present as pseudodementia.

What class of medications should be avoided in demented patients?


Benzodiazepines

What is the general treatment for dementia?


Treat underlying cause if one is found, otherwise supportive treatment. Patients should also learn to use
environmental clues.

What is delirium?
Sudden and transient global cognitive deficits that wax and wane

What specific clinical symptom distinguishes delirium from dementia?


Patients with delirium have sensory deficits which include auditory and visual hallucinations.

What are the symptoms of delirium?


Waxing and waning levels of consciousness and sensory disturbances. Patients are often found to be
anxious, combative, have poor memory, and have decreased attention span.

What are the main etiologies of delirium?


Remember the mnemonic HIDE:
Hypoxia
Infection
Drugs
Electrolyte abnormalities

What tests would you order if you suspected delirium?


CBC, electrolytes, glucose, BUN, creatinine, AST, ALT, TSH, thiamine, B12, urinalysis, chest x-ray (CXR),
urine toxicology screen, pulse oximetry, possibly a head CT

What else would be very important to examine in a patient with delirium?


Patient’s medication list

What two-drug classes are often found to cause delirium?


1. Anticholinergics
2. Benzodiazepines

What is the most common infection leading to delirium in the elderly?


Urinary tract infection (UTI)

What is the main treatment for delirium?


Treat the underlying cause. Antipsychotics can be used to help control symptoms.

How can dementia be distinguished from delirium?


See Table 4-4.

Table 4-4 Characteristics of Dementia Vs Delirium


What is the most common cause of dementia?


Alzheimer

What is found in the cerebral cortex in patients with Alzheimer?


Amyloid plaques and neurofibrillary tangles

What is the most common symptom of Alzheimer?


Memory deficits

What can be seen on CT in a patient with Alzheimer?


Cortical atrophy

What genotype is Alzheimer associated with?


Apolipoprotein E

How can Alzheimer be diagnosed?


It is a clinical diagnosis because it can only be diagnosed definitively at autopsy.

What medications can slow the cognitive decline in Alzheimer?


Anticholinesterase inhibitors: donepezil (Aricept), tacrine

What is the underlying pathology in Parkinson disease?


Degeneration of dopaminergic neurons in the substantia nigra

What are the pathognomonic symptoms of Parkinson disease?


Cogwheel rigidity, resting tremor, bradykinesia, shuffling gait, mask-like faces, postural instability

What are the treatment options for Parkinson disease?


Amantadine; Sinemet (levodopa/carbidopa); benztropine, selegiline, bromocriptine

What is the mechanism of amantadine and what symptom is it best for?


Blocks dopamine reuptake in presynaptic neurons and treats bradykinesia mainly in mild disease

What is the mechanism of Sinemet and what symptom does it best treat?
Sinemet is a combination of levadopa and carbidopa. Levodopa is converted into dopamine in the substantia
nigra. Carbidopa is necessary because it cannot cross the blood-brain barrier and prevents levodopa
metabolism by peripheral tissues. It is also best for treating bradykinesia.

Name some of the anticholinergic drugs.


Benztropine, trihexyphenidyl

What symptom of Parkinson disease do anticholinergics best treat?


Tremor

What is the mechanism of selegiline?


Monoamine oxidase (MAO) inhibitor which blocks dopamine metabolism

What is the mechanism of bromocriptine?


Dopamine agonist

What kind of genetic pattern does Huntington disease follow?


Autosomal dominant

On what chromosome is the genetic alteration found and what is the genetic defect?
Chromosome 4; triple repeat of CAG

In what age range does Huntington disease usually present?


Between 30 and 50 years of age

What is the underlying pathophysiology of Huntington disease?


Atrophy of the caudate nucleus

What are the typical signs and symptoms of Huntington disease?


Choreiform movements, dementia, schizophreniform changes, ataxic gait

What is the treatment for Huntington disease?


Supportive treatment. Antipsychotics can be used as needed for psychotic symptoms.

What is the problem in Wilson disease?


Defect in copper metabolism

What are the symptoms of Wilson disease?


Tremors and rigidity as well as psychiatric changes such as schizophrenia, manic depression; patients have
parkinsonian features

What is the pathognomonic physical examination finding in Wilson disease?


Kayser-Fleischer ring around the cornea

How is Wilson disease diagnosed?


Elevated serum ceruloplasmin

What is the treatment for Wilson disease?


Penicillamine with pyridoxine (vitamin B6) and zinc. Liver transplantation is the final treatment if patient
fails medical therapy.

HEADACHE


What is the most common type of headache?
Tension headache

What are signs and symptoms of a tension headache?


Bilateral, band-like, dull, most intense at neck/occiput, worsened with stress

What psychiatric disorder is it most commonly associated with?


Depression

What is the most common age group with this type of headache?
Between 20 and 50 years of age

What type of headache is characterized by rhinorrhea, being unilateral, stabbing, retro-orbital pain,
ipsilateral lacrimation, ptosis, and nasal congestion?
Cluster headache

What type of headache is characterized by photophobia, nausea, aura, and being unilateral?
Migraine headache

What are some common triggers for migraines?


Menstruation, stress, foods, alcohol

What type of headache is associated with jaw claudicating?


Temporal arteritis (usually a unilateral temporal headache with temporal artery tenderness)

What is the predilection for temporal arteritis?


Female > male

What are the risks of temporal arteritis?


Optic neuritis and blindness if not treated
What is it associated with?
Polymyalgia rheumatica
How is it diagnosed?
Must do a temporal artery biopsy; elevated ESR is just a screening test

What is the treatment for temporal arteritis?


High-dose steroids

INTRACRANIAL BLEEDING

What is “the worst headache of my life”?


Subarachnoid hemorrhage (SAH)

What is the most common cause of SAH?


Head trauma

What is the most common underlying cause of a spontaneous SAH?


Aneurysm rupture

What is the most common heritable disorder associated with SAH?


Autosomal dominant polycystic kidney disease
How is an SAH diagnosed?
CT scan shows subarachnoid blood (dark); lumbar puncture shows bloody CSF with xanthochromia;
cerebral angiography can be done to find berry aneurysms.

What is the immediate treatment for an SAH?


ICU admission. Goal is to decrease intracranial pressure (ICP). Give nimodipine to decrease chance of
vasospasm by controlling blood pressure, raise the head of the bed, and administer IV fluids as needed.

What is the second-line treatment for an SAH?


Surgical evacuation of blood via burr holes

What is a berry aneurysm?


Outpouching of vessels in the circle of Willis, usually at bifurcations (looks like a berry)

What medical condition is a berry aneurism associated with?


Polycystic kidney disease, Marfan disease

What is a symptom of berry aneurysm rupture?


Third nerve palsy

What is the most common location for a berry aneurysm?


Anterior communicating artery (30%), followed by posterior communicating artery, then middle cerebral
artery

What type of hemorrhage is associated with a lateral skull fracture?


Epidural hematoma

What artery is involved in an epidural hematoma?


Middle meningeal artery

What is the sequence of events in an epidural hematoma?


The patient has a lucid interval lasting from minutes to hours followed by a loss of consciousness and
hemiparesis.

What can cause a “blown” pupil in a patient with an epidural hematoma?


Uncal herniation

What is seen on CT in a patient with an epidural hematoma?


Convex (lens shaped) hyperdensity that does not cross the midline

What is the treatment for an epidural hematoma?


Surgical evacuation of the hematoma via burr holes

What vessels are involved in a subdural hemorrhage?


Bridging veins

In what population are subdural hematomas most common?


The elderly and alcoholics

What is the course of events in a subdural hematoma?


Patient can have symptoms similar to dementia since mental status changes and hemiparesis can present
subacutely.
What is seen on CT in a patient with a subdural hematoma?
Crescent-shaped, concave hyperdensity that may cross the midline

VERTIGO

What type of vertigo is characterized by horizontal nystagmus?


Peripheral vertigo

What type of vertigo is characterized by vertical nystagmus?


Central vertigo

What is the most common cause of vertigo?


Benign positional vertigo

What are the signs of benign positional vertigo?


Sudden, episodic vertigo that occurs with quick head movement and lasts for seconds

How is benign positional vertigo diagnosed?


Dix-Hallpike maneuver—From a sitting position, the physician turns the patients head 45° to the right as the
patient lies back to the supine position. After 30 seconds the patient is returned to the sitting position and
observed. Vertigo with or without nystagmus is a positive test.

What is the treatment of benign positional vertigo?


Epley maneuvers

What is the etiology of Meniere disease?


Excess endolymph causes dilation of the membranous labyrinth

What is the triad of symptoms?


1. Tinnitus
2. Hearing loss
3. Episodic vertigo lasting hours

What does audiometry show in Meniere disease?


Low-frequency pure-tone hearing loss

What is the treatment for Meniere disease?


Low salt intake and acetazolamide. If acute, you could use antihistamines, anticholinergics, or antiemetics.
Surgery may be necessary.

What type of vertigo follows a viral respiratory illness?


Viral labyrinthitis

How long does the vertigo last?


Days to weeks

What is the treatment for viral labyrinthitis?


Meclizine

CLINICAL VIGNETTES

A 64-year-old male smoker with past medical history of hypertension, hyperlipidemia, and type 2 diabetes
presents to the ER with a right-sided facial droop as well as weakness of the right arm and leg. Symptoms
began about 3 hours prior. The patient currently takes aspirin 81 mg as a part of his daily regimen. What
is this patient’s strongest modifiable risk factor for his current condition?
Hypertension

A 22-year-old female patient with no past medical history comes to your office complaining of increased
“fatigue” particularly with repeated effort of muscle use or activity. She has noticed that her vision is
blurry lately and that her eye lids seem to be droopy. You examine her and find that her muscle strength
seems to deteriorate with repeated efforts. Sensation is normal. What condition do you initially suspect?
Myasthenia gravis

A 36-year-old male with a past medical history of psoriasis comes in complaining of “dizziness.” He is
particularly dizzy and nauseous when he lies in bed and turns his head from left to right. He denies any
tinnitus or hearing loss or any other neurologic symptoms. You suspect he has benign positional vertigo.
What physical examination test could help with this diagnosis?
Dix-Hallpike maneuver

An 85-year-old female presents to your office complaining of new-onset headache over the past month.
Headache tends to be on the left side only. She denies any problems with her vision. She has no
photophobia with headaches nor is she bothered by sound. On review of symptoms she does complain of
some jaw pain. What condition should be ruled out in this case?
Temporal arteritis

A 76-year-old female who lives in a retirement community is admitted for an UTI. In the evening time, the
patient becomes combative and disoriented. What is the most likely diagnosis?
Delirium
CHAPTER 5
Gastroenterology

ESOPHAGEAL DISORDERS

What is dysphagia?
Difficulty swallowing

What is odynophagia?
Pain with swallowing

How does oropharyngeal dysphagia present?


More difficulty initiating the swallowing of liquids than solids

How does esophageal dysphagia present in terms of swallowing?


Difficulty swallowing both liquids and solids

What are the causes of oropharyngeal dysphagia?


Neurologic disorders (muscular, cranial nerve diseases), Zenker diverticulum, thyromegaly, sphincter
dysfunction, oropharyngeal cancers

What is a Zenker diverticulum?


Lower pharyngeal outpouching of the muscular wall. Pulsion diverticulum secondary to pressure from
swallowing.

What are the signs and symptoms of Zenker diverticulum?


Halitosis, neck mass on the left, dysphagia, aspiration

How is Zenker diverticulum diagnosed?


Clinical palpation of a left-sided neck mass or a barium swallow

What is the treatment for Zenker diverticulum?


Cricopharyngeal myotomy or surgical excision

What are the causes of esophageal dysphagia?


1. Mechanical obstruction: esophageal cancer, Schatzki ring, peptic stricture
2. Problem with esophageal motility: achalasia, diffuse esophageal spasm, or scleroderma

How do symptoms of mechanical dysphagia differ from dysphagia secondary to motility problems?
Patients with mechanical dysphagia have more difficulty with solids than liquids whereas motility disorders
cause difficulty with both solids and liquids.

What is the most common motility disorder often seen in patients with scleroderma?
Esophageal hypomotility

What defines achalasia?


Loss of esophageal peristalsis with an inability of the lower esophageal sphincter to relax due to ganglionic
loss of Auerbach plexus.
What is the diagnostic feature seen on barium swallow in a patient with achalasia?
“Bird’s beak” appearance (dilation of the proximal esophagus with narrowing of the distal esophagus)

What would manometry demonstrate in a patient with achalasia?


Increased pressure at the lower esophageal sphincter with no relaxation with swallowing

How is achalasia treated?


Balloon dilatation, sphincter myotomy, local botulinum toxin

What is the diagnostic feature seen on barium swallow in a patient with diffuse esophageal spasm?
“Corkscrew pattern”

What is the treatment for diffuse esophageal spasm?


Nitroglycerin, calcium channel blockers

What is Schatzki ring?


Narrowing of the lower esophageal ring

What is Plummer-Vinson syndrome?


Esophageal webs, atrophic glossitis, and dysphagia associated with iron deficiency anemia

GASTROESOPHAGEAL REFLUX DISEASE

What are the underlying causes of gastroesophageal reflux disease (GERD)?


Incompetent lower esophageal sphincter, obesity, hiatal hernia, pregnancy, decreased esophageal motility,
delayed gastric emptying

What are the signs and symptoms of GERD?


Postprandial epigastric (chest) burning worse in supine position, cough, hoarse voice, regurgitation

How is GERD diagnosed?


It is a clinical diagnosis.

What is the treatment for GERD?


First-line treatment is lifestyle modification; weight loss; avoidance of instigating foods such as caffeine,
fatty foods; avoid eating right before going to sleep
Second-line treatment: H2 blockers Third-line treatment: If H2 blockers fail, try proton pump inhibitors
(PPIs)
Last resort: Nissen fundoplication

What can be the long-term effects of chronic GERD?


Barrett esophagus, peptic stricture, and esophageal cancer

What is Barrett esophagus?


Transformation of normal squamous epithelium to columnar epithelium

What is the risk with Barrett esophagus?


10% lifetime risk of transforming into esophageal adenocarcinoma

GASTRITIS

What is gastritis?
Inflammation of the gastric mucosa

Name the type of gastritis that matches the statement below:

What are the signs and symptoms of gastritis?


May be asymptomatic; otherwise symptoms are epigastric pain, weight loss, nausea, vomiting, hematemesis,
coffee ground emesis

How is gastritis diagnosed?


Endoscopy

What is the treatment for gastritis?


It depends on the etiology.
If caused by H pylori—triple therapy with PPI, two antibiotics, and bismuth compound
If caused by NSAID use—discontinue NSAID use; start sucralfate, PPI, or H2 blocker
If caused by post-procedure, hospitalization stress—intravenous (IV) H2 blocker
If caused by pernicious anemia—vitamin B12 treatment

PEPTIC ULCERS

What are the two types of peptic ulcers?


1. Duodenal ulcer
2. Gastric ulcer

Which type of ulcer is more common?


Duodenal ulcers are twice as common.
What is the underlying pathology in a patient with a duodenal ulcer?
Most have increased acid production.

How does the underlying pathology of gastric ulcers differ from that of duodenal ulcers?
Gastric ulcers are not caused by increased acid production. Patients are more likely to have decreased
mucosal protection.

What bacterial infection is found in 90% of patients with duodenal ulcers?


H pylori

What percentage of gastric ulcers are associated with H pylori infection?


70%

What test can determine if a patient may be infected with H pylori?


Stool H pylori antigen, urea breath test, serum IgG test

What is the drawback of the H pylori blood test?


It does not indicate an active infection. It will be positive even if the patient was infected in the past and is
not currently infected. The test also has a low sensitivity.

What are the two most common causes of peptic ulcer disease?
1. H pylori infection
2. Frequent NSAID use

What are the risk factors for a peptic ulcer?


Smoking, significant alcohol use, frequent NSAID use, significant physiologic stress (examples are surgery,
trauma, burns), and hypersecretory states

Name three hypersecretory states.


1. Zollinger-Ellison syndrome
2. Multiple endocrine neoplasia type I (MEN I)
3. Antral G-cell hyperplasia

What are the signs and symptoms of a duodenal ulcer?


Burning epigastric pain that is usually 2-3 hours postprandially; relieved by food or antacids; pain may
radiate to the back; pain awakens patient at night; nausea and sometimes vomiting; hematemesis/melena if
patient has a gastrointestinal (GI) bleed

What are the signs and symptoms of a gastric ulcer?


Same as that for a duodenal ulcer except that pain is greater with meals, so patients often lose weight

What tests would you order if you suspected a peptic ulcer?


Complete blood count (CBC) to make sure patient is not anemic; upper GI endoscopy or upper GI series; H
pylori screening

What should be ruled out in a patient with a gastric ulcer?


Malignancy

How can malignancy be ruled out?


A biopsy of the ulcerated region should be done during endoscopy

What is a gastric ulcer in a burn patient called?


Curling ulcer
What is a gastric ulcer in a patient with central nervous system (CNS) damage called?
Cushing ulcer

How is peptic ulcer disease treated?


Avoidance of instigating factors such as smoking and NSAIDs; H2 blockers or PPIs, mucosal protectors
such as bismuth, and antibiotics if the patient is infected with H pylori

How is an H pylori infection treated?


Triple therapy: PPI + bismuth compound + two antibiotics for 14 days For example: omeprazole +
amoxicillin (or metronidazole) + clarithromycin + bismuth compound

What are some complications of peptic ulcer disease?


Hemorrhage, obstruction, perforation

When would you suspect a perforated duodenal ulcer?


Severe epigastric pain that radiates to the back

What studies would you order if you suspected a perforated ulcer?


Abdominal series or upper GI series with contrast (do not use barium)

What would you expect to see on an abdominal series if there was a perforated ulcer?
Free air under the diaphragm

What is the treatment for a perforated ulcer?


Npo (nothing by mouth), IV fluids, antibiotics, emergent surgery

What are the typical symptoms of gastric outlet obstruction?


Nausea, vomiting, weight loss, distended abdomen, loud bowel sounds

What is the most serious complication of a posterior duodenal ulcer?


Erosion into the gastroduodenal artery can lead to a massive hemorrhage.

What symptoms could be a red flag for a gastric malignancy?


Early satiety with weight loss

What are the risk factors for gastric cancer?


Diets with high nitrosamines or salt content, history of chronic gastritis, low-fiber diets, gastric ulcer,
smoking, blood group A, Japanese ethnicity

What blood group type is more likely to develop gastric cancer?


Type A

In what part of the stomach is gastric cancer usually found?


In the antrum of the stomach

What is the most common type of gastric cancer?


Adenocarcinoma

Name the physical findings associated with metastatic gastric cancer described below:

What is the most fatal form of gastric cancer?


Linitis plastica (diffusely infiltrating gastric cancer)

GI BLEED

What are signs of an upper GI bleed?


Hematemesis, coffee ground emesis, melena (black, tarry stools), bright red blood per rectum (BRBPR) only
if the bleed is very brisk

What are the six main causes of upper GI bleeds?


Remember the mnemonic PAGE ME!
1. Peptic ulcer
2. Atrioventricular (AV) malformation
3. Gastritis
4. Esophageal varices
5. Mallory-Weiss tear
6. Esophagitis

What is a Mallory-Weiss tear?


Small esophageal tear usually near the gastroesophageal (GE) junction that is caused by vomiting or retching

What blood tests would you order in a patient you thought may have a GI bleed?
CBC (look for anemia, platelet abnormality), blood urea nitrogen (BUN) (fresh bleeding may lead to
elevated BUN), prothrombin time (PT), partial thromboplastin time (PTT), international normalized ratio
(INR), bleeding abnormalities

What is the best diagnostic test in a patient with upper GI bleed?


Endoscopy

How are bleeding varices treated?


Ligation or injection of vessels with sclerosing or vasoconstrictive agents

How should all GI bleeds be treated?


Emergency airway, breathing, circulation (ABCs) as well as IV fluid resuscitation, gastric lavage and
nasogastric (NG) tube if needed

What are the signs of a lower GI bleed?


BRBPR, maroon or dark red stool, anemia

What are the six most common causes of lower GI bleeding?


1. Diverticulosis
2. AV malformation
3. Hemorrhoids
4. Colitis
5. Colon cancer
6. Colonic polyps

What is the most common cause of a major lower GI bleed in a patient older than 60?
Diverticulosis

What physical examination and imaging study would you do on a patient with suspected lower GI bleed?
Always do a rectal examination; colonoscopy

If no clear source is found, what other studies can be done?


Endoscopy to rule out an upper GI source, tagged red blood cell (RBC) scan; arteriography, gastric lavage;
barium enema (but not if there is acute blood loss)

COLON

What is a true diverticulum?


Colonic herniation involving the full thickness of bowel wall

What is a false diverticulum?


Colonic mucosal herniation through the muscular layer which is acquired

Which type of diverticulum is more common?


False

In what part of the colon are diverticula most commonly found?


Sigmoid

What is diverticulosis?
Presence of multiple diverticula in the colon

What is thought to be an important risk factor for the development of diverticulosis?


Low-fiber diet

Why do diverticula bleed?


Diverticula which are inflamed erode through an artery and cause profuse bleeding that usually subsides on
its own.

What is the treatment for diverticulosis?


Increase of fiber in diet and decrease of obstructing foods such as seeds and fatty foods

What is diverticulitis?
“-itis” implies inflammation. Diverticulitis is inflammation of a diverticulum secondary to infection.

What is the most common symptom of diverticulitis?


The most common presenting symptom is left lower quadrant abdominal pain.

What are other signs and symptoms of diverticulitis?


Constipation, fever, elevated white blood cells (WBCs), bleeding is much less common than with
diverticulosis

What are the four serious complications of diverticulitis?


1. Perforation through the bowel wall causing peritonitis
2. Fistula formation
3. Abscess
4. Obstruction

How do patients who develop a colovesicular fistula present?


Multiple urinary tract infections (UTIs)

What is the best imaging test to diagnose diverticulitis?


Computed tomography (CT) of the abdomen and pelvis

What studies are contraindicated in diverticulitis?


Colonoscopy, contrast enema

What is the treatment for diverticulitis?


Npo, IV fluids, antibiotics to cover anaerobes and enteric organisms

What is the treatment for recurrent bouts of diverticulitis or severe cases?


Elective sigmoid colectomy

How would you treat an abscess secondary to diverticulitis?


CT or ultrasound-guided percutaneous drainage

How do you treat obstruction or perforation secondary to diverticulitis?


Surgical resection of affected bowel with a colostomy that is usually temporary

What is the most common nosocomial enteric infection?


Clostridium difficile

What can a C difficile infection lead to?


Pseudomembranous colitis

What antibiotic is classically associated with C difficile infection?


Clindamycin

What are the symptoms of C difficile infection?


Diarrhea and abdominal cramping/pain

How is a C difficile infection diagnosed?


C difficile stool toxin, stool leukocytes

How is a C difficile infection treated?


Stop the offending agent and treat with po metronidazole or vancomycin.
How is pseudomembranous colitis confirmed?
On colonoscopy or sigmoidoscopy, a yellow plaque adherent to the colonic mucosa can be seen.

What is volvulus?
Twisting of the bowel around the mesenteric base

What is the most common location of volvulus?


Sigmoid colon

What is the second most common location of volvulus?


Cecum

What are the symptoms of a volvulus?


Painful, distended abdomen; high-pitched bowel sounds; tympany on percussion

What is the classic sign of volvulus on an abdominal series?


Dilated loops of bowel with a kidney-bean appearance

What is the sign of volvulus on a barium enema?


Bird’s beak appearance with the beak pointing to the area where the rotation has occurred

What is the treatment for volvulus?


Sigmoidoscopy or colonoscopy is usually therapeutic for decompression.

What is the second most common cancer causing death in the United States?
Colon cancer

What are the risk factors for colon cancer?


Family history
Low-fiber diet
Familial adenomatous polyposis (FAP)
Hereditary nonpolyposis colorectal
cancer
High-fat diet
Colonic adenomas
Age >50
Inflammatory bowel disease

What are the general signs and symptoms of colon cancer?


Weight loss, fatigue, iron deficiency anemia in a male >50 years of age is colon cancer (CA) until proven
otherwise; GI bleed, constipation, distended abdomen secondary to obstruction, pencil-thin stools

How do the symptoms of right-sided-and left-sided colon cancer differ?


Left-sided colon cancer presents as constipation.
Right-sided colon cancer presents as anemia secondary to blood loss.

What are the recommendations for colon cancer screening?


Starting age 50, a colonoscopy every 10 years or a sigmoidoscopy every 5 years with annual digital rectal
and hemoccult examination

How are the screening recommendations different in patients with a family history of colon cancer?
Start screening 10 years prior to the age that the family member was diagnosed with cancer.
How is colon cancer diagnosed?
Biopsy of the lesion on colonoscopy/sigmoidoscopy

What laboratory marker can be used to help follow the progression of colon cancer and its treatments?
Carcinoembryonic antigen (CEA)—but it cannot be used as a screening test

How is colon cancer staged and what is the prognosis of each stage?
TNM (tumor node metastasis) classification (Tables 5-1A and 5-1B)

Table 5-1A Colon Cancer Staging

Table 5-1B Colon Cancer Prognosis Based on Staging


What is the treatment of colon cancer?


Surgical resection; radiation therapy (if rectal cancer), and chemotherapy for stages B and C

INFLAMMATORY BOWEL

What is ulcerative colitis (UC)?


Inflammatory bowel disease that affects the colon

What classic symptom is associated with UC?


Bloody diarrhea

What other serious symptom can sometimes occur with UC?


Toxic megacolon

Where are lesions found in UC?


Large intestine only

Where do the lesions usually first appear?


Rectum

How do lesions spread in UC?


Proximally from the rectum

How is UC diagnosed?
Colonoscopy with biopsy

What is seen on colonoscopic biopsy in a patient with UC?


Crypt abscess; distorted cells
How is the mucosa of the colon described in a patient with UC?
Friable mucosa with erosions and erythema

On biopsy, what is the depth of involvement of the lesions?


Mucosa and submucosa only

What is ulcerative proctitis?


A subtype of UC in which only the rectum is involved

What is the treatment for each of the following severities of UC:


Fulminant colitis?
Broad-spectrum antibiotics, surgery

What is Crohn disease?


Inflammatory bowel disease that affects the GI tract; there could be an infectious etiology

What part of the GI tract can Crohn disease involve?


From the mouth to the rectum, but often with rectal sparing

What is the classic symptom of Crohn disease?


Bloody or watery diarrhea (although the diarrhea does not always have to be bloody)

What are some other physical examination findings in Crohn disease?


Fistulas, fissures, fever, abdominal pain

How is Crohn disease diagnosed?


Colonoscopy and biopsy

How are the lesions classically spread in Crohn disease?


There are skip lesions, which means that there is no contiguous spread. The lesions are disseminated through
the entire colon.

What is the depth of the lesions on biopsy?


Lesions go through all layers—they are transmural.

On physical examination, what type of lesion is often found in the mouth of a patient with Crohn disease?
Aphthous ulcer

What is the mnemonic to remember Crohn disease?


The old, Crohn skipped over the cobblestone.
What is the treatment for Crohn disease?
Sulfasalazine, corticosteroids; for unresponsive patients, try mercaptopurine, azathioprine, infliximab

What are the differences between UC and Crohn disease?


See Table 5-2.

Table 5-2 Crohn Disease Vs Ulcerative Colitis

Name six extraintestinal manifestations of both UC and Crohn disease?


1. Erythema nodosum
2. Pyoderma gangrenosum
3. Uveitis
4. Ankylosing spondylitis
5. Primary sclerosing cholangitis
6. Arthritis

DIARRHEA

What is the definition of diarrhea?


Daily stool weighing >200 g

What are the most common causes of bacterial and parasitic bloody diarrhea?
Remember the mnemonic whY CaSES:
Yersinia
Campylobacter, cholera
Shigella
Escherichia coli, Entamoeba histolytica
Salmonella

What is the treatment for bacterial bloody diarrhea?


Ciprofloxacin or bactrim

What are viral causes of bloody diarrhea?


Rotavirus and Norwalk virus
What is the treatment for bloody diarrhea caused by a virus?
IV fluids

What is the treatment for parasitic bloody diarrhea?


Metronidazole

What studies would you order in a patient with bloody diarrhea?


CBC, stool for ova and parasites, stool for fecal leukocytes, stool culture

What acid-base disorder can you expect to see in a patient with severe diarrhea?
Metabolic acidosis

MALABSORPTION DISORDERS

Name the malabsorption disorder described below:

PANCREAS

What is pancreatitis?
Inflammation of the pancreas

What are the two most common causes of pancreatitis?


1. Alcoholic pancreatitis
2. Gallstone pancreatitis

What is the mnemonic for the causes of pancreatitis?


I GET SMASHED
Idiopathic
Gallstones
Ethanol
Trauma
Steroids
Mumps
Autoimmune
Scorpion bites
Hyperlipidemia
Endoscopic retrograde
cholangiopancreatography (ERCP)
Drugs (such as thiazide diuretics)
What are the signs and symptoms of pancreatitis?
Epigastric pain that radiates to the back; nausea, vomiting, decreased bowel sounds, fever

What are signs of retroperitoneal bleeding?


Grey Turner sign and Cullen sign
What is Grey Turner sign?
Ecchymosis seen on the patient flank in hemorrhagic pancreatitis
What is Cullen sign?
Periumbilical ecchymosis seen in hemorrhagic pancreatitis

What laboratory findings are consistent with pancreatitis?


↑ amylase, ↑ lipase, hypocalcemia

What would you expect to see on an abdominal x-ray?


Sentinel loop or colon cutoff sign

What is a sentinel loop?


Dilated bowel or air fluid levels near the pancreas

What is the colon cutoff sign?


Transverse colon distended with no colonic gas distal to the splenic flexure

What is the best study to evaluate pancreatitis?


Abdominal CT

What test should be ordered if there is a suspicion of gallstone pancreatitis?


Right upper quadrant (RUQ) ultrasound

What is the treatment for pancreatitis?


Npo, NG tube for ileus or vomiting, IV fluid hydration, and treat the underlying cause

What do we use to determine the prognosis of a patient with pancreatitis?


Ranson criteria (predicts risk of mortality based on risk factors)

What are Ranson criteria on admission?


Remember the mnemonic GA LAW
Glucose >200
Age >55
Lactate dehydrogenase (LDH) >350
Aspartate aminotransferase (AST) >250
WBC >16,000

What are Ranson criteria after 48 hours?


Remember the mnemonic: C and HOBBS
Calcium <8
Hematocrit (Hct) drop >10%
Oxygen <60 mm
BUN >5
Base deficit >4
Sequestration of fluid >6 L

How is the risk of mortality calculated based on Ranson criteria?


<3 risk factors: 1% mortality
3-4 risk factors: 16% mortality
5-6 risk factors: 40% mortality
7-8 risk factors: close to 100% mortality

BILIARY TRACT

What is cholelithiasis?
Gallstones

What are the four classic risk factors for cholelithiasis?


1. Female
2. Fat
3. Fertile
4. Forty

What is the most common type of stone?


Cholesterol stone

What other type of stone can be found?


Pigment stone

What is the predisposition to pigment stones?


Hemolytic anemia or hemoglobinopathies

Which type of stone is radiopaque?


Pigment stones

What are the common signs and symptoms of cholelithiasis?


RUQ pain, nausea, and vomiting especially after a fatty meal

What is the most specific and sensitive test to diagnose cholelithiasis?


RUQ ultrasound

When should cholelithiasis be treated?


Only if the patient is symptomatic

What is the treatment for cholelitihiasis?


Elective cholecystectomy

What is cholecystitis?
Gallbladder inflammation secondary to infection caused by an obstructing stone

What bacteria cause cholecystitis?


Remember the mnemonic KEEEP:
Klebsiella
E coli
Enterococcus
Enterobacter
Pseudomonas

What are the symptoms of cholecystitis?


Prolonged RUQ pain, fever, nausea, vomiting, referred pain to subscapular region on the right + Murphy’s
sign

What is Murphy’s sign?


Acute pain and inspiratory arrest with deep palpation of RUQ during inspiration

How is cholecystitis diagnosed?


RUQ ultrasound, which will show gallstones, gallbladder wall thickening, pericholecystic fluid, and
sonographic Murphy’s sign

What imaging study should be performed if the ultrasound results are equivocal?
Hepatobiliary iminodiacetic acid (HIDA) scan

What is the treatment for cholecystitis?


Npo, IV fluids, IV antibiotics (third-generation cephalosporin + aminoglycoside + metronidazole,
cholecystectomy

What pain medicine has historically been referred to as being more appropriate to treat pain from
cholecystitis and why?
Demerol because morphine is thought to cause spasm of the sphincter of Oddi; however, this is not always
done in clinical practice

What is choledocholithiasis?
Gallstones in the common bile duct

What are the signs and symptoms of choledocholithiasis/cholangitis?


Jaundice secondary to obstruction, RUQ pain, Murphy’s sign, hypercholesterolemia, ↑ alkaline phosphatase,
↑ bilirubin, ↑ alanine aminotransferase (ALT)

What is the treatment for choledocholithiasis?


1. ERCP with papillotomy and stone removal
2. Common bile duct exploration at time of surgery

What are the complications of choledocholithiasis?


Ascending cholangitis and pancreatitis

What is ascending cholangitis?


Bacterial infection of the biliary tract secondary to obstruction

What is the most common organism causing cholangitis?


E coli

What is Courvoisier sign?


Gallbladder enlargement with jaundice secondary to carcinoma of the head of the pancreas leading to a firm
palpable gallbladder

What are the classic symptoms of ascending cholangitis?


Charcot triad:
1. Jaundice
2. Fever
3. RUQ tenderness
Or
Reynold’s pentad (Charcot triad + altered mental status and shock)
What are the laboratory findings consistent with ascending cholangitis?
↑ WBC, ↑ alkaline phosphatase, ↑ direct bilirubin, ↑ ALT

How is ascending cholangitis definitively diagnosed?


ERCP or percutaneous transhepatic cholangiogram (PTC)

What is the treatment for ascending cholangitis?


Npo, IV fluids, IV antibiotics (ampicillin + aminoglycoside + metronidazole), and ERCP to remove stones

What is primary sclerosing cholangitis?


Chronic inflammation and fibrosis of the biliary tree

What is a common medical diagnosis that patients with sclerosing cholangitis also have?
UC

LIVER

What is cirrhosis?
Chronic hepatic injury leading to fibrosis, necrosis, and nodular regeneration

What is the most common cause of cirrhosis?


Alcoholism

What are some nonalcoholic causes of cirrhosis?


Alpha-1 antitrypsin deficiency, hemochromatosis, primary or secondary biliary cirrhosis, Wilson disease,
hepatitis B, hepatitis C

What are the signs and symptoms of cirrhosis?


Jaundice, ascites, asterixis, bleeding, edema, hepatomegaly, encephalopathy, palmar erythema, spider
angiomata on the abdomen

What is asterixis?
Downward flapping of hands when held in a dorsiflexed position

Why do cirrhotic patients get ascites?


Because they have low albumin

What is SAAG?
Serum-ascites albumin gradient

How is SAAG calculated?


Albumin concentration of serum – albumin concentration of ascites fluid

What is SAAG >1.1g/dL indicative of?


Ascites related to portal hypertension

What is a SAAG <1.1 g/dL indicative of?


Non-portal hypertension etiologies of ascites such as nephrotic syndrome, malignancy, tuberculous
peritonitis, biliary or pancreatic ascites

How can the ascites be treated?


Spironolactone and paracentesis

What is a major complication of ascites?


Spontaneous bacterial peritonitis (SBP)

What is the most common organism causing SBP?


E coli

What is the most classic sign of SBP?


Rebound abdominal tenderness in a patient with ascites

How is SBP diagnosed?


Paracentesis with fluid sent for cell count and Gram stain, culture, and sensitivity

What are the diagnostic criteria for SBP?


Ascites fluid neutrophil count >250 and positive Gram stain/culture OR ascites fluid neutrophil count >500

What is the treatment for SBP?


Third-generation cephalosporin with albumin

Why do cirrhotic patients tend to bleed?


PT is elevated and platelets are low.

What is the treatment for cirrhosis?


Stop alcohol consumption; multivitamin including thiamine and B12; and nutrition

What marker can detect an alcohol binge?


Gamma-glutamyltransferase (GGT)

What is portal hypertension?


Elevated portal vascular resistance secondary to presinusoidal, postsinusoidal, or sinusoidal obstruction
Presinusoidal: portal vein thrombosis, schistosomiasis
Postsinusoidal: hepatic vein thrombosis, right heart failure
Sinusoidal: cirrhosis

Internationally, what is the most common cause of portal hypertension?


Schistosomiasis

What are the classic physical examination findings in a patient with portal hypertension?
Remember the mnemonic CHASE:
Caput medusa
Hemorrhoids
Ascites
Splenomegaly
Esophageal varices

What are the treatments for portal hypertension?


Decrease portal pressure with propranolol; transjugular intrahepatic portosystemic shunt (TIPS); last resort is
a liver transplant

What is a common cause of hematemesis in a patient with portal hypertension?


Variceal bleeding
How is a variceal bleed diagnosed?
Esophagogastroduodenoscopy (EGD)

What is the treatment for a variceal bleed?


Start with emergent therapy assessing airway, breathing and circulation, initiate IV fluids, if indicated
correct clotting factors with fresh frozen plasma (FFP), vitamin K. Vasoactive drugs such as octreotide,
somatostatin, or vasopressin have been found to be safer than sclerotherapy to control the bleeding.
Sclerotherapy and variceal banding are also options.
Nonselective beta blockers such as propranolol, timolol, and nadolol.

What is the treatment for esophageal varices with no history of bleeding?


If contraindicated—long-acting nitrates
Alternative—variceal ligation

What are some treatments for hepatic encephalopathy?


Lactulose to decrease absorption of ammonia, neomycin, and protein-restricted diet

What is hepatorenal syndrome?


Patients with advanced hepatic disease develop acute renal failure.

How is hepatorenal syndrome diagnosed?


Elevated BUN/creatinine (CR), hyponatremia, oliguria, hypotension, and urine Na <10

What are the three different etiologic categories of hepatitis?


1. Viral (elevated ALT; ALT:AST 2:1)
2. Alcoholic (elevated AST; AST: ALT 2:1)
3. Toxin-induced (Tylenol) (elevated AST)

What is a mnemonic to remember the etiological differences in hepatitis?


virALT
Drugs blAST

Name the hepatitis viruses transmitted via the fecal-oral route.


Hepatitis A and E

Name the hepatitis viruses transmitted via blood and sexual contact.
Hepatitis B, C, D

Name the only DNA hepatic virus.


Hepatitis B

Which hepatitis viruses have a chronic carrier state?


Hepatitis B, C, D

Which hepatitis viruses have a vaccine available?


Hepatitis A and B (and D)

Which hepatitis viruses have a vaccine available?


Hepatitis A and B (and D)

How can you detect an acute hepatitis A infection?


Anti-hepatitis A virus (HAV) IgM
How can you detect immunity to hepatitis A?
Anti-HAV IgG

How is hepatitis A treated?


It is a self-limiting disease.

Which disease state does each of the following hepatitis B markers detect?

What can be given to a patient exposed to hepatitis B to prevent infection?


Hepatitis B immunoglobulin (HBIG)

What is the treatment for a person infected with hepatitis B?


Interferon, lamivudine, adefovir

When is the window period for hepatitis B?


The time when HBsAg has become undetectable but HBsAb is not yet detectable (Fig 5-1)

Figure 5-1 Scheme of typical clinical and laboratory features of acute hepatitis B. (Reproduced, with
permission, from Fauci AS, Braunwald E, Kasper DL. Harrison’s Principles of Internal Medicine. 17th ed.
New York: McGraw-Hill; 2008:1934.)

What is the worst complication of hepatitis B?


Hepatocellular carcinoma

Which hepatitis virus carries the highest risk of developing into hepatocellular carcinoma?
Hepatitis B

What is the treatment for a person infected with hepatitis C?


Interferon + ribavirin

Which hepatitis virus must have concomitant infection with hepatitis B?


Hepatitis D

CLINICAL VIGNETTES

A 19-year-old male presents to your office complaining of abdominal pain. He describes it as being crampy
pain and furthermore he has had diarrhea that is runny in nature. He thinks he has had fever on some
occasions as well. You examine the patient and find that this thin male has lower abdominal tenderness
with no rebound or guarding. Stool for occult blood is tested and found to be negative. In his mouth you
find an aphthous ulcer. What is the most likely diagnosis?
Crohn disease

A 40-year-old obese female presents to the ER complaining of abdominal pain with nausea and vomiting
that began 5 hours ago after she ate a large hamburger and French fries. She has had similar symptoms in
the past but none that ever lasted this long. Her examination demonstrated the following:

Temp: 101.9°F; BP: 143/85 mm Hg; HR: 80 beats/min; RR: 18; O2 sat: 100%

General: appears to be in pain

Cardiovascular (CV): regular rate and rhythm, no murmurs

Pulmonary: clear bilaterally

Abdomen: soft, tender to palpation in the right upper quadrant; normal bowel sounds

Ext: no clubbing, cyanosis, or edema

What imaging modality would you utilize to try to make the diagnosis?

Ultrasound—It is highly sensitive and specific for detecting cholecystitis.

Your patient complains of midepigastric pain relieve by food for several months. He also complains of
nausea and occasional back pain. He is under an immense amount of stress because of the failing economy
and all his expenses. A stool H pylori comes back positive. What is the appropriate treatment?
Triple therapy: amoxicillin and clarithromycin + bismuth compound + proton pump inhibitor. This patient is
also at risk for a duodenal ulcer and should have CBC as well as upper GI endoscopy.

A 35-year-old female comes to the ER complaining of epigastric pain that radiates to the back along with 2
days of nausea, vomiting, and documented fever. The patient admits to drinking two “fifths” of vodka
every day. She smells strongly of alcohol. A blood draw demonstrates megaloblastic anemia on CBC,
normal kidney function. Liver function tests (LFTs) are elevated also with elevated amylase and lipase.
What is the best diagnostic test to make the diagnosis?
Abdominal CT to look for pancreatitis

A 56-year-old male with a history of alcoholism presents to the ER with hematemesis. On examination the
patient has a fluid wave, and there are spider angiomata present on his abdomen. He is also clearly
jaundiced. His blood pressure is 90/50 with a heart rate of 105 beats/min. You suspect that he has a
variceal bleed. What is your first step in treating this patient?
ABCs—Airway establishment, breathing, circulation as well as volume resuscitation with IV fluids
CHAPTER 6
Hematology-Oncology

ANEMIA

What are the three basic categories of anemia?


1. Microcytic (mean corpuscular volume [MCV] <80)
2. Macrocytic (MCV>100)
3. Normocytic (MCV between 80 and 100)

Match the following anemias with their correct category (microcytic, macrocytic, or normocytic):
See Table 6-1.
Iron deficiency anemia
Thalassemia
Folate deficiency
Sideroblastic anemia
Anemia of chronic disease
Lead poisoning
B12 deficiency
Chronic renal failure

Table 6-1 Etiologies of Different Types of Anemia

MICROCYTIC ANEMIAS

What is the most likely etiology of iron deficiency anemia?


In women of childbearing age, it is most likely because of menses; in children it is usually a dietary
deficiency; and in the elderly it is colon cancer until proven otherwise.

A 68-year-old man with iron deficiency anemia presents to your clinic and denies any hematochezia or
melena. What is the first thing you would do?
Screen for colon cancer (iron deficiency anemia in the older population is cancer until proven otherwise.)

What are the symptoms for iron deficiency anemia?


Pallor, tachycardia, easy fatigability, Pica, esophageal webs

What is the triad for Plummer-Vinson?


1. Microcytosis
2. Atrophic glossitis
3. Esophageal webs

What are the laboratory findings seen in iron deficiency anemia?


↓ iron, ↓ ferritin, ↑ total iron-binding capacity (TIBC), ↓ reticulocyte count
(Think: Since there is less iron in the body, there is greater capacity for binding iron.)

How is iron deficiency anemia treated?


Ferrous sulfate

What is sideroblastic anemia?


Anemia caused by a disorder of the porphyrin pathway leading to ineffective erythropoiesis

What drugs commonly cause sideroblastic anemia?


Isoniazid; chloramphenicol, copper chelators, lead

What are some other causes of sideroblastic anemia?


Alcoholism, heredity

What are laboratory findings?


↑ Iron, ↑ ferritin, ↑ TIBC

How is it diagnosed?
Iron stain of bone marrow shows ringed sideroblasts with Prussian blue stain

What is the treatment?


Withdraw the offending agent, if one is identified, and give pyridoxine (B6)

What type of anemia is sickle cell anemia?


Microcytic

What kind of genetic inheritance pattern does sickle cell anemia exhibit?
It is an autosomal recessive disorder

What causes “sickling” of red blood cells (RBCs)?


Hemoglobin S tetramer polymerizes when RBCs are deoxygenated.

What are some signs and symptoms of sickle cell anemia?


Pain crisis caused by vaso-occlusion
Infarcts of the lungs, kidneys, bone, spleen
Intravascular hemolysis
Osteomyelitis caused by Salmonella
Aplastic anemia from parvovirus B19 infection
Myocardiopathy
“Fish mouth” vertebrae
Splenomegaly
Priapism
Stroke, transient ischemic attack (TIA)

What kind of infection are sickle cell patients with an autosplenectomy at risk for?
Infection with encapsulated bacteria which include pneumococcus, meningococcus, and Haemophilus
influenzae

What can intravascular hemolysis lead to in children?


“Pigment” gallstones

How is sickle cell anemia diagnosed?


Hemoglobin electrophoresis shows hemoglobin S.

How is sickle cell treated?


Remember the mnemonic HOPE:
Hydroxyurea—to prevent pain crises
Oxygen—to prevent sickling of cells
Pneumococcal vaccine
Exchange transfusion—when hydroxyurea ineffective

What are thalassemias?


Hereditary diseases in which there is a decreased production of globins causing a decrease in the production
of hemoglobin

What causes alpha-thalassemia?


A decrease in the alpha-globin chain production. There are four alpha alleles and anywhere from one to all
four of these alleles may be affected.

Match the alpha-thalassemia to the correct number of affected alleles and all the matching characteristics.
See Table 6-2.

Table 6-2 Alpha-Thalassemia


In what ethnicity is alpha-thalassemia most likely to be found?


More common in Asians. Also seen in people of Mediterranean and African descent.

What causes beta-thalassemia?


A decrease in the synthesis of one or both of the beta chains (there are two beta-chains in hemoglobin.)

In what ethnicities is beta-thalassemia most likely to be found?


African and Mediterranean descent

Match the description below to the correct beta-thalassemia:

How is beta-thalassemia definitively diagnosed?


Through gel electrophoresis. Thalassemia major will have increased levels of Hgb F as well as very
decreased Hgb A; while thalassemia minor will have normal levels of Hgb F with somewhat decreased Hgb
A.

MACROCYTIC ANEMIAS

What are five different etiologies of macrocytic anemia?


1. Folate deficiency
2. Vitamin B12 deficiency
3. Alcoholism
4. Liver disease
5. Hypothyroidism

Where is vitamin B12 absorbed?


In the terminal ileum

What factor is needed for vitamin B12 absorption?


Intrinsic factor

What are the signs and symptoms of vitamin B12 deficiency?


Neurologic symptoms such as ataxia, parasthesias, demyelination of corticospinal tract and dorsal columns.
Memory problems can also develop.

What is the most common cause of vitamin B12 deficiency?


Pernicious anemia

What is the underlying pathology in pernicious anemia?


In pernicious anemia, there is a decreased production of intrinsic factor because the gastric parietal cells are
destroyed by autoantibodies; there is atrophic gastritis.

How is pernicious anemia diagnosed?


↑ Methylmalonic acid
↑ Homocysteine levels
Atrophic gastritis on esophagogastroduodenoscopy (EGD)
Abnormal Schilling test (not used as much any more)

What are other causes of vitamin B12 deficiency?


Malabsorption because of resection of the terminal ileum or gastric resection, celiac sprue, Crohn disease,
infection with Diphyllobothrium latum or Giardia lamblia. Rarely, B12 deficiency is due to
hypoalimentation. This can be seen in strict vegetarians or alcoholics.

How is vitamin B12 deficiency treated?


Vitamin B12 supplementation

What foods contain folic acid?


Green leafy vegetables

Where is folate mainly absorbed?


Jejunum

What is the most common cause of folate deficiency?


Hypoalimentation

What are other causes of folate deficiency?


Pregnancy, tropical sprue, hemolytic anemia, long-term treatment with bactrim, methotrexate use, 5-
flourouricil use

What can folate deficiency in pregnancy cause?


Neural tube defects in the developing fetus

How can the diagnosis of folate deficiency be differentiated from that of B12 deficiency?
Normal methylmalonic acid
↑ Homocysteine levels
No neurologic symptoms

NORMOCYTIC ANEMIA

What are the most common causes of normocytic anemia?


Anemia of chronic disease, aplastic anemia, renal disease, hemolytic anemia, acute blood loss

How is anemia of chronic disease diagnosed?


↓ Iron; ↓ TIBC, normal ferritin

Why does renal failure cause anemia?


Erythropoietin is produced by the kidneys, and in chronic renal failure, erythropoietin levels are low.

What is the treatment of anemia in a patient with renal failure?


Erythropoietin supplementation

What is aplastic anemia?


Bone marrow failure leading to pancytopenia

Name six different etiologies of aplastic anemia.


1. Parvovirus B19 in the presence of sickle cell anemia
2. Hepatitis
3. Chloramphenicol
4. Benzene
5. Radiation therapy
6. Idiopathic

How is aplastic anemia diagnosed?


Normocytic, normochromic pancytopenia; hypocellular bone marrow in a bone marrow biopsy

What are the two main treatments for aplastic anemia?


1. Bone marrow transplant
2. Immunosuppression

What is the most common enzyme deficiency that causes hemolytic anemia?
G6PD deficiency

Why is G6PD important?


It is part of the hexose monophosphate pathway which reduces glutathione which is used to protect RBCs
against oxidative damage.

How is G6PD deficiency genetically transferred?


It is sex-linked.

In what ethnicities is G6PD deficiency most common?


Sephardic Jews, Mediterraneans, Middle Easterners, Africans, Asians
What are the signs and symptoms of G6PD deficiency?
Signs of hemolysis, which include dark urine, jaundice, weakness, pallor, abdominal and back pain caused
by mesenteric/renal ischemia, hepatosplenomegaly.

What can trigger an attack in a patient with G6PD deficiency?


Infection, fava beans, dapsone, sulfa drugs, primaquine, nonsteroidal anti-inflammatory drugs (NSAIDs)

What is the pathopneumonic G6PD deficiency diagnostic feature?


Peripheral smear shows Heinz bodies, schistocytes, and bite cells.

What are Heinz bodies?


Inclusions within red blood cells made of denatured hemoglobin

What is the treatment for anemia caused by G6PD deficiency?


It is usually self-limited. Remove inciting factors such as drugs. Transfuse only in very severe cases.

What two infections are associated with cold autoimmune hemolytic anemia?
Mycoplasma pneumonia and mononucleosis

Cold autoimmune hemolytic anemia is mediated by which immunoglobulin (lg)?


IgM

How is cold autoimmune hemolytic anemia diagnosed?


A positive cold agglutinin test or positive indirect Coombs test

How is cold autoimmune hemolytic anemia treated?


Staying warm as well as immunosuppresives

What general lab results would be seen in hemolytic anemia?


Unconjugated bilirubinemia, hemoglobinuria, elevated urine urobilinogen

COAGULOPATHIES

What does partial thromboplastin time (PTT) measure?


Intrinsic pathway

What does prothrombin time (PT) measure?


Extrinsic and common pathway

Which pathway does heparin affect?


Intrinsic pathway

Which pathway does warfarin affect?


Extrinsic pathway

What are some causes of PT elevation?


Warfarin treatment, vitamin K deficiency, liver disease

What are the many causes of thrombocytopenia?


Two categories:
1. ↑ Destruction/sequestration
Platelet disorders: TTP, ITP, DIC, HUS Splenomegaly
Drugs (heparin, aspirin, chemotherapy)
2. ↓ Production
Leukemia
Liver disease/alcohol
Aplastic anemia

At what platelet level does significant bleeding begin?


20,000

At what platelet level is a patient at risk for an intercranial bleed?


10,000

At what platelet level is there an increased risk for bleeding?


≤50,000

Name the platelet disorder associated with the following features:


Describe how each of the following platelet disorders can be diagnosed:

What is the treatment for each of the following platelet disorders?

What is the most common genetic coagulopathy?


von Willebrand factor deficiency

How is vWF deficiency inherited?


Autosomal dominant pattern

What are the signs and symptoms of vWF deficiency?


Easy bruisability as well as mucosal and gastrointestinal (GI) bleeding

How is vWF deficiency diagnosed?


Normal PT/PTT, ↑ bleeding time, ↓ factor VIII antigen, normal platelet count, ↓ ristocetin platelet study

What is the treatment for vWF deficiency


Desmopressin (DDAVP) in mild cases; severe cases need factor VIII concentrate, cryoprecipitate for
bleeding

Name the hemophilia described below:

What are the clinical signs and symptoms of the hemophilias?


Hemarthroses; bleeding with minimal trauma, multiple ecchymosis

How are the hemophilias diagnosed?


↑ PTT, normal PT, normal bleeding time, normal vWF; factor VIII deficiency in hemophilia A and factor
IX deficiency in hemophilia B

What is the treatment for each of the hemophilias?


Hemophilia A: factor VIII concentrate
Hemophilia B: factor IX concentrate

What treatment can be given to a patient with hemophilia A prior to a surgical procedure?
Desmopressin—It increases the production of endogenous factor VIII

LEUKEMIAS

What are the signs and symptoms of leukemia?


Pallor, fatigue, anemia, infection, petechiae

Name the type of leukemia described below:


What is the peak age of ALL?


Age 3-4 (most common cancer in children)

What are the subtypes of ALL?


L1, L2, L3

What subtype is most common in children?


80% are L1.

Of the adult cases of ALL, what subtype is most common?


L2

What is the L3 subtype morphologically identical?


Burkitt lymphoma

How is ALL diagnosed?


Peripheral blood smear with increased blast cells and TdT+, periodic acid-Schiff positive (PAS+),
CALLA+

What is the treatment plan for ALL?


Induction with chemotherapy (4-5 drugs)
Consolidation
Maintenance—radiation or low-dose chemotherapy
What is a poor prognostic factor in ALL?
Presence of Philadelphia chromosome

What is the treatment in patients who have the presence of the Philadelphia chromosome?
Bone marrow transplant

What is the prognosis of ALL in children?


80% remission

What is the prognosis in adults?


30% remission

What is the most common leukemia in adults?


AML

At what age does AML peak?


Age 15-39

What are the subtypes of AML?


M1-M7
M1-M3 granulocyte differentiation
M4-M5 monocytic precursors
M6 erythroblasts
M7 megakaryocytes

What hematologic disorder is the M3 subtype associated with?


DIC

How is AML diagnosed?


Peripheral blood smear with increased blast cells. Myeloblasts are myeloperoxidase+, Auer rod+, Sudan
black.

What is the treatment for AML?


Induction with daunorubicin and cytarabine; add all-trans retinoic acid for M3 subtype
Consolidation—continue chemotherapy Maintenance

What is the prognosis in adults with AML?


Those younger than age 60 have about a 70%-80% remission rate.

What age group does chronic lymphocytic leukemia (CLL) affect?


65 and older

Which blood cell type does CLL mainly affect?


B cells

How is CLL usually diagnosed?


Bone marrow infiltrated with lymphocytes, lymphocytes express CD5 protein, lymphocytosis on complete
blood count (CBC)

What is the progression of the disease?


Very slow progression

What is the treatment for CLL?


Supportive therapy because early therapy does not prolong life. Later there are the COP and CHOP
regimens. COP: cyclophosphamide, vincristine, prednisone/prednisolone; CHOP: COP plus doxorubicin.

What age group does CML most commonly affect?


40-60 years of age

What carcinogenic agent might CML be associated with?


Prior exposure to radiation

What are the unique signs and symptoms of CML?


Abdominal pain/fullness, anorexia, diaphoresis, bone pain

What chromosomal abnormality is CML associated with?


90% have the Philadelphia chromosome.

What is the Philadelphia chromosome?


Translocation of the ABL gene from chromosome 9 to BCR gene on chromosome 22

How is CML diagnosed?


90% have the Philadelphia chromosome; peripheral blood smear shows increased myeloblasts but less than
30%, basophils, and white blood cells. Low leukocyte alkaline phosphatase.

What are the different phases of CML?


1. Chronic phase: hepatosplenomegaly and increase in WBCs
2. Accelerated phase: platelet and RBC decrease while patient develops symptoms of night sweats, fever, bone
pain, and weight loss
3. Blastic phase: acute phase of the disease; blood and marrow are rapidly filled with proliferating blast cells

What is a blast crisis?


Acute phase of the disease in which the blood and marrow are rapidly filled with proliferating blast cells;
this takes about 3-4 years to develop and death is usually within 3-6 months.

What is the treatment for CML?


Bone marrow transplant is main treatment. Hydroxyurea and interferon alfa can reduce WBC count.
Chemotherapy is for patient who cannot have bone marrow transplant.

What is the prognosis after a bone marrow transplant in CML?


About 60% of patients go into remission.

What can CML progress to?


AML

Which type of leukemia has peripheral leukocytes with tartrate-resistant acid phosphatase and cytoplasmic
projections?
Hairy cell leukemia

What is the treatment for hairy cell leukemia?


Interferon alfa, splenectomy

LYMPHOMA


Name the type of lymphoma (Hodgkin lymphoma vs non-Hodgkin lymphoma [NHL]) described below:

What are the four subtypes of Hodgkin lymphoma?


1. Nodular sclerosing
2. Lymphocyte predominating
3. Mixed cellularity
4. Lymphocyte depleted

What is the most common type of Hodgkin lymphoma?


Nodular sclerosing

Which of the four subtypes of Hodgkin lymphoma has the worst prognosis?
Lymphocyte depleted

What clinical feature distinguishes Hodgkin lymphoma from NHL


Adenopathy is regional rather than systemic.

What are the symptoms of Hodgkin lymphoma and what are they called?
“B” symptoms—fever, night sweats, malaise, weight loss

How is Hodgkin lymphoma diagnosed?


Lymph node biopsy will show Reed-Sternberg cells.

What are the next steps to be taken after a biopsy determines a lymphoma is present?
Chest x-ray (CXR) to see extent of involvement as well as possible bone marrow biopsy and computed
tomographic (CT) scan

What is the staging of Hodgkin lymphoma?


Stage 1: one lymph node
Stage 2: two or more lymph nodes on the same side of the diaphragm
Stage 3: involvement on both sides of the diaphragm
Stage 4: dissemination to organs and tissues

What is the treatment for Hodgkin lymphoma?


Radiation therapy for localized disease (stages 1 and 2) and chemotherapy for more extensive disease (stages
3 and 4)

What chemotherapy regimens are most commonly used?


ABVD: adriamycin, bleomycin, vincristine, dacarbazine
MOPP: meclorethamine, oncovin, procarbazine, prednisone

How are the different types of NHL characterized?


Low, intermediate, and high grade

Name the most common subtypes of NHL?


Low grade: follicular small cleaved cell
Intermediate grade: diffuse large-cell
lymphoma
High grade: lymphoblastic lymphoma
Burkitt lymphoma: American type and African type

Name the subtype of NHL described below:


How is NHL diagnosed?


Biopsy of lymph node

What are the next diagnostic studies to consider after the biopsy?
CXR, CT scan, bone marrow biopsy to determine the extent of the disease

What is the prognostic factor in NHL?


Histologic subtype is a more prognostic factor than the extent of spread of disease.

How is the adenopathy in NHL described?


Painless adenopathy

What is the treatment for NHL?


Radiation and chemotherapy depending on subtype

MYELOPROLIFERATIVE DISEASES

What are myeloproliferative diseases?


A number of diseases in which there is excessive production of differentiated myeloid cell lines

What can the myeloproliferative diseases transform into?


Acute leukemias

What is polycythemia vera?


A myeloproliferative disorder in which there is excess production of ALL blood cell lines but especially red
blood cells

What are the different etiologies of polycythemia vera?


It can be a primary disorder which is idiopathic in nature or it can be secondary to hypoxia, dehydration, low
erythropoietin production, and smoking.
What is the peak of onset of polycythemia vera?
Age >60

In what sex is polycythemia vera most commonly seen?


Males

What are the signs and symptoms of polycythemia vera?


Pruritis after showering due to basophilia, epistaxis, plethora, blurred vision, splenomegaly, gout, basophilia,
headache, retinal hemorrhages, cerebrovascular accidents (CVA), gastric ulcers

How is polycythemia vera diagnosed?


On CBC excess of all blood cell lines especially red blood cells. Patient may have low erythropoietin and
low erythrocyte sedimentation rate (ESR).

What is the treatment for polycythemia vera?


Serial phlebotomy to decrease the volume of blood; hydroxyurea to suppress excess blood cell production;
aspirin to thin the blood

What is a possible long-term complication that occurs in about 20% of patients with polycythemia vera?
Fibrosis of the bone marrow

What is essential thrombocytosis?


Disease in which there is an idiopathic increase of platelets to >5 × 105 cells/μL

What are the clinical signs and symptoms of essential thrombocytosis?


Burning and throbbing hands and feet as well as splenomegaly; bleeding from nose and gums due to platelet
dysfunction

What are the main treatments for essential thrombocytosis?


Platelet exchange, hydroxyurea, anagrelide

What is idiopathic myelofibrosis?


Disorder in which there is extensive extramedullary hematopoiesis causing replacement of marrow with
fibrous connective tissue

What is the pathopneumonic sign of myelofibrosis?


Peripheral smear shows tear drop cells.

What is the treatment for myelofibrosis?


The prognosis is poor and the treatment is mainly supportive.

What is multiple myeloma?


Malignant disease of plasma cells which produce monoclonal immunoglobulins or light chains

What is the ratio of white to African Americans who have multiple myeloma?
1:2

What can be seen on an x-ray of a patient with multiple myeloma?


Lytic lesions (“punched out” areas of bone)

What are the signs and symptoms of multiple myeloma?


Bone pain, pathologic fractures due to lytic lesions; anemia, hypercalcemia, renal failure
What is the triad that is often seen in multiple myeloma?
1. Anemia
2. Back pain
3. Renal failure

How can multiple myeloma be diagnosed?


24-hours urine collection followed by urine protein electrophoresis (UPEP) and serum protein
electrophoresis (SPEP). These studies will demonstrate free kappa and lambda light chains known as Bence
Jones proteins, and monoclonal elevation of one cell line. There will be an “M-spike” (or a peak) in the
SPEP if there is whole antibody-made. There will be an “M-spike” in the UPEP if light chains only are
made. To make the diagnosis there should be a spike in the SPEP or UPEP as well as one of the following:
lytic lesions, Bence Jones proteinuria, or increased plasma cells in the bone marrow.

What is the treatment for multiple myeloma?


Chemotherapy in addition to bisphosphonates and allopurinol as needed for hypercalcemia and elevated uric
acid, respectively. Bone Marrow stem cell transplantation is available to some patients. Patients with spinal
cord compression are given corticosteroids and/or radiation. Radiation can also be used to treat lesions that
are symptomatic.

What is the most common type of plasma cell dyscrasia?


Monoclonal gammopathy of undetermined significance (MGUS)

What is MGUS?
Presence of monoclonal immunoglobulin or M-protein in serum or urine without evidence of any other
lymphoproliferative disorder

What are the characteristics that distinguish MGUS from other lymphoproliferative diseases?
Serum M-protein <3 g/dL; no lytic bone lesions, very little or no Bence Jones proteins in urine; bone marrow
contains <10% plasma cells, no signs of end-organ damage; patients are asymptomatic

What condition can MGUS patients suddenly develop?


Multiple myeloma

What is the treatment for MGUS?


No treatment is necessary but patients should be followed to make sure they do not develop other
lymphoproliferative disorders since these patients are at higher risk.

CLINICAL VIGNETTES

A 30-year-old male who is known to have atrophic gastritis is found to have anemia on a CBC. His MCV is
105. He also has an elevated methylmalonic acid level and elevated homocysteine level. His vitamin B12
level is low. What is the most likely cause of his anemia?
Pernicious anemia

An asymptomatic 50-year-old female is found to have serum M-protein of 4 g/dL. On further examination
she is found to have neither Bence Jones proteins on urinalysis nor any evidence of lytic bone lesions on
radiographic imaging. Her renal function and liver function tests are within normal limits. What is the
most likely diagnosis?
MGUS (monoclonal gammopathy of undetermined significance)
A 20-year-old female presents with a rash on her skin and gingival mucosa. On examination you find that
she has petechiae. You also find that she has bruising in many parts of her body. On review of systems she
states that she has felt very tired recently. A CBC demonstrated an elevated white blood cell count. A
peripheral blood smear was done and showed blast cells as well as Auer rods. What is the diagnosis?
Acute myelogenous leukemia

A 28-year-old female with a past history of HIV is brought in by her brother. He states that she has been
acting strangely recently. He states that she seems “confused” recently. He noted a fever as well as
petechiae and purpura on her chest. On examination you find hemoglobin of 7 and platelet count of 50,000.
Her creatinine is also significantly elevated. What diagnosis do you suspect?
TTP

A 63-year-old male who seems to have a “reddish” complexion presents to your office with complaints of
headache and blurred vision. His past medical history is significant for diabetes, stroke, and gastric ulcer.
On examination you find him to have a blood pressure of 160/95 and splenomegaly. A CBC demonstrates
thrombocytosis. What is this patient’s condition called?
Polycythemia vera
CHAPTER 7
Rheumatology

ARTHROPATHIES

What is rheumatoid arthritis (RA)?


An autoimmune symmetric inflammatory arthritis

What HLA type is RA associated with?


HLA-DR4

In what sex is RA more common?


Females

What classical physical examination findings can be found in RA?


Boutonniere deformity; swan neck deformity; ulnar deviation; pain in the proximal interphalangeal (PIP)
and metacarpophalangeal (MCP) joints; rheumatoid nodules

What are the seven diagnostic criteria for RA?


1. Morning stiffness >1 hour
2. Three or more joints with arthritis
3. One hand joint with arthritis
4. Symmetric arthritis
5. Rheumatoid nodules
6. Elevated serum rheumatoid factor (RF)
7. Radiographic evidence of erosive arthritis

How many of the criteria must be positive for a diagnosis of RA?


Four

How long does each of the criteria need to be present to make a diagnosis?
At least 6 weeks

What is a boutonniere deformity?


Hyperextension of distal interphalangeal (DIP) and flexion of PIP joints (Fig 7-1)

Figure 7-1 Boutonniere deformity. (Reproduced, with permission, from Wilson FC, Lin PP. General
Orthopedics. New York: McGraw-Hill; 1997:413.)

What is a swan neck deformity?


Flexion of DIP and extension of PIP joints (Fig 7-2)

Figure 7-2 Swan neck deformity. (Reproduced, with permission, from Knoop KJ, Stack LB, Storrow AB.
Atlas of Emergency Medicine. New York: McGraw-Hill; 1997:291.)

What laboratory findings could you expect in a patient with RA?


↑ RF, ↑ erythrocyte sedimentation rate (ESR)

What is the treatment for pain associated with RA?


First-line: nonsteroidal anti-inflammatory drugs (NSAIDs) to decrease inflammation Second-line:
corticosteroids

What disease-modifying agents are available for patients with RA?


Methotrexate, hydroxychloroquine, gold compounds

What are some newer biologic agents used to treat RA?


Infliximab, etanercept, abatacept, Rituxan
What is the most common type of arthritis?
Osteoarthritis (OA)

What is the main underlying cause of OA?


Degenerative changes of the joints

What are the two classic physical examination findings in OA?


1. Heberden nodules which affect the DIP joints
2. Bouchard nodes which affect the PIP joints (Fig 7-3)

Figure 7-3 Bouchard nodes. (Reproduced, with permission, from Knoop KJ, Stack LB, Storrow AB. Atlas of
Emergency Medicine. New York: McGraw-Hill; 1997:291.)

How do the symptoms of OA differ from RA?


Morning stiffness resolves within 30 minutes; outer joints of the hand are mainly affected (DIP joints in
addition to MCP and PIP joints).

What are the x-ray findings seen in OA?


Narrowed joint spaces, osteophyte formation

What is the treatment for OA?


NSAIDs to relieve pain; muscle-strengthening exercises; steroid joint injection; last resort is joint
replacement

What is gout?
Arthropathy caused by urate crystal deposit in a single joint

What are the etiologies of gout?


Decreased uric acid excretion (high-protein diet, alcohol use, diuretic use) or increased uric acid production
(genetic diseases, hemolysis, cancer)

What are the signs and symptoms of gout?


Acute pain accompanied by redness and swelling of a joint

What is the most common joint to be affected?


First metatarsophalangeal joint

What is podagra?
Inflammation of the first metatarsophalangeal joint of the foot which is of sudden onset

What are tophi?


Aggregates of gouty crystal and giant cells secondary to chronic gout

What is the classic radiographic finding in advanced gout?


“Rat-bite” appearance

How is gout diagnosed?


Fluid aspirated from the joint would reveal needle-shaped monosodium urate crystals with negative
birefringence under polarized light

How is acute gout treated?


Colchicine and NSAIDs for pain

What is used for maintenance therapy of gout?


Allopurinol to prevent production; probenecid to increase excretion; low-protein diet; refrain from alcohol

What is pseudogout?
Deposition of calcium pyrophosphate crystals in joints, causing inflammation

What are the risk factors for pseudogout?


Advanced age; gout, hemochromatosis, extensive osteoarthritis, diabetes, hyperparathyroidism,
hypothyroidism, hypomagnesemia, neuropathic joint

What are the signs and symptoms of pseudogout?


May present as an acute arthritis affecting one or multiple joints like gout, or it may present as a chronic
polyarthritis similar to OA or RA

What does joint fluid aspiration in pseudogout demonstrate?


Positively birefringent rhomboid crystals

What is the treatment for pseudogout?


NSAIDs

Name the autoimmune disorder which is characterized by sacroiliitis, with fusion of adjacent vertebral
bodies.
Ankylosing spondylitis

What HLA type is ankylosing spondylitis associated with?


HLA-B27

What joint is always affected in ankylosing spondylitis?


Sacroiliac joint

What are the typical symptoms?


Chronic lower back pain in young men lasting more than 1 hour and relieved with rest

What is the classic x-ray finding seen with ankylosing spondylitis?


Bamboo spine (Fig 7-4)

Figure 7-4 Bamboo spine. (Reproduced, with permission, from Wilson FC, Lin PP. General Orthopedics.
New York: McGraw-Hill; 1997:454.)

What other disorder is ankylosing spondylitis associated with?


Ulcerative colitis

SYSTEMIC DISORDERS

What are the signs and symptoms of systemic lupus erythematosus (SLE)?
Fatigue, malaise, malar rash, arthralgias, pericarditis, endocarditis, neurologic symptoms, polyarthritis

What is the sex distribution of SLE?


90% female predominance

How is SLE distributed based on race?


Black > white

What is the mnemonic for diagnosing SLE?


DOPAMINE RASH:
Discoid rash: raised, erythematous circular rash with scale
Oral ulcers
Photosensitivity
Arthritis > 2 joints
Malar rash: butterfly rash on cheeks
Immunologic criteria: anti-Sm Ab, anti–double-stranded DNA, false-positive venereal disease research
laboratory (VDRL) test
Neurologic symptoms: seizures, psychosis
ESR elevated (not part of the 11 criteria)
Renal disease
Antinuclear antibody (ANA) positive
Serositis : pericarditis, pleurisy
Hematologic disorder: hemolytic anemia, leukopenia, thrombocytopenia, lymphopenia

How many of the criteria must be present to make the diagnosis of SLE?
Four or more

What is the pathognomonic heart disorder seen in SLE patients?


Libman-Sacks endocarditis (LSE)

What autoantibody is most sensitive for SLE?


ANA (it is not specific)

Which autoantibody is most specific for SLE?


Anti–double-stranded-DNA (very high titers are associated with renal involvement), anti-SM antibody

What other autoantibodies are associated with SLE?


Anti-La antibody
Anti-Ro antibody

What are lupus anticoagulant and anticardiolipin associated with?


Thrombosis, central nervous system (CNS) lupus, thrombocytopenia, valvular heart disease, fetal loss

What serologies can be falsely positive in patients with SLE?


Rapid plasma reagin (RPR)/VDRL

Anticardiolipin can cause a falsely elevated result with which lab test?
Elevated partial thromboplastin time (PTT), but in reality SLE patients are more likely to develop blood
clots

What are the treatments for SLE?


Avoid sun exposure, NSAIDs for joint pain, systemic steroids, immunosuppressives such as
cyclophosphamide in refractory cases with more advanced development of disease

How is drug-induced lupus different from SLE?


Symptoms resolve with discontinuation of the drug and anti-histone antibody positive

What drugs are known to cause drug-induced SLE?


SIQ CHaMP:
Sulfasalazine
Isoniazid (INH)
Quinidine
Chlorpromazine
Hydralazine
a
Methyldopa, minocycline
Procainamide, penicillamine

What is the most common drug to cause lupus-like symptoms?


Procainamide

What autoimmune disorder is characterized by systemic fibrosis secondary to excess collagen and
extracellular matrix production?
Scleroderma

What are the signs and symptoms of scleroderma?


Tight, thick skin; Raynaud phenomenon; dysphagia; renal artery fibrosis; pulmonary hypertension secondary
to fibrosis; telangiectasias

What is a more limited form of scleroderma called?


CREST syndrome

What does CREST stand for?


Calcinosis
Raynaud phenomenon
Esophageal dysmotility
Sclerodactyly
Telangiectasias

What laboratory test is 80% sensitive for CREST syndrome?


Anticentromere antibody

What laboratory test is highly specific to scleroderma?


Anti–Scl-70 antibody

What is the treatment for scleroderma?


Remember the mnemonic CAPS:
Calcium channel blocker
Ace inhibitor (captopril)
Penincillamine
Steroids

What systemic disease is characterized by noncaseating granulomas in the lung?


Sarcoidosis

What race is more predisposed to arcoidosis?


African Americans

What are some findings associated with sarcoidosis?


Remember the mnemonic GRUELING:
Granulomas
RA
Uveitis
Erythema nodosum
Lymphadenopathy
Interstitial fibrosis
Negative TB test
Gamma-globulinemia
What renal problem is associated with sarcoidosis?
Nephrolithiasis because of hypercalciuria

What is the most important component of diagnosing sarcoidosis?


Transbronchial biopsy showing noncaseating granuloma

What is seen on a chest x-ray (CXR) of a patient with sarcoidosis?


Bilateral hilar adenopathy with perihilar calcifications

What classic laboratory findings are seen in sarcoidosis?


Hypercalcemia and ↑ angiotensin-converting enzyme (ACE)

What is the main treatment for sarcoidosis?


Symptomatic treatment and corticosteroids

What autoimmune disorder is associated with the following triad: keratoconjunctivitis sicca, xerostomia,
and arthritis?
Sjögren syndrome

What HLA type is Sjögren syndrome associated with?


HLA-DR3

What type of cancer are patients with Sjögren syndrome at high risk for?
Non-Hodgkin lymphoma

What autoantibodies is Sjögren syndrome associated with?


Anti–single-stranded (SS)-A (Ro) and anti-SS-B (La)

What is the treatment for Sjögren syndrome?


Corticosteroids

Name the syndrome associated with the following: conjunctivitis, uveitis, urethritis, and asymmetric
arthritis.
Reiter syndrome

What is the mnemonic used to remember the associated findings of Reiter syndrome?
“Can’t see. Can’t Pee. Can’t climb a tree.”
Can’t see: conjunctivitis, uveitis
Can’t pee: urethritis
Can’t climb a tree : arthritis

What HLA type is Reiter syndrome associated with?


HLA-B27

What are the two forms of Reiter syndrome?


1. Sexually transmitted
2. Postinfectious: Campylobacter, Yersinia, Salmonella, Shigella

What will a urethral culture often grow out in a patient with Reiter syndrome?
Chlamydia trachomatis

What is the treatment for Reiter syndrome?


Doxycycline to cover for Chlamydia and NSAIDs for pain
What is the autoimmune syndrome associated with the following: aphthous ulcers, genital ulcers, arthritis,
uveitis, psychiatric symptoms
Behçet syndrome

MUSCLE DISORDERS

What is polymyositis?
Autoimmune disease which causes proximal muscle weakness

How is polymyositis different from dermatomyositis?


Dermatomyositis includes rash as a symptom, whereas with polymyositis there is no rash.

What sex is more likely to have polymyositis?


Females are twice as likely

What are the signs and symptoms of polymyositis?


Symmetric proximal muscle weakness, dysphonia, and dysphagia; patients have difficulty standing up
from a chair or brushing their hair

What are the classic signs of dermatomyositis?


Symmetric proximal muscle weakness, heliotropic periorbital rash, shawl sign (erythematous macules on
shoulders and upper back), Gottron papules (violacious papules on DIP joints)

What autoantibody is associated with polymyositis and dermatomyositis?


Anti–Jo-1

What are the four criteria for polymyositis?


1. ↑ Creatine phosphokinase (CPK)
2. Proximal muscle weakness
3. Low-amplitude potentials and fibrillations on electromyogram (EMG)
4. ↑ Muscle fiber size on muscle biopsy

What is the treatment for polymyositis and dermatomyositis?


Corticosteroids and methotrexate or cyclophosphamide in refractory cases

What is myasthenia gravis?


Autoimmune disease in which autoantibodies block the postsynaptic acetylcholine receptors preventing
acetylcholine from binding leading to muscle weakness

What are the two peak incidences of myasthenia gravis?


Women: second to third decades of life Men: fifth to sixth decades of life

What can myasthenia gravis be associated with?


Thymomas or other autoimmune diseases

What are the signs and symptoms of myasthenia gravis?


Muscle weakness and increasing fatigue with use, proximal muscle weakness, ptosis, diplopia, dysphagia

What is the “classic” test used to diagnose myasthenia gravis?


Edrophonium test (Tensilon test)
How does the test work?
Edrophonium inhibits acetylcholinesterase allowing for higher levels of acetylcholine to be available to
stimulate receptors and, therefore, if the patient has myasthenia gravis, edrophonium administration will lead
to improved muscle strength.

What are the newer diagnostic methods for myasthenia gravis?


Single-fiber EMG; anti-acetylcholine receptor antibody test

Myasthenia gravis is often associated with what other finding?


Thymoma

What blood test in the presence of myasthenia gravis is highly associated with the presence of a thymoma?
Anti-striated muscle (SM) antibody—present in >80% of patients younger than 40 with thymoma

What is the treatment for myasthenia gravis?


Pyridostigmine and acetylcholinesterase inhibitor as well as steroids

What is the pathology in Lambert-Eaton syndrome?


There are autoantibodies to presynaptic calcium channels.

How does Lambert-Eaton syndrome differ from myasthenia gravis?


Increased muscle use improves symptoms making muscles stronger.

VASCULITIS

Name the vasculitis associated with the following:


CLINICAL VIGNETTES

A 58-year-old male states that the previous evening he had sudden pain and swelling in the great toe of his
right foot. Joint fluid aspiration demonstrates negatively birefringent needle-shaped crystals. What is the
treatment for an acute attack?
NSAIDs (Colchicine classically)

A 22-year-old male presents with a history of low back pain for the past 6 months. He does not recall any
trauma to that area. Rest does not seem to relieve the pain but exercise seems to be helpful. He has
decreased range of motion in the lumbar spine. An x-ray demonstrates a “bamboo spine.” What is the
diagnosis?
Ankylosing spondylitis
A 35-year-old female has photosensitivity, rash on her cheeks, arthritis, and oral ulcers. She also has a
positive ANA. You suspect SLE. What serology may be falsely positive in this patient?
RPR/VDRL

A 48-year-old female presents with difficulty swallowing. You notice that her skin appears very shiny,
thick, and tight. On review of systems, she mentions that the tips of her fingers become blue and painful in
the cold. What laboratory test is specific for the diagnosis of her condition?
Anti–scl-70 antibody to test for scleroderma

An 83-year-old female complains of new-onset headaches. They are unilateral and often unrelieved with
NSAIDs. She points to her right temple when asked to describe where the pain is. Her laboratory
evaluation demonstrates a significantly elevated ESR. What test would you suggest next to make a
diagnosis?
Temporal artery biopsy to evaluate for temporal arteritis
CHAPTER 8
Nephrology

ACUTE RENAL FAILURE

What is azotemia?
A high level of urea or other nitrogen-containing compounds in the blood usually secondary to renal failure

What is acute renal failure (ARF)?


Newly increased azotemia with an increase in blood urea nitrogen (BUN) and creatinine

What are the three categories of acute renal failure?


Prerenal
Renal
Postrenal

What causes prerenal ARF?


Low perfusion

What are examples of prerenal causes of ARF?


Congestive heart failure (CHF)
Volume loss
Hypotension
Sepsis
Burns
Low blood flow to the kidneys (renal artery stenosis [RAS])

What is the underlying cause of intrinsic ARF?


Injury to the nephron due to ischemia or toxins

What is the most common cause of intrinsic renal failure?


Acute tubular necrosis (ATN)

What are some other causes of intrinsic ARF?


Acute interstitial nephritis (AIN)
Glomerulonephritis (GN)
Ischemia
Vasculitis

What are some causes of postrenal acute renal failure?


Obstruction caused by:
Kidney stones
Enlarged prostate (BPH)
Tumors such as bladder cancer (CA), cervical CA, prostate CA

What are some signs and symptoms of ARF secondary to uremia?


Asterixis, nausea, vomiting, anemia, pericarditis, pruritis, urea crystals on the skin (“uremic frost”), fatigue,
oliguria

What are some signs and symptoms of ARF not secondary to uremia?
Metabolic acidosis
Hyperkalemia → arrhythmias
Fluid overload → pulmonary edema, CHF, hypertension
Hyperphosphatemia
Hypertension 2° excess renin secretion

What defines oliguria?


Urine output of <400 cc/24 h

What tests would you initially order to evaluate for ARF?


Urine/serum electrolytes; urine/serum BUN/creatinine (Cr); urinalysis including urine osmolality

What is FENa?
FENa stands for fractional sodium excretion and is the best diagnostic test to help discriminate between the
different types of ARF.

How is FENa calculated?

How do you distinguish between prerenal, renal, and postrenal ARF?


See Table 8-1.

Table 8-1 ARF: Laboratory Differences Between Prerenal, Renal, and Postrenal Etiologies

Name the type of ARF associated with the following urinary sediment findings:

What are the causes of ATN?


There are two categories:
1. Ischemic: shock, trauma, hypoxia, hemorrhage, sepsis
2. Toxic: medications, rhabdomyolysis (which causes myoglobinuria), IV contrast

What medications classically cause ATN?


Remember the mnemonic CLAAP:
Contrast
Lithium
Aminoglycosides
Amphotericin
Pentamine

How is ATN treated?


Remove insulting agent, IV fluids to maintain urine output, IV diuretic therapy to increase urinary output
and prevent overload, protein-restricted diet, close monitoring of electrolytes; dialysis if needed

What are the causes of AIN?


Inflammation of the renal parenchyma caused by:
1. Medications: diuretics, nonsteroidal anti-inflammatory drugs (NSAIDs), penicillin
2. Infection: cytomegalovirus (CMV), Epstein-Barr virus (EBV), toxoplasmosis, syphilis
3. Systemic diseases: Sjögren syndrome, sarcoidosis

Name the cause of ARF classically indicated by the following serologic tests:

How is AIN treated?


Treatment is the same as ATN.

CHRONIC RENAL FAILURE

What is chronic renal failure (CRF)?


Progressive loss of nephrons

What is the most common cause of CRF?


Diabetes

What is uremia?
Clinical manifestations of elevated levels of urea in the blood usually secondary to renal failure

What characterizes uremic syndrome?


Uremic syndrome is chronic renal failure that has effects on multiple organs and systems.
Cardiovascular: hypertension, pericarditis
Pulmonary: pleural effusions, pulmonary edema
Central nervous system (CNS): asterixis, clonus
Hematology: anemia because of low erythropoietin; increased bleeding time due to platelet dysfunction
Gastrointestinal (GI): nausea; vomiting
Metabolic: acidosis, electrolyte imbalances (especially hyperkalemia), hypocalcemia (lack of vitamin D),
azotemia

What can be used to measure the severity of CRF?


Glomerular filtration rate (GFR); the lower the GFR the worse the renal function

How is GFR estimated?


Creatinine clearance is approximately equal to GFR.

How is creatinine clearance calculated?


Urine creatinine × urine volume in millimeter/serum creatinine × time in minutes

Estimated creatinine clearance =

In CRF, there is decreased synthesis of what two entities?


1. Vitamin D
2. Erythropoietin

What electrolyte abnormalities are seen in CRF?


Hyperkalemia
Hypocalcemia
Hyperphosphatemia

What is the treatment for chronic renal failure?


Dialysis: either hemodialysis or peritoneal dialysis

How should medications prescribed to patients in CRF be adjusted?


They should be renally dosed.

What are the indications for dialysis?


Remember the mnemonic AEIOU:
Acidosis
Electrolyte abnormalities
Ingestion of toxins
Overload of fluid
Uremic symptoms

How is vascular access achieved in a hemodialysis patient?


There are three possible types of access:
1. Arteriovenous (AV) fistula: A shunt between an artery and a vein usually placed in the forearm. Healing may
take up to 4 months before it can be used, but once it is healed it may be used for several years.
2. Arteriovenous graft: Often used in patients with small veins that won’t properly turn into a fistula. It is a shunt
between an artery and a vein using plastic tubing.
3. Catheter: Not permanent. Used until the AV fistula or graft is healed and ready for use.

How is peritoneal dialysis achieved?


Patient gets a permanent catheter in the peritoneum and the peritoneum is used as a membrane through
which dialysis is achieved. Dialysis fluid is infused rapidly, then allowed to stay in the peritoneal cavity for
several hours, then drained and new fluid infused.
What kind of infection are patients on peritoneal dialysis classically at risk for?
Bacterial peritonitis

How is bacterial peritonitis treated?


Intraperitoneal vancomycin or antibiotics based on culture sensitivity

GLOMERULONEPHROPATHIES

What is nephrotic syndrome?


Nephrotic syndrome is glomerular damage leading to proteinuria (>3.5 g/day)

What are other defining features of nephrotic syndrome?


Hypoalbuminemia, generalized edema, hyperlipidemia, hypercoagulable state (because of loss of protein C
and S), immunocompromised state

What is nephritic syndrome?


Glomerular disease leading to syndrome of hematuria, edema, and often hypertension (HTN)

How can urinary cholesterol be identified?


If urine is seen under polarized light, there will be “maltese crosses.”

What are some causes of nephrotic syndrome?


Minimal change disease (MCD)
Focal segmental glomerulosclerosis
Membranous glomerulonephritis
Membranoproliferative glomerulonephritis

What are the other names for minimal change disease?


Nil disease, lipoid nephrosis

Name the nephrotic syndrome associated with each of the following:


What is the main treatment for each of the following?


What is nephritic syndrome?


Glomerulonephropathy also known as glomerulonephritis in which there is acute-onset hematuria, azotemia,
hypertension, edema, and mild proteinuria

What is classically seen on microscopy in nephritic syndrome?


Red blood cell (RBC) casts

Name the nephritic syndrome associated with the following:


What is the most common glomerulonephropathy?


Buerger disease

What is Goodpasture disease?


Glomerulonephritis with pneumonitis

When is the peak incidence of Goodpasture disease?


Males in the second decade of life

What is the most common presenting symptom of Goodpasture disease?


Hemoptysis

URINARY TRACT

What is nephrolithiasis?
Kidney stones

What are the classic signs and symptoms of nephrolithiasis?


Back pain or flank pain that radiates to groin, nausea, vomiting, microscopic vs gross hematuria

What is the most common type of kidney stone?


Calcium pyrophosphate

What is the underlying etiology?


Hypercalciuria
What is the treatment for calcium pyrophosphate stones?
Hydration and thiazide diuretics; lithotripsy if stone is too large to pass

What is the second most common type of kidney stone?


Ammonium magnesium phosphate

What is another name for ammonium magnesium phosphate stones?


Struvite stones

What are the underlying bacterial etiologies of ammonium magnesium phosphate stones?
Proteus, Pseudomonas, Providencia, or Staphylococcus saprophyticus

How are struvite stones treated?


Treat the underlying infection and lower the urinary pH

Which type of stone is radiolucent?


Uric acid stones

What disorders are often an underlying cause of uric acid stones?


Gout or myeloproliferative disease

How are uric acid stones treated?


Raise urinary pH

Which type of stone is radiopaque?


Calcium pyrophosphate and ammonium magnesium phosphate

How is nephrolithiasis diagnosed?


Plain films can identify radiopaque stones. Renal ultrasound (US) can visualize hydronephrosis; IV
pyelogram is another option. Noncontrast helical computed tomography (CT) scan can visualize small stones
and is the gold standard for diagnosis.

What is the most common pathogen in urinary tract infections (UTIs)?


Escherichia coli

What is the mnemonic for common pathogens causing UTIs?


KEEPS:
Klebsiella
E coli
Enterobacter
Proteus
S saprophyticus

What are the signs and symptoms of UTI?


Urinary urgency, frequency; burning with urination; hematuria; sense of incomplete bladder emptying

How is a UTI diagnosed?


Urinalysis can demonstrate a high number of WBCs, positive leukocyte esterase, positive nitrites, and
moderate to large number of bacteria.

What is the indication of a contaminated urinalysis?


Many epithelial cells or many types of bacteria present
Other than urinalysis, what test should be ordered in a patient suspected to have a UTI?
Urine culture, Gram stain, and sensitivity

What is the first-line treatment for UTI?


3-day course of trimethoprim-sulfamethoxazole (TMP-SMX); however, in areas of high resistance to
TMP-SMX, flouroquinalones, typically ciprofloxacin, have become first line

In what type of patient should flouroquinalones be avoided?


Pregnant patients

What would you suspect in a patient with urinary frequency, burning on urination, costovertebral angle
tenderness as well as fever and chills?
Pyelonephritis

What is the treatment for pyelonephritis?


po or IV antibiotics

ACID-BASE DISORDERS

What are the normal lab values for each of the following components of an arterial blood gas (ABG)?

How is anion gap calculated and what is a normal range?


Na − (Cl + HCO3). Normal range is 9-14.

What is the definition of metabolic acidosis?


↓ pH with ↓ HCO3

What is Winter’s formula?


It determines if there was appropriate compensation in the setting of metabolic acidosis: 1.5 × (HCO3−) + 8
± 2 = Pco2.

What are the causes of anion gap metabolic acidosis?


Remember the mnemonic MUD PILES:
Methanol, Metformin
Uremia
DKA (diabetic ketoacidosis)
Paraldehyde
INH (isoniazid), iron tablets
Lactic acidosis
Ethanol
Salicylates

How is the etiology of the metabolic acidosis determined?


Check for ketonuria.

Which of the etiologies are present with and without ketonuria?


See Table 8-2.

Table 8-2 Anion Gap Metabolic Acidosis Etiologies

What are the causes of normal anion gap metabolic acidosis?


Renal tubular acidosis, diarrhea, colostomy, ileostomy, ingestion of magnesium sulfate, calcium chloride,
acetazolamide, hyperparathyroidism

What is the treatment for metabolic acidosis?


Correct the underlying cause.

What is the definition of respiratory acidosis?


Hypoventilation causing Paco2 and ↓ pH

What is the treatment for respiratory acidosis?


Treat the underlying cause and mechanical hyperventilation can help to release some CO2.

What is the definition of metabolic alkalosis?


↑ pH, ↑ plasma bicarbonate, and compensatory ↑ PaCO2

What are the underlying causes of metabolic acidosis?


Vomiting, diarrhea, nasogastric (NG) tube suction for prolonged period, diuretic use, hypomagnesemia,
hypokalemia, licorice, tobacco use, Cushing syndrome, RAS

What is the treatment for metabolic acidosis?


Treat the underlying cause. These patients are usually volume-depleted so rehydration is needed. Replete
potassium and magnesium as needed.

What is the definition of respiratory alkalosis?


Hyperventilation causing ↑ arterial pH, ↓ Pco2, ↓ serum bicarbonate

What is the treatment of respiratory alkalosis?


Decrease the rate of breathing.
RENAL ARTERY STENOSIS

What are the classic findings in renal artery stenosis (RAS)?


Hypertension that is poorly controlled despite multiple medications, often with hypokalemia

What are the underlying causes of RAS?


Atherosclerosis or fibromuscular dysplasia

What is the more common cause of RAS in females?


Fibromuscular dysplasia

What is in the differential diagnosis when a patient has the classic finding of hypertension with
hypokalemia?
Conn hyperaldosteronism vs secondary hyperaldosteronism due to renal artery stenosis

How is RAS diagnosed?


Imaging via renal arteriogram, magnetic resonance angiography, or Doppler ultrasonography

How is RAS treated?


Angioplasty and in some cases surgery

CLINICAL VIGNETTES

Your patient is hospitalized for abdominal pain. During the workup a CT of the abdomen and pelvis is
done with contrast. His initial labs showed a slightly elevated WBC count, but otherwise his electrolytes,
BUN, creatinine, glucose, AST, ALT, amylase, and lipase were all within normal limits. The following
morning, you check the labs and find that the creatinine has suddenly risen dramatically. You check a
urinalysis and find that there are granular casts. What class of acute renal failure do you suspect?
Acute tubular necrosis

A 48-year-old male with a past medical history of hypertension and hyperlipidemia rushes to your office.
He just had hematuria and he is very concerned. He has also had a very bad sore throat in the last few
days. An ASO titer is elevated. What is the most likely reason for this person’s hematuria?
Poststreptococcal glomerulonephritis

A 31-year-old female patient comes for follow-up on her hypertension. Despite three different medications,
her blood pressure is 148/92. She states that she is very frustrated. She has been trying so hard to follow
her low sodium diet, she has been exercising and taking her medications religiously but despite all that her
blood pressure is still high. She is also hypokalemic. You suspect renal artery stenosis. What test could be
used to definitively diagnose this condition?
Renal angiography

In the patient described in the previous vignette, you find through testing that she does indeed have renal
artery stenosis. What is the most likely underlying cause in this particular patient?
Fibromuscular dysplasia

Your patient develops acute renal failure. In your workup you check some labs and calculate a FENa of
0.5%; urine sodium of 15; urine osmolality above 500. What category of ARF etiologies would you place
this patient in?
Prerenal cause
CHAPTER 9
Endocrinology

DIABETES

What is the pathophysiology of type 1 diabetes?


Insulin deficiency due to autoinflammatory destruction of pancreatic B cells

What is the pathophysiology of type 2 diabetes?


Insulin resistance and relative insulin deficiency

What is the age of onset of type 1 and type 2 diabetes?


Type 1 usually begins in childhood/adolescence and type 2 usually begins in adulthood.

Which of the two types of diabetes has a stronger genetic factor?


Type 2 diabetes (seems counterintuitive)

What are the early symptoms of diabetes?


“The three polys”: polyuria, polydipsia, and polyphagia; and weight loss

What are chronic complications of diabetes?


Retinopathy, nephropathy, neuropathy, cerebrovascular disease, coronary artery disease (CAD), peripheral
vascular disease

What type of fatal fungal infection can diabetics get?


Mucor, especially sinusitis (Note: They love to ask this on the boards!)

What is the histologic description of Mucor?


Nonseptate hyphae with branching at 90° (looks like the letter M)

What are the diagnostic criteria for diabetes?


Both types of diabetes are diagnosed based on the same criteria.
Fasting glucose over 126 two separate times
Random glucose over 200 with symptoms of diabetes
Or
Glucose tolerance test (2-hour test with 75-g glucose load) over 200

What is the treatment for type 1 diabetes?


Insulin replacement. Since these individuals do not have insulin, hypoglycemics will not work.

Define each of the following complications of insulin treatment:

What is the first-line treatment for type 2 diabetes?


Metformin

In what patients would metformin be absolutely contraindicated?


In patients who have compromised kidney function because of concern for lactic acidosis

How do we believe metformin works?


Increases sensitivity to insulin

Give an example of each of the following classes of hypoglycemic agents, how they work, and major side
effects:

When is it most appropriate to treat a type 2 diabetic with insulin?


Refractory to oral hypoglycemic agents

What medication slows the progression of nephropathy in diabetes?


Angiotensin-converting enzyme (ACE) inhibitors and angiotensin receptor blockers (ARBs)

Other than medication, what other therapy is important in diabetes?


Nutrition education

What is HgA1c?
Blood marker of glucose control over the last 3 months. HgA1c <7 is ideal.

What preventative measures are recommended to minimize diabetic complications?


Lipid control (low-density lipoprotein [LDL] <70, TG <150).
BP control <130/80.
HgA1c <7.
Annual foot examinations.
Check for microalbuminuria and proteinuria.
Annual funduscopic examination.

What is the appropriate treatment in a diabetic patient with microalbuminuria?


ACE inhibitor or ARB

What is the major complication of type 1 diabetes?


Diabetic ketoacidosis (DKA)

What are the signs and symptoms of DKA?


Severe hyperglycemia (glucose often >500), ketoacidosis, hyperkalemia, fruity breath, slow deep breaths,
abdominal pain, dehydration, lethargy

What are slow deep breaths in DKA called?


Kussmaul hyperpnea

What is the most important treatment in DKA?


Intravenous (IV) fluid hydration (usually with normal saline)

What are the other treatments in DKA?


Insulin drip. Add potassium if potassium is low or normal and add glucose when blood sugar reaches 250
because insulin needs to be continued to be given despite normal glucose until ketones are no longer present.

What are the most severe complications of DKA treatment?


Cerebral edema or cardiac arrest due to hyperkalemia

What is the major complication of type 2 diabetes?


Hyperosmolar hyperglycemic nonketotic (HHNK) coma; although on rare occasions type 2 diabetics can
also go into DKA

What are some of the signs and symptoms of HHNK?


Hypovolemia, hyperglycemia (glucose can be >1000), no ketoacidosis, renal failure, altered mental status,
seizure, disseminated intravascular coagulation (DIC); often precipitated by acute stress such as trauma or
infection.
The difference between HHNK and DKA is that in HHNK there is no ketoacidosis.

What is the treatment for HHNK?


The mortality is >50%; as a consequence, immediate treatment is urgent.
Treatment includes rapid IV fluid resuscitation; insulin and potassium are usually needed earlier than in
DKA because the intracellular shift of plasma potassium during therapy is accelerated in the absence of
acidosis.

PITUITARY

What hormones are secreted from the anterior pituitary?


Follicle-stimulating hormone (FSH), luteinizing hormone (LH), adrenocorticotropic hormone (ACTH),
thyroid-stimulating hormone (TSH), prolactin, GH
(Note: FAST P:G)

What hormones are secreted from the posterior pituitary?


Vasopressin (antidiuretic hormone), oxytocin

What is the action of each of the following hormones?


What is the most common type of pituitary tumor?


Prolactinoma

What type of tumor is a prolactinoma?


A pituitary adenoma which secretes prolactin

What are the two mechanisms by which a prolactinoma causes symptoms?


1. Endocrine effect: due to hyperprolactinemia
2. Mass effect: pressure of the tumor on surrounding tissues

What are some signs and symptoms of a prolactinoma?


Headache, diplopia, hypogonadism, amenorrhea, gynecomastia, galactorrhea, hypopituitarism

What cranial nerve (CN) can be affected by a prolactinoma?


CN III

How is a prolactinoma diagnosed?


Magnetic resonance imaging (MRI)/computed tomography (CT)

What is the first-line treatment for a prolactinoma?


Dopamine agonist such as bromocriptine

What are other treatment options?


Surgical resection or radiation therapy if tumor is very large or refractory to medical treatment

Other than a prolactinoma, what are other causes of hypopituitarism?


Sheehan syndrome (postpartum pituitary necrosis), hemochromatosis, neurosyphilis, tuberculosis (TB),
surgical destruction of pituitary

What disorder is seen with elevated levels of GH?


Acromegaly

What is the most likely underlying cause of acromegaly?


Pituitary adenoma secreting GH

When must there be an elevation in GH in order for acromegaly to result?


Elevated levels of GH must be present after epiphyseal closure.

What results if there is excess GH secretion before epiphyseal closure?


Gigantism

What are the signs and symptoms of acromegaly?


Coarse facial features, large hands and feet, large jaw, deepening of voice, decreased peripheral vision due to
compression of optic chiasm, hyperhidrosis

How is acromegaly diagnosed?


1. MRI/CT demonstrating pituitary tumor,
2. Nonsuppressiblity of GH after an oral glucose challenge
3. Elevtated IGF-1 (insulin-like growth factor)

What are the treatment options for acromegaly?


Surgery or radiation of pituitary tumor, or medical treatment with octreotide or somatostatin, which blocks
GH or dopamine agonists

What malignancy are patients with acromegaly at increased risk for?


Colon cancer

THYROID

What is hyperthyroidism?
Increased secretion of thyroid hormones

In what sex is hyperthyroidism more common?


Ten times more common in women than men

What is the most common cause of hyperthyroidism?


Graves disease (80%-90% of US cases)

What are some other causes of hyperthyroidism?


Plummer disease; toxic multinodular goiter; subacute thyroiditis; amiodarone therapy

What are some of the signs and symptoms of hyperthyroidism?


Heat intolerance, weight loss, exophthalmos, tachycardia, anxiety, palpitations, atrial fibrillation, tremor,
sweating, fatigue, weakness, diarrhea, increased reflex amplitude

What is Graves disease?


Autoimmune disease causing hyperthyroidism. It is due to antibody stimulation of TSH receptors causing
excess secretion of free thyroid hormone.

What are the two symptoms only seen in Graves disease?


1. Pretibial myxedema
2. Infiltrative ophthalmopathy (exophthalmos)

What is pretibial myxedema?


Pruritic, nonpitting edema found on shins that usually remits spontaneously

What is infiltrative ophthalmopathy?


Exophthalmos that may not resolve despite treatment of Graves disease most likely due to autoimmune
damage in extraocular muscles

How is Graves disease diagnosed?


All hyperthyroidism is diagnosed via measurement of TSH, free T4, and free T3. In Graves disease, since
there is excess stimulation of the thyroid gland causing increased production of thyroid hormone, laboratory
tests show high levels of free T4 and free T3, and low levels of TSH (because of negative feedback) (Table
9-1). Also, a radioactive iodine uptake scan should be done. If uptake is low, then thyroiditis or medication-
induced hyperthyroidism is considered.

Table 9-1 Thyroid Function Evaluation

What is another name for toxic multinodular goiter?


Plummer disease

What is the underlying cause of hyperthyroidism in Plummer disease?


Multiple thyroid nodules develop autonomous T4 secretion and, therefore, more T4 is released.

How is Plummer disease diagnosed?


Radioactive iodine uptake tests show “hot” nodules with the rest of the gland being “cold”; also, clinically,
nodules can sometimes be felt.
What is another name for subacute thyroiditis?
de Quervain thyroiditis

What are the signs and symptoms of subacute thyroiditis?


Prodrome of viral upper respiratory infection (URI) followed by rapid onset of thyroid swelling and
tenderness as well as hyperthyroid symptoms that can later turn into a hypothyroid state

What is the treatment for de Quervain thyroiditis?


Usually self-limiting, but aspirin and corticosteroids may be indicated to control inflammation.

What are the treatment options for a hyperthyroid state?


1. Medication: propylthiouracil (PTU) or methimazole
2. Radioactive iodine ablation
3. Surgery: subtotal thyroidectomy

What is the first-line treatment for Graves disease?


Radioactive iodine ablation except in children and pregnant women

What is radioactive iodine ablation?


Radioactive iodine is concentrated in the gland and destroys tissue.

What are the possible side effects of radioactive iodine ablation?


Hypothyroidism; thyrotoxic crisis secondary to the release of thyroid hormone into the blood stream

What is the mechanism by which PTU works?


It inhibits the peripheral conversion of T4 to T3, decreases iodine uptake, and decreases T4 synthesis.

Do patients need to be on therapy for the rest of their lives?


No. After a 1-2 year course of treatment about 50% no longer need to be treated.

What are the potential side effects of PTU?


Leukopenia, rash, nausea

What other adjunctive treatment is given to patients with hyperthyroidism?


Beta-blocker, usually propranolol, to control symptoms

What is the most serious complication of hyperthyroidism?


Thyroid storm

What can induce thyroid storm?


Infection, surgery, trauma, abrupt stop of antithyroid medication, serious acute medical problems such as
cerebrovascular accident (CVA) or myocardial infarction (MI)

What are the signs and symptoms of thyroid storm?


Exaggerated symptoms of hyperthyroidism are tachycardia, high output congestive heart failure (CHF),
abdominal pain, hyperpyrexia >104, altered mental status (ultimately coma)

What is the mortality rate of thyroid storm?


Up to 50%

What is the initial treatment for thyroid storm?


It is an emergency, so think of the ABCs:
Airway stabilization
Breathing/oxygen administration
Circulation (check pulse/blood pressure [BP]) and start IV fluids

After primary stabilization of the patient, what is the medical management of thyroid storm?
The goal of therapy is to decrease circulating thyroid hormone and treat the patient’s symptoms.
1. Prevent hormone synthesis: methimazole or PTU
2. Prevent hormone release: cold iodine (about 2 hours after PTU to prevent worsening symptoms)
3. Prevent conversion of T4 to T3: glucocorticoids and beta-blockers
4. Symptomatic treatment: beta-blockers and Tylenol (for fever)

What are the signs and symptoms of hypothyroidism?


Cold intolerance, fatigue, lethargy, weakness, constipation, weight gain, arthralgias, hoarse voice, skin is
dry, coarse, and with nonpitting edema, loss of outer third of eyebrows, delayed relaxation phase of deep
tendon reflexes

What is primary hypothyroidism?


Thyroid gland dysfunction

What are some examples of primary hypothyroidism?


Hashimoto thyroiditis, thyroid ablation or neck radiation therapy in the past, subacute thyroiditis, iodine
excess or deficiency, medication-induced

What medication can cause hypothyroidism?


Lithium

What is the most sensitive lab test for primary hypothyroidism?


Elevated TSH

What other lab results are present in primary hypothyroidism?


Low T3 and T4

What is Hashimoto thyroiditis?


Painless chronic autoimmune thyroid inflammation of autoimmune etiology

What lab results can help diagnose Hashimoto thyroiditis?


Elevated antithyroglobulin and antimicrosomal antibody titers

What is subacute thyroiditis?


Tender, enlarged thyroid; often postviral infection can begin with hyperthyroid symptoms, then hypothyroid
symptoms

How can you distinguish Hashimoto from subacute thyroiditis?


On clinical examination, in Hashimoto the thyroid gland is not tender to palpation but in subacute thyroiditis
it is tender to palpation.

How can Graves disease and Hashimoto thyroiditis be distinguished?


Radioactive iodine uptake is increased with Graves and decreased with Hashimoto.

What is secondary hypothyroidism?


Hypothyroidism caused by pituitary dysfunction

What are some examples of secondary hypothyroidism?


Sheehan syndrome, pituitary neoplasm, TB
What is Sheehan syndrome?
Postpartum pituitary necrosis

What lab results indicate a secondary hypothyroidism?


Low to normal TSH as well as normal thyrotropin-releasing enzyme (TRH), low levels of T3 and T4

What is tertiary hypothyroidism?


Deficiency of TRH

What is an example of tertiary hypothyroidism?


Hypothalamic radiation

Other than TSH, TRH, T3, T4, what other abnormal lab tests may be found in a hypothyroid patient?
Elevated serum cholesterol (TG, LDL, total cholesterol); elevated aspartate aminotransferase (AST) and
alanine aminotransferase (ALT); anemia; hyponatremia

What is the treatment for hypothyroidism?


Levothyroxine

What is subclinical hypothyroidism?


Elevated TSH levels but with normal thyroid hormone levels and with no clinical symptoms

What is the life-threatening complication of hypothyroidism called?


Myxedema coma

What are the signs and symptoms of myxedema coma?


Severe lethargy or coma, hypothermia, areflexia, bradycardia

What causes myxedema coma?


Prolonged cold exposure, infection, sedatives, narcotics, trauma, or surgery

What is the treatment for myxedema coma?


This is an emergency, so start with ABCs (airway, breathing, circulation); IV fluids, steroids, levothyroxine,
treat any precipitating causes

What is the initial appropriate workup of a thyroid mass?


Fine-needle biopsy and TSH

What other studies are done to workup a thyroid mass?


Thyroid ultrasound to determine the number and sizes of masses; and thyroid technetium 99m scan

What is a hot nodule and a cold nodule on a thyroid scan?


Hot nodule indicates a hyperactive nodule and is less likely to be malignant. A cold nodule indicates a
hypoactive nodule that is more likely to be malignant.

What is the most common type of thyroid cancer?


Papillary cancer

What is the prognosis for papillary cancer?


85%, 5-year survival

What is seen on pathology?


Psammoma bodies, Orphan Annie nucleus
Which type of thyroid carcinoma is associated with multiple endocrine neoplasia types 2 and 3 (MEN 2 and
3)?
Medullary cancer

What can be used to monitor medullary carcinoma?


Calcitonin, because it is a calcitonin-secreting tumor

Which type of thyroid carcinoma has the worst prognosis?


Anaplastic cancer

In what patient population is anaplastic carcinoma usually found?


Older patients

What is the 5-year prognosis for anaplastic carcinoma?


5%-14% survival at 5 years

Which thyroid cancer has the second worst prognosis?


Medullary cancer

Which thyroid carcinoma often has metastasis to the bone and lungs?
Follicular cancer

Name the tumors that are part of each of the MEN syndromes.
1. MEN 1: Wermer syndrome: three Ps: prolactinoma, parathyroid, pancreatoma
2. MEN 2: Sipple syndrome: pheochromocytoma, medullary thyroid, parathyroid
3. MEN 3: same as MEN 2B: pheochromocytoma, medullary thyroid, mucocutaneous neuromas

PARATHYROID

What is primary hyperparathyroidism?


Increased secretion of parathyroid hormone (PTH)

What is the most common cause of primary hyperparathyroidism?


Adenoma is the most common cause; however, other etiologies include hyperplasia, carcinoma, MEN 2 or 3.

What does elevated PTH cause?


There is an ultimate increase in serum calcium (hypercalcemia) because PTH leads to increased vitamin D
hydroxylation and, therefore, increased calcium resorption as well as decreased resorption of phosphate
(hypophosphatemia). Calcium levels are also increased because of increased osteoclastic activity
(osteoporosis).

What are the signs and symptoms of hyperparathyroidism?


Same as those for hypercalcemia: “Stones, moans, groans, and psychiatric overtones.” Because of the
osteoclastic activity it can also lead to osteoporosis.

What EKG finding could you expect with hyperparathyroidism?


Shortened QT, because of hypercalcemia

How is hyperparathyroidism diagnosed?


Hypercalcemia, hypophosphatemia, hypercalciuria, and PTH level
What other differential diagnoses should be considered with hypercalcemia?
Neoplasm, sarcoidosis, thiazide diuretic treatment, Paget disease, vitamin D intoxication, milk alkali
syndrome, myeloma

What is the acute medical treatment for hyperparathyroidism?


Asymptomatic patients with calcium levels below 13 should just be watched. However, symptomatic
patients or those with higher calcium levels should be treated with furosemide and bisphosphonates to
decrease bone resorption and prevent osteoporosis. Calcitonin can be used as well.

What long-term treatment must be considered in hyperparathyroidism?


Surgical treatment. Adenomas should be removed. In hyperplasia, all four parathyroids are removed and a
small piece is placed usually near the sternocleidomastoid for functionality.

What are the most common complications of parathyroidectomy?


Hoarseness because of damage of the recurrent laryngeal nerve and hypocalcemia

What is secondary hyperparathyroidism?


Increased PTH secretion secondary to chronic renal failure or vitamin D deficiency

What is hypoparathyroidism?
Decreased PTH

What are the causes of hypoparathyroidism?


Idiopathic, DiGeorge syndrome, hypomagnesemia, secondary to surgery or neck irradiation

Why does hypomagnesemia lead to hypoparathyroidism?


Because magnesium is necessary for the parathyroid to secrete PTH.

In what conditions is low magnesium seen?


Syndrome of inappropriate secretion of antidiuretic hormone (SIADH), pancreatitis, alcoholism

How is hypoparathyroidism diagnosed?


Hypocalcemia, hyperphosphatemia, low PTH

What are the signs and symptoms of hypoparathyroidism?


Same as that for hypocalcemia: perioral paresthesias, tetany, seizures, Trousseau sign, Chvostek sign,
anxiety

What EKG findings could you expect in hypoparathyroidism?


Prolonged QT interval because of the hypocalcemia

What is Trousseau sign?


Carpal spasm with arterial occlusion with BP cuff

What is Chvostek sign?


Spasm of the facial nerve upon tapping

How is hypoparathyroidism treated?


Emergently treat with IV calcium, then treat with vitamin D and oral calcium for maintenance treatment.

ADRENALS


What are the two main parts of the adrenal gland and what is the secretory product of each part?
1. Adrenal cortex
2. Adrenal medulla
The cortex secretes aldosterone, cortisol, and sex hormones and the medulla secretes the catecholamines
including epinephrine and norepinephrine.

What is the function of aldosterone?


Kidney resorption of sodium and secretion of potassium and hydrogen ions

What is Addison disease?


Primary adrenal insufficiency caused by the destruction of the adrenal cortex leading to a deficiency in both
mineralocorticoids as well as glucocorticoids

What is secondary adrenal insufficiency?


Decreased secretion of ACTH by the pituitary gland; the adrenal gland is functional

What is the cause of tertiary adrenal insufficiency?


Decreased hypothalamic function

What is the most likely etiology of Addison disease in the United States?
Autoimmune destruction of the adrenal gland

What are some other causes of Addison disease?


TB, amyloidosis, sarcoidosis, HIV, adrenal hemorrhage secondary to DIC or trauma, Waterhouse-
Friderichsen syndrome, congenital adrenal hyperplasia, metastasis to the adrenals

What is Waterhouse-Friderichsen syndrome?


Endotoxin-mediated adrenal hemorrhage usually caused by meningococcemia that leads to fulminant adrenal
failure

What is the most likely cause of secondary adrenal insufficiency?


Hypothalamic-pituitary axis disturbance, usually by sudden cessation of exogenous corticosteroids, which
leads to decreased ACTH secretion

What are some other causes of secondary adrenal insufficiency?


Pituitary infarction, Sheehan syndrome, pituitary adenoma

What are some signs and symptoms of Addison disease?


Because of low aldosterone and cortisol there are hyponatremia, hyperkalemia, pica (craving for salt),
weakness, anorexia, hypotension, nausea, vomiting, and hyperpigmentation.

What is the test used to diagnose adrenal insufficiency?


Hyperpigmentation, ↑ACTH,↓ cortisol and aldosterone response to ACTH challenge

Why do patients get hyperpigmentation?


ACTH stimulates melanin secretion.

What are the diagnostic findings in primary adrenal insufficiency?


ACTH (Cortrosyn) test in which a dose of ACTH is given to the patient and then serum cortisol levels as
well as serum ACTH levels are measured about half an hour later
Primary adrenal insufficiency: ↑cortisol levels in response to ACTH and ↑aldosterone levels
Secondary adrenal insufficiency: ↑cortisol levels (more than double normal limits) in response to ACTH and
normal aldosterone levels
How is the diagnosis of secondary adrenal insufficiency distinguished from primary adrenal insufficiency?
No hyperpigmentation, ↑cortisol response, ↑ACTH

What kind of metabolic disturbance is seen in primary adrenal insufficiency?


Metabolic acidosis due to aldosterone and cortisol deficiency and, therefore, lack of secretion of hydrogen
ions

What is the treatment for adrenal insufficiency?


Glucocorticoid replacement. Extra glucocorticoids should be given in times of physical stress such as
infection. You should instruct patients to taper off this extra replacement slowly as to prevent an adrenal
crisis.

What is Cushing syndrome?


A term used to describe the symptoms caused by hypercortisolism

How is Cushing syndrome different from Cushing disease?


Cushing disease refers to a type of Cushing syndrome caused specifically by ACTH hypersecretion by the
pituitary.

What are the different causes of hypercortisolism?


Exogenous glucocorticoids Pituitary hypersecretion of ACTH Hypersecretion of cortisol due to adrenal
hyperplasia/neoplasm Ectopic ACTH production such as with small cell lung carcinoma

What is the most common cause of Cushing syndrome?


Exogenous corticosteroids

What is the most common cause of endogenous hypercortisolism?


Cushing disease (pituitary hypersecretion of ACTH)

What are the signs and symptoms of Cushing syndrome?


Buffalo hump, moon facies, truncal obesity, striae, virilization/menstrual disorders, hyperglycemia,
hypertension, hypokalemia, immune suppression, osteoporosis, hirsutism, acne

What tests are used to diagnose hypercortisolism?


24-hour urine-free cortisol and the dexamethasone suppression tests, ACTH level, diurnal cortisol variation

What is the dexamethasone suppression test?


First a low dose of dexamethasone is given and cortisol is measured. If cortisol is not elevated then Cushing
is ruled out; if it is elevated then a high-dose dexamethasone suppression test is done and ACTH is
measured. If ACTH is decreased then the pituitary has good feedback and, therefore, it must be an adrenal
etiology. However, if the ACTH is high or normal then it is probably ectopic ACTH; and if it is only
partially suppressed, then the pituitary is the etiology. Dexamethasone → ↑ACTH (ectopic/pituitary)
↓ACTH (adrenal)

What are some other studies to consider to localize the lesion in hypercortisolism?
A CT scan can look for an adrenal mass and an MRI can look for a pituitary mass.

What is the treatment for hypercortisolism?


Treat the underlying cause. If it is a resectable tumor, tumor resection with postoperative glucocorticoids. In
nonresectable tumors, medical therapy with ketoconazole, mitotane, metyrapone, or aminoglutethimide. If
the etiology is exogenous glucocorticoids; taper off the glucocorticoids and eventually stop.

What is Conn syndrome?


Primary hyperaldosteronism

What is the etiology of Conn syndrome?


Either hyperplasia of the zona glomerulosa or aldosterone-producing adenoma

What are the signs and symptoms of Conn syndrome?


Hypertension, muscle cramps, palpitations, polyuria, polydipsia, hypokalemia

What percent of hypertensive patients have Conn syndrome?


1%-2%

What are some of the laboratory findings in Conn syndrome?


↑Na, ↑Cl, ↓K (muscle cramps, palpitations), ↓renin-angiotensin feedback, metabolic alkalosis

What are some ways to diagnose Conn syndrome?


Captopril stimulation test; fludrocortisone suppression test; sodium loading

What is the captopril stimulation test?


Captopril (an ACE inhibitor) is administered and then serum renin and aldosterone levels are measured.
↑aldosterone and ↓ renin confirm the diagnosis.

What is the fludrocortisone suppression test?


Fludrocortisone, a synthetic corticosteroid, is administered to the patient. Serum aldosterone levels are then
measured. In a normal patient it would be expected that aldosterone levels would be suppressed but not in a
patient with Conn syndrome.

What is the sodium loading test?


The patient is loaded with sodium via IV saline and then urinary aldosterone levels are tested. No decrease in
urinary aldosterone confirms diagnosis.

What is the renin level in Conn syndrome?


Low renin

What other study can help in the diagnosis of Conn syndrome?


CT demonstrating an adrenal nodule or hyperplasia

What is the treatment for Conn syndrome?


Adrenal adenoma: resection of tumor; unilateral adrenal hyperplasia: unilateral adrenalectomy; bilateral
adrenal hyperplasia: spironolactone (potassium-sparing diuretic) or ACE inhibitor to control blood pressure

What is secondary hyperaldosteronism?


Elevated aldosterone levels due to elevated renin levels secondary to renal ischemia in CHF, renal artery
stenosis, shock, renal tumor.

How is secondary hyperaldosteronism diagnosed?


↓Renin

What can be measured to differentiate primary from secondary hyperaldosteronism?


Renin (this is very important)

What is the treatment for secondary hyperaldosteronism?


Treat the hypertension with a potassium-sparing diuretic, a beta-blocker, and treat the underlying cause.
What is a pheochromocytoma?
Tumor of the adrenal medulla that produces excess catecholamines

What percentage of people with hypertension have a pheochromocytoma?


0.5%

What are the possible etiologies for a pheochromocytoma?


MEN 2 or 3, von Hippel-Lindau disease, Recklinghausen disease, neurofibromatosis

What are the five Ps of pheochromocytoma?


1. Pain (headache)
2. Pressure
3. Perspiration
4. Palpitation
5. Pallor and hypertension

What is the most common sign of a pheochromocytoma?


Hypertension

What is the diagnostic test for a pheochromocytoma?


Urine screen for elevated VMA (vanillylmandelic acid), a urine catecholamine; as well as elevated urine and
serum epinephrine and norepinephrine levels

What other test can be done to localize a pheochromocytoma?


A CT scan can identify a suprarenal mass (adrenal mass).

What are some other laboratory findings in a pheochromocytoma?


Hyperglycemia, polycythemia

What is the “rule of 10s” for a pheochromocytoma?


10% malignant
10% bilateral
10% extrarenal
10% familial
10% in kids
10% multiple tumors
10% calcified

What must be ruled out in a patient with a pheochromocytoma?


MEN type 2 or 3 or

What is the treatment for a pheochromocytoma?


In operative cases preoperative alpha-blockers and beta-blockers, then surgical resection; in inoperable cases
phenoxybenzamine or phentolamine

Why treat with preoperative alpha-blockers and beta-blockers?


To prevent unopposed vasoconstriction, and thus volume depletion

BONES

What is osteoporosis?
Reduction in bone mass leading to increased risk of fracture

What are the risk factors for osteoporosis?


Female, postmenopausal or low estrogen state, hypercortisolism, hyperthyroidism, calcium deficiency, low
physical activity, smoking

What are the typical fractures that occur in osteoporosis?


Hip, vertebrae, and Colle fractures

How is osteoporosis diagnosed?


Dual-energy x-ray absorptiometry (DEXA) scan which shows low bone density or an incidental fracture in
the elderly

What are the treatments for osteoporosis?


Bisphosphonates, calcitonin, selective estrogen receptor modulators, calcium

How much calcium should be taken daily?


1500 mg daily with vitamin D

What is the calcitonin most useful for?


Treating bone pain; however, it cannot be used chronically because the effects wear off

What are some examples of selective estrogen modulators?


Tamoxifen, raloxifene

What do the selective estrogen modulators increase the risk for?


Thromboembolism

What is osteomalacia?
Vitamin D deficiency in adults

What is osteomalacia called in children?


Rickets

What are the signs and symptoms in children?


Pigeon breast, craniotabes (thin skull bones), rachitic rosary (costochondral thickening)

How is osteomalacia diagnosed?


Low levels of vitamin D as well as diffuse osteopenia on x-ray

How is osteomalacia treated?


Vitamin D supplementation

What is Paget disease of the bone?


Localized hyperactivity of the bone which leads to disordered bone matrix being replaced with soft, enlarged
bone

What is the etiology of Paget?


Unknown, but some think it may be viral

What are the signs and symptoms of Paget disease of the bone?
Hearing loss (impingement of cranial nerve [CN] VIII), multiple fractures, bone pain, high-output cardiac
failure, increased hat size
What is the typical finding on x-ray?
Hyperlucent area surrounded by hyperdense border-sclerotic lesions

How is Paget diagnosed?


Elevated alkaline phosphatase, sclerotic lesions on bone scans/x-rays

What are the complications associated with Paget disease of the bone?
Pathologic fractures, high-output cardiac failure, hearing loss, kidney stones, sarcoma, spinal cord
compression

What is the treatment for Paget disease?


Most patients do not need treatment; however, patients with complications associated with Paget disease are
treated with bisphosphonates as first line and calcitonin as second line.

CLINICAL VIGNETTES

You diagnose a patient with type 2 diabetes. You check a urine microalbumin and find that it is elevated.
With what class of medication would you treat this patient?
ACE inhibitor

A 24-year-old male comes to your office complaining of terrible headaches over the past several months.
His only past medical history is GERD. He has no past surgical history. The only family history is prostate
cancer in his grandfather, otherwise the rest of his family is healthy. On review of systems, he complains of
chest palpitations and says that he sweats a lot. His vitals demonstrate a BP of 173/98. On examination you
notice that he appears somewhat pale. His cardiovascular, pulmonary, and abdominal examinations are
unremarkable. His electrolytes are within normal limits. You suspect a secondary cause of hypertension.
What specific diagnostic test would help you screen for your suspected diagnosis?
Urine VMA to screen for pheochromocytoma

Your patient has weight loss, heat intolerance, and palpitations. She complains of swelling and tenderness
of her neck. She just got over a head cold. What is the suspected diagnosis?
Subacute thyroiditis

Your diet-controlled diabetic patient presents for a follow-up. The only medication he currently takes is
lisinopril. His vitals are as follows: BP: 125/70; P: 73; RR: 15; Temp: afebrile. You review his most recent
laboratory tests with him. His HgA1c is 6.8. His urinalysis shows no protein. The lipid profile demonstrates
LDL : 110, HDL: 45, TG: 100. His most recent fundoscopic examination was 4 months ago and was
normal. You do a foot examination and that is normal. What medication change do you suggest?
Add a statin to bring the LDL down below 100.

A 34-year-old male with hypertension presents to your clinic trying to seek your advice regarding his
recent weight gain He has gained 20 lb over the course of the last 3 months but denies any change in his
diet. He appears to have quite a bit of abdominal girth as well as noticeable striae on his abdomen. His face
is also noticeably round and with significant acne. What do you suspect is this patient’s condition?
Cushing syndrome
CHAPTER 10
Infectious Disease

HIV/AIDS

What is HIV (human immunodeficiency virus?


A retrovirus that destroys CD4 cells

How is HIV transmitted?


Sexual contact, blood products, mother to child in HIV positive mothers, needle stick injury

How is acquired immunodeficiency syndrome (AIDS) defined?


CD4 count <200 or evidence of an AIDS defining condition or T-helper cell <200/μL of blood or 14% of all
lymphocytes

Describe the life cycle of HIV?


gp120 bind CD4 molecule → gp41 molecule helps HIV to fuse with host cell → HIV RNA released into
host cell → reverse transcriptase converts viral RNA into DNA → viral DNA translocates into nucleus and
viral DNA fuses with host DNA → host cell transcribes the integrated DNA → mRNA is translated into
HIV polypeptides which are cleaved by viral proteases → new virus particles assemble to create a new virus
cell

How is an HIV infection diagnosed?


A positive enzyme-linked immunosorbent assay (ELISA) for HIV is then confirmed with a Western blot
assay

How is HIV ruled out?


A negative ELISA for HIV

What marker is used to follow the extent of disease?


CD4 count

What can be used as a marker of disease progression?


Viral load (it will tell how well the treatment is working)

What are the signs and symptoms of acute HIV?


Flu-like symptoms that can later subside

Name the complications associated with each of the following CD4 counts:

When should antiretroviral therapy be initiated?


At CD4 counts <350

What is the antiretroviral therapy called?


Highly active antiretroviral therapy (HAART)

What does HAART therapy usually include?


Two nucleoside analogues and a protease inhibitor

Name the medical management that should be initiated for each of the following CD4 counts:

Name the AIDS-related opportunistic infection/complication associated with the following:


What is the treatment for each of the following opportunistic infections?

SEXUALLY TRANSMITTED DISEASES


Which sexually transmitted disease (STD) is caused by the spirochete Treponema pallidum?
Syphilis

Name the stage of syphilis associated with the following:

What are gummas?


Rubbery granulomatous lesions in CNS, aorta, heart, skin, bone

What is tabes dorsalis?


Posterior column degeneration

How is syphilis diagnosed?


Four possible tests:
1. Venereal disease research laboratory (VDRL)/rapid plasma reagin (RPR)-rapid test, however nonspecific blood
test (eg, can be falsely positive in systemic lupus erythematosus [SLE]).
2. Dark-field microscopy would show motile spirochetes.
3. EIA (enzyme immunoassay): tests for antitreponemal IgG; can be used to screen for syphilis.
4. FTA-ABS/MHA-TP (fluorescent treponemal antibody/microhemagglutination assay—T pallidum): sensitive
and specific; it remains positive for life.

What is the treatment for syphilis?


Penicillin; doxycycline or tetracycline can be given to penicillin-allergic patients (but not for CNS disease)

Which STD often coexists with gonorrhea?


Chlamydia

How can Chlamydia present?


Asymptomatic, cervicitis, urethritis, salpingitis or pelvic inflammatory disease (PID)

What are the signs and symptoms of Chlamydia infection with PID?
Mucopurulent discharge with adnexal pain

What is Fitz-Hugh-Curtis syndrome?


Complication of gonorrhea or Chlamydia in which there is perihepatic inflammation and fibrosis

What is lymphogranuloma venereum?


Systemic disease caused by the Chlamydia L serotype causing painful inguinal lymphadenopathy called
buboes

What is the treatment for Chlamydia infection?


Doxycycline or azithromycin

What sexually transmitted disease is caused by a gram-negative diplococcus?


Gonorrhea

What is a major complication of gonorrhea?


PID

On what type of medium is gonorrhea diagnosed?


Thayer-Martin

How is gonorrhea treated?


Third-generation cephalosporin with concomitant treatment of Chlamydia

How is PID diagnosed?


Cervical motion tenderness plus at least one of the following: positive Gram stain; fever; elevated WBCs,
tuboovarian abscess; pus on culdocentesis

What is the most common cause of vaginitis?


Bacterial vaginosis caused by Gardnerella

What are the signs and symptoms of vaginitis?


Vaginal itching, burning, bad odor, discharge, and dyspareunia

What is the classic odor associated with bacterial vaginosis?


Fishy odor = positive Whiff test with KOH prep

How is bacterial vaginosis diagnosed?


Clue cells (epithelial cells coated with bacteria) on wet mount

How is bacterial vaginosis treated?


Metronidazole

Which type of vaginitis is caused by a flagellated, motile protozoan?


Trichomonas

How is Trichomonas diagnosed?


The protozoa are seen on wet mount.

What are the classic symptoms of Trichomonas infection?


Fishy odor of discharge and strawberry cervix

How is Trichomonas treated?


Patient and partner are treated with metronidazole.

Which type of vaginitis is associated with a cheesy white discharge?


Candida (also known as yeast infection)

How is candidiasis diagnosed?


Pseudohyphae on KOH prep

How is a Candida infection treated?


Nystatin cream or oral fluconazole (Diflucan)

Which types of human papillomavirus (HPV) are associated with cervical cancer?
16, 18, 31, 45, 51, 52, 53

What are the two vaccines approved to protect against cervical cancer?
1. Gardasil
2. Cervarix

Who should get the vaccine?


Females aged 12-26

Which of the two vaccines is also protective against genital warts?


Gardasil

When should a female start getting Pap smears?


Age 21 or 3 years after first sexual activity, whichever comes first

How often should a Pap smear be done?


If a patient has had three normal consecutive Pap smears, they can get them every 3 years after age 30.

SEPSIS

What is sepsis?
An infection that causes systemic inflammatory response syndrome (SIRS)

What is SIRS?
Includes the following:
1. Tachycardia
2. Tachypnea
3. Fever
4. WBC count >12,000, <4000, or >10% bands

What is septic shock?


Sepsis-induced hypotension

What type of bacteria causes shock secondary to exotoxin-induced fluid loss?


Gram-positive bacteria

What type of bacteria causes shock secondary to endotoxin-induced vasodilatation?


Gram-negative bacteria

What are some of the signs and symptoms of sepsis?


Fever, hypotension, tachycardia, tachypnea, disseminated intravascular coagulation (DIC), increased cardiac
output
What is the treatment of sepsis?
Intravenous (IV) fluids, antibiotics to treat infection, vasopressors, remove potential sources of infection
such as Foley catheter, sometimes steroids

OSTEOMYELITIS

What is osteomyelitis?
Bone infection

What are the two main routes of bone infection?


1. Direct spread from soft tissue infection
2. Hematogenous seeding

What types of patients are predisposed to getting osteomyelitis by direct spread?


Diabetics, people with peripheral vascular disease, deep soft tissue injuries

What is the most common organism causing osteomyelitis?


Staphylococcus aureus

What is the most common cause of osteomyelitis in a patient with sickle cell anemia?
Salmonella

What are the two most common causes of osteomyelitis in a patient who is an IV drug user?
Pseudomonas, S aureus

What is the most common cause of osteomyelitis in a patient with a deep foot puncture wound?
Pseudomonas

What is the gold standard diagnostic technique to evaluate osteomyelitis?


Fever, bone pain, warmth, swelling, erythema of overlying skin, with limited range of motion of the area
affected

What is the classic finding on x-ray?


Periosteal elevation; lytic lesion

What are the signs and symptoms of osteomyelitis?


Magnetic resonance imaging (MRI)

What is the treatment for osteomyelitis?


Appropriate IV antibiotics for 4-6 weeks

What are possible complications of osteomyelitis?


Chronic osteomyelitis, sepsis, septic arthritis, squamous cell carcinoma secondary to a draining sinus tract

CLINICAL VIGNETTES

A 32-year-old sexually active female complains of vaginal itching and burning with malodorous discharge.
On a wet mount you find epithelial cells coated with bacteria. How do you treat this patient?
Bacterial vaginosis is treated with metronidazole.
Your diabetic patient presents with erythema and swelling over the anterior portion of his shin. It is warm
and very painful. He is febrile. He states that his blood sugars have been poorly controlled recently. His
HgA1c is 9.6. You get an x-ray and find periosteal elevation. What is the diagnosis?
Osteomyelitis

A 23-year-old sexually active female presents with vaginal discharge. She has also had a fever. A pelvic
examination is not tolerated by the patient due to severe pain. You do a Gram stain and find gram-negative
diplococci. How do you treat her?
Third-generation cephalosporin to treat the gonorrhea and add doxycycline or azithromycin to cover for a
possible coinfection with Chlamydia
CHAPTER 11
Dermatology

TERMINOLOGY

Name the primary dermatologic skin lesion described below:

SKIN CANCERS

What is the most common type of skin cancer?


Basal cell carcinoma (BCC)

What are the three main characteristic of a BCC seen on physical examination?
1. Pearly papule
2. Telangiectasias
3. Traslucent border

What is the classic description of a BCC?


“Rodent ulcer”(Fig 11-1)

Figure 11-1 Rodent ulcer. (Courtesy of Noah Craft, MD, PhD)

What is the skin cancer most likely to cause death?


Melanoma
What are the risk factors for BCC (Fig 11-2)?
Sun exposure, fair skin, radiation therapy

Figure 11-2 BCC-pearly papule. (Courtesy of Noah Craft, MD, PhD)

Where are BCCs most commonly found?


Sun-exposed skin, ie, head, neck, hands

How is a BCC diagnosed?


Biopsy

What is the treatment?


Excision

What is the prognosis?


Prognosis is excellent because this cancer rarely metastasizes.

What is second most common skin cancer?


Squamous cell carcinoma (SCC)

What is the precursor lesion to SCC?


Actinic keratosis (also known as solar keratosis)

What are the characteristics of an actinic keratosis on physical examination?


Red, scaly, rough patches usually found in sun-exposed area of skin

How are actinic keratoses treated?


Cryotherapy for a small number of lesions, topical 5-FU (an antimitotic agent) for large areas on face and
scalp, or imiquimod cream, also photodynamic therapy

What are the risk factors for developing a SCC?


Sun exposure, fair skin, radiation therapy, xeroderma pigmentosa, exposure to arsenic, immunosuppression

Where are SCCs most commonly found?


Sun-exposed areas of skin, ie, head, neck, hand

How is SCC diagnosed?


Biopsy shows “keratin pearls” in the dermis.

What is the treatment?


Excision; radiation in cases where surgery is not an option

What is the prognosis?


Prognosis is very good. They metastasize more often than BCC but not as often as melanoma.

What is the type of skin cancer most likely to be found in younger age groups?
Melanoma (Fig 11-3)

Figure 11-3 Melanoma. (Courtesy of Noah Craft, MD, PhD)

What are the risk factors for melanoma?


Remember the mnemonic ABCDEE:
Asymmetry
Borders are irregular
Colors vary
Diameter is >6 mm (larger than a pencil eraser)
Enlarged over time (growing)
Elevation

What characteristics are most suggestive of melanoma?


Sun exposure (particularly childhood sunburn), fair skin, family history

How is melanoma diagnosed?


Excisional or incisional biopsy shows melanocytes with atypia. Do NOT do a shave biopsy.

What is the most important prognostic factor for melanoma?


Depth of invasion or thickness of melanoma; the deeper the lesion the worse the prognosis (Fig 11-4)

Figure 11-4 Clark and Breslow classification schemes.

What is Breslow classification?


Breslow classification: Staging is done by measuring the depth of the lesion in millimeters.

What is Clarke classification?


Clarke classification: Staging is done by determining the penetration of the lesion in relation to the layers
of the dermis.

Which classification scheme is most predictive of survival?


Breslow classification

Name the different types of melanoma.


1. Superficial spreading melanoma
2. Nodular melanoma
3. Lentigo maligna
4. Acral lentiginous

Name the type of melanoma described below:

What is the treatment for melanoma?


Excision; chemotherapy if metastasis is suspected

What is the type of skin cancer associated with HIV?


Kaposi sarcoma

Which herpes virus is associated with Kaposi?


Human herpes virus (HHV) 8

What are the clinical findings of Kaposi?


Red/purple macular or papular nodules on skin, mucous membranes, and viscera (especially lungs,
gastrointestinal [GI] so it may present as shortness of breath)

What is the treatment?


Treat human immunodeficiency virus (HIV); treat lesions if they cause discomfort; intralesional vinblastine;
radiation; chemotherapy

What is mycosis fungoides?


Cutaneous T-cell lymphoma

What is the leukemic phase of the disease called?


Sézary syndrome

What are the clinical findings of mycosis fungoides?


Chronic progressive eczema unresponsive to treatment

PSORIASIS

What do psoriatic lesions look like?


Pink plaques with silvery-white scale (Fig 11-5)

Figure 11-5 Psoriasis. (Courtesy of Noah Craft, MD, PhD)

Where are psoriatic plaques classically found?


On the elbows and knees (extensor surfaces)

What other clinical findings can psoriasis be associated with?


1. Fingernail pitting
2. Oncholysis (separation of distal nail plate from nail bed)
3. Psoriatic arthritis (rheumatologic factor negative)

Which joints do psoriatic arthritis most commonly affect?


Distal interphalangeal (DIP) joints

What is Köbner phenomenon?


Psoriatic lesions that occur at the site of injury

What is Auspitz sign?


Pinpoint bleeding at sites where overlying scale is removed

How is psoriasis affected by season?


Psoriasis is worse in winter and better in summer because sunlight improves lesions.

How is psoriasis treated?


See Table 11-1.
Table 11-1 Management of Psoriasis

What blood tests should be done on patients taking methotrexate?


Complete blood count (CBC) to monitor for bone marrow suppression; liver function test to check for
hepatotoxicity; renal function tests

What blood test should be done on patients taking cyclosporine?


Renal function tests due to the risk of nephrotoxicity

BLISTERING DISEASES

Name the blistering diseases described below:


What is the Nikolsky sign?


Sloughing of epidermis with gentle traction

What is the treatment for blistering diseases?


Oral steroids and antibiotics if infection occurs

VECTOR-BORNE DISEASES

Which vector-borne illness is caused by Rickettsia rickettsii?


Rocky Mountain spotted fever (RMSF)

What are the symptoms?


Fever, headache, rash, myalgias, nausea, photophobia
(Note: Rash Myalgias Severe headache Fever)

What kind of rash is it?


Maculopapular

How does the rash spread?


The rash spreads centrally. It starts at the wrists and ankles and spreads to the palms, soles, and trunk.
(Note: The rash WRAPS: WRists Ankles Palms Soles)
In what months is it likely to be seen?
April through September

In what regions is this illness found?


It is an illness of the Western hemisphere; mainly southeastern states (North/South Carolina, Tennessee,
Oklahoma); rare in the Rocky Mountains

How is RMSF diagnosed?


Usually a clinical diagnosis with a history of being outdoors or tick bite; clinical test results are slow and it is
important to start treatment immediately

What is the most specific and sensitive clinical test for RMSF?
Indirect fluorescent antibody assay

What are some clinical tests to diagnose RMSF?


Serologies for R rickettsii; Weil-Felix test, biopsy showing necrotizing vasculitis

What is considered the best treatment for RMSF?


Doxycycline

What is the treatment of RMSF in children?


Doxycycline, even though tetracyclines are typically not given to children because of the risk of staining
teeth, this is one exception.

How would you treat patients that are pregnant but not in the third trimester?
Chloramphenicol—This is avoided in the third trimester due to the risk of gray baby syndrome.

What is the major side effect of chloramphenicol to watch for?


Aplastic anemia

What vector-borne illness is caused by Borrelia burgdorferi?


Lyme disease

What is this transmitted by?


Ixodes deer tick

What are the symptoms?


Fever, headache, myalgias, photophobia, rash, myocarditis

What is the classic rash called and how does it spread?


Erythema chronicum migrans—erythematous annular plaques at the sites of tick bites expand with central
clearing
(Note: Looks like a target)

How is this rash different from that seen in RMSF?


It does not involve the palm and soles; usually rash is on trunk, extremities, axilla, inguinal regions

In what months is Lyme disease usually seen?


May through September

In what region of the United States is it mostly found?


Northeast
How is Lyme disease diagnosed?
Clinically and confirmed by polymerase chain reaction (PCR) or skin biopsy for B burgdorferi (spirochete)

What is the treatment?


Doxycycline, penicillin

What are the potential complications if treatment is delayed?


Cardiac: carditis, atrioventricular (AV) block
Neurologic: meningitis, encephalitis, Bell palsy

FUNGAL INFECTIONS

Name the fungal infection described below:

What is the causative agent of t versicolor?


Pityrosporum ovale also known as Malassezia furfur

How are these infections diagnosed?


KOH (potassium hydroxide) preparation

What is the “classic finding” on KOH preparation for t versicolor?


Termed “spaghetti and meatballs”
(Note: The spaghetti is the hyphae and the meatballs are the yeast.)

What is seen in the KOH preparation of Candida?


Satellite scrapings show budding yeast and pseudohyphae.

What is seen in KOH preparation of t corporis?


Hyphae

What is the treatment for each of the following?

BACTERIAL AND VIRAL INFECTIONS

What is the causative agent of acne vulgaris?


Propionibacterium acnes cause inflammation of the pilosebaceous unit.

What is the term used for a “blackhead?”


Open comedone

What is the term used for a “whitehead?”


Closed comedone

What are the topical treatments for acne?


Mild acne: Use topicals alone.
Benzoyl peroxide, retinoic acid, erythromycin, or clindamycin, and antiseptics.

What are the oral treatments for acne?


Use in moderate to severe cases (cystic acne).
Oral tetracyclines (doxycycline), erythromycin, clindamycin.
Isotretinoin in very severe cases.

What is the warning that female patients should receive before being placed on an isotretinoin (Accutane)?
Female patients should be put through the “I Pledge” system and be told that they should not become
pregnant while taking this drug because it will cause severe fetal abnormalities.
What is cellulitis?
Subcutaneous, soft tissue infection with classic signs of inflammation. Area of skin is shiny and poorly
demarcated and borders are not elevated.

What are the classic signs of inflammation?


Red (rubor)
Hot (calor)
Painful (dolor)
Swollen (tumor)

What are the most common causative agents of cellulitis?


Staphylococcus and Streptococcus

What is the term used to describe a superficial spreading cellulitis?


Erysipelas

What is the most common causative agent?


Streptococcus pyogenes

What patients are at high risk for cellulitis?


Immunocompromised patients
(Note: If diabetic with tender, erythematous rash on lower extremity unilaterally, think cellulitis)

How is the diagnosis confirmed?


Gram stain with gram-positive cocci

How is it treated?
Penicillin or cephalosporin (cephalexin) If penicillin- or methicillin-resistant Staphylococcus aureus
(MRSA)-allergic, use vancomycin or clindamycin

What are the signs and symptoms of folliculitis?


Erythematous pustules in areas of hair growth especially in beard region

What is the most common causative agent?


S aureus

What is the most common causative agent of “hot tub” folliculitis?


Pseudomonas

What is the treatment?


Keep area clean, if severe can use fluoroquinolone

What is a furuncle?
A collection of puss in one hair follicle

What is a carbuncle?
A collection of puss in multiple hair follicles

What is an abscess?
Localized collection of pus “walled off” by a cavity formed by the surrounding tissue

What is the most common causative agent?


S aureus
What is the abnormal lab value seen?
High white blood cell (WBC) count

What is the treatment for an abscess, carbuncle, and furuncle?


Incision and drainage; antibiotics may be added if needed (cephalexin is typical but it does not cover MRSA
which has become more prevalent and may need an antibiotic such as clindamycin or Bactrim)

What is impetigo?
Superficial skin infection

What is the characteristic description of impetigo?


Honey-crusted lesion

What is the treatment?


Cephalexin, clindamycin (if MRSA)

What are the most common causative agents?


S aureus (children) or S pyogenes (adults)

What is erythrasma?
An erythematous rash along major skin folds (eg, axilla, groin)

In what patient population is it most commonly found?


Diabetics

What is the causative agent?


Corynebacterium

How is it diagnosed?
Under Wood lamp there is coral red fluorescence; KOH preparation is negative.

What is the treatment?


Erythrasma is treated with erythromycin.

What is the term used to describe a plugged apocrine sweat gland that has become infected?
Hidradenitis suppurativa

In what regions of the body is it usually found?


Axilla and groin

What is the treatment?


Surgical debridement and antibiotics

What is the term used to describe an infection of the skin surrounding the nail plate?
Paronychia

What are the most common infective agents?


Staphylococcus or Streptococcus

What is the treatment?


Warm compress, incision and drainage (ID) if purulent, keflex if severe

What is herpes simplex?


Recurrent, painful vesicular eruptions in groups due to the herpes simplex virus (HSV) infection

Where are the lesions most commonly found?


Oral-labial region or genitals

What form of the virus is most commonly found at each of the regions above?
HSV 1: oral-labial
HSV 2: genital
(Note: Think from top to bottom—type 1 then type 2) (Fig 11-6)

Figure 11-6 Typical locations of HSV 1 and HSV 2 eruptions.

How is it diagnosed?
Tzanck smear—positive for HSV when multinucleated giant cells are seen

What is the treatment?


Acyclovir ointment reduces duration but does not prevent recurrence. Oral acyclovir reduces frequency and
recurrence.

What is herpes zoster?


Also known as shingles; an acute, dermatomal vesicular eruption caused by the reactivation of latent
varicella zoster that has been dormant in the sensory root ganglion (Fig 11-7)

Figure 11-7 Dermatomal distribution of herpes zoster.

What is the typical history of symptoms?


Day 1: dermatomal pain (no lesions), can also present with fever, malaise
Day 3-5: unilateral grouped vesicles along a dermatome
Day 5-10: crust formation

Which nerves are most commonly involved?


Thoracic nerves

What test is used to confirm the diagnosis?


Tzanck smear—multinucleated giant cells revealed (same as with herpes simplex)

What is the term used to describe herpes infection of the geniculate ganglion which leads to vesicles
forming on the external auditory meatus?
Ramsay Hunt syndrome (RHS)

What can happen if RHS is not treated rapidly?


It could extend to meningitis. It can also lead to facial paralysis and hearing loss.

What is the treatment for herpes zoster?


Oral acyclovir within 3 days of infection; immunocompromised patient: IV acyclovir
Analgesia to all patients (it hurts)

What are potential complications of herpes zoster?


1. Superficial infection of affected area.
2. Postherpetic neuralgia (may last for years).
3. V1 (primary visual area) involvement can lead to corneal scarring.

PIGMENTARY DISORDERS

Name the pigmentary disorders described below:

What autoimmune disorder is associated with vitiligo?


Thyroid disease

What are the potential treatments for vitiligo?


Vitiligo cannot be cured because it is autoimmune in nature, but skin grafting or total depigmentation are
options, also psoralen–UV-A (PUVA) and narrow band UV-B

What are the characteristics seen in albinism?


White skin and hair, red eyes, translucent iris, impaired vision with nystagmus

What are albinos predisposed to?


Skin cancer

In what patient population is acanthosis nigricans seen?


Obese patients and patients with diabetes

What can acanthosis nigricans be a sign of?


It may indicate the presence of a malignancy.
HYPERSENSITIVITY REACTIONS

What is Henoch-Schönlein purpura?


An IgA small vessel hypersensitivity vasculitis in which immune complexes lodge in small vessels resulting
in inflammation, fibrinoid necrosis, and palpable purpura. Patients have a hypersensitivity reaction to
antigens in immune complex.

What is seen on physical examination?


Palpable purpura, usually of the lower extremities and buttocks.
Lesions may be crusted because of tissue necrosis.
Patients also present with abdominal pain, pruritis, fever, and malaise.

What is palpable purpura?


Nonblanchable, red papules

What patient population is Henoch-Schönlein purpura usually seen in?


Children

What are the criteria for diagnosis of a hypersensitivity vasculitis according to the American College of
Rheumatology?
Three of the following must be present:
Meds taken at onset of disease
Age >16 at onset of disease
Palpable purpura
Maculopapular rash
Eosinophils seen on biopsy

What can it potentially progress to and why?


Rarely it progresses to glomerulonephritis because IgA deposits in glomeruli.

What is the treatment?


Treat the underlying cause; systemic corticosteroids; immunosuppressives in serious cases but often self-
limiting

What is erythema multiforme?


Immune complex hypersensitivity reaction to various causative agents

What are the various causes of erythema multiforme?


Half of all cases are idiopathic but other causes are:
Infections
Bacterial (Streptococcus, Mycoplasma)
Viral (herpes simplex, hepatitis A or B) Fungal
Drugs: nonsteroidal anti-inflammatory drugs (NSAIDs), penicillin, sulfonamides, thiazide diuretics,
barbiturates, phenytoin
Malignancy
Collagen vascular disease

What is the pathopneumonic lesion?


Erythematous target lesions with red center and dark outer ring in many different shapes (that is why it is
called multiforme) (Fig 11-8)

Figure 11-8 Target lesions. (Courtesy of Noah Craft, MD, PhD)

Where are lesions mostly found?


On the palms, soles, and extremities

What forms can the lesions take?


Many forms—vesicles, papules, bullae

What is the treatment?


Treat the underlying cause. Stop any drugs causing the reaction or treat any underlying infection.

What is Stevens-Johnson syndrome?


A severe form of erythema multiforme with systemic symptoms as well as mucous membrane involvement
(oral mucosa and conjunctiva); <10% of body; potentially fatal (Fig 11-9)

Figure 11-9 Stevens-Johnson syndrome. (Courtesy of Noah Craft, MD, PhD)

What is the treatment?


Remove/treat causative agent; systemic corticosteroid therapy; treat skin lesions as burns; immune globulin
intravenous (IGIV) potentially helpful

What can Stevens-Johnson syndrome progress to?


Toxic epidermal necrolysis (TEN)

How is TEN different from Stevens-Johnson?


>30% of body surface area with full-thickness skin necrosis; higher risk of being fatal

What happens to the target lesions in TEN?


Lesions become confluent, tender, erythematous, and become bullae. There is eventual loss of the epidermis.

What is “positive” sign for TEN?


Nikolsky sign—Gentle manual traction leads to sloughing off of epidermis.

What is the treatment?


Remove/treat causative agent (acyclovir to prevent herpes recurrence); fluid and electrolyte replacement;
systemic corticosteroids; IGIV may be helpful

What is erythema nodosum?


A painful inflammation of subcutaneous fat

What is the etiology?


Most cases are idiopathic.
Other causes:
Drugs: oral contraceptives, sulfonamides
Infections: Streptococcus, TB, leprosy, Chlamydia
Autoimmune: inflammatory bowel disease, Behçet, sarcoidosis, rheumatic fever, pregnancy

In what patient population is it most commonly seen?


Young women between the ages of 15 and 30

What is seen on physical examination?


Erythematous nodules on lower legs that are extremely tender to touch
(Note: Nodules are bilateral but not symmetric. Occasionally found on forearms or other areas with fat.) (Fig
11-10)

Figure 11-10 Erythema nodosum. (Courtesy of Noah Craft, MD, PhD)

How is the diagnosis confirmed?


CBC, CXR, throat culture, antistreptolysin-O

What is the treatment?


Treat the underlying cause as well as anti-inflammatories for pain and leg elevation.

What is pityriasis rosea? And what is the sequence of eruption?


A self-limiting maculopapular pruritic rash with central scale that begins as a single herald patch on the
trunk, then followed by a generalized rash of pink scaly patches within 2 weeks of the initial eruption.
Caused by HHV-7. (Fig 11-11)

Figure 11-11 Herald patch in pityriasis rosea. (Courtesy of Noah Craft, MD, PhD)

What is the pattern of distribution of the generalized rash?


Christmas tree pattern on the back (Fig 11-12)

Figure 11-12 Herald patch and Christmas tree pattern.

In what season is this most commonly seen?


Spring

In what patient population does it most commonly present?


Children and young adults

What is the treatment?


Treatment is symptomatic only; it usually self-resolves in 6-8 weeks; however, sunlight helps.
Symptomatic treatment includes antihistamines, topical corticosteroids, and calamine lotion.

What is scabies?
An infection by the Sarcoptes scabiei mite which causes an extremely pruritic papular rash. Lesions are
contagious.

What should you look for on physical examination if you suspect scabies?
Burrows in webs of finger, toes, and other intertriginous areas

How is it diagnosed?
Microscopic identification of the S scabiei mite in skin scraping

What is the treatment?


Permethrin 5% cream to entire body for 8 hours, then repeat 1 week later. Wash all linens. Antihistamines
can help with pruritis.

How long can postscabies pruritis last after treatment?


6-8 weeks after treatment

Who should be treated?


Patient with scabies and all close contacts

CLINICAL VIGNETTES

An obese 37-year-old diabetic, hyperlipidemic male presents to your clinic for a follow-up. His HgA1c is 8.
His LDL is 100 and HDL is 40. He complains of a dark “rash” that has slowly appeared in the posterior
fold of his neck. It is nonpruritic, and not painful. On examination there is an area of hyperpigmented skin
that has a “velvety” texture to it. What is the most likely diagnosis?
Acanthosis nigricans

In the month of March, a healthy 23-year-old female presents to your office complaining of a very pruritic
rash. She states that the rash started as a single larger lesion; then about 5 days later she broke out in a
rash on her entire back that is extremely pruritic. She has not started any new medications or used any
new products either. This has never happened to her in the past. On examination you find a single larger
scaly lesion on her upper back and a generalized rash consisting of pink scaly patches. Her vitals are within
normal limits. What is the most likely diagnosis?
Pityriasis rosea

A 17-year-old male presents complaining of some lesions on his back. He states that they do not cause him
any discomfort. There is no itching or pain. On examination you find small, hypopigmented patches on his
upper back. With scratching there is fine overlying scale. A KOH preparation shows pseudohyphae and
yeast. What is the organism causing his rash?
Malassezia furfur is the organism in tinea versicolor.

An 87-year-old female who lives in a convalescent home is brought in for evaluation of a rash. The rash is
extremely pruritic, especially in the evening. The caretaker mentions that her roommate has similar
symptoms. The patient has a generalized rash with a lot of excoriation marks on her body. When you look
in the webs of her fingers you find superficial burrows. What is the most likely diagnosis?
Scabies

A fair-skinned 57-year-old construction worker presents for a skin check. On the helix of his ear you find
red, scaly rough patches. They are then treated with liquid nitrogen. What are these lesions precursors of?
SCC
CHAPTER 12
Clinical Vignettes Review

A 45-year-old female who recently had surgery for a thyroid cancer develops perioral paresthesias,
confusion, and muscle weakness. An EKG was performed and it demonstrated a prolonged QT interval.
What is the most likely reason for this woman’s symptoms?
Hypocalcemia

A 30-year-old African-American female presents to your office complaining of a photosensitive skin rash
over her nose and cheeks as well as fever and polyarthritis. She reports no pain. You listen to her heart and
hear a murmur. Based on the previous findings, what do you think is the most likely cause of the murmur?
Libman-Sacks endocarditis (Mnemonic: SLE causes LSE)

A 60-year-old alcoholic female presents with severe back pain with nausea and vomiting. Abdominal x-ray
shows a sentinel loop. What is the most likely diagnosis?
Pancreatitis

A patient diagnosed with leukemia has Auer rods on blood smear. What type of leukemia does he have?
Acute myelogenous leukemia (AML)

A young boy presents to the dentist and is found to have excessive bleeding. Laboratory tests are
performed and he is found to have a prolonged bleeding time with normal prothrombin time (PT)/partial
prothrombin time (PTT). What is the diagnosis of choice?
von Willebrand disease

A 58-year-old female presents with acute renal failure of unknown etiology. Urinalysis shows Bence Jones
proteinuria and she is found to have a monoclonal gammopathy. What is the diagnosis?
Multiple myeloma

A 70-year-old male in the intensive care unit (ICU) on total parenteral nutrition (TPN) for 10 days
develops jaundice. Liver function test demonstrates a total bilirubin of 12. What is the most likely cause of
his hyperbilirubinemia?
Cholestasis caused by parenteral nutrition

A 60-year-old female is found to be hypotensive on pressors with minimal improvement. Her chest x-ray
(CXR) demonstrates an enlarged heart that resembles a water bottle. What would be the test you would
order to make the diagnosis?
Echocardiogram. This patient most likely has a pericardial effusion.

A 50-year-old male who had a myocardial infarction (MI) approximately 3 weeks prior presents with fever
and elevated erythrocyte sedimentation rate (ESR). What is the most likely diagnosis?
Dressler syndrome

You are called by the nurse about a hospitalized patient with a blood pressure of 100/60. You go to evaluate
the patient and on examination she has distant heart sounds and jugular venous distention (JVD). You
order an EKG and you notice that the height of the QRS complex varies from beat to beat. What is your
diagnosis?
Cardiac tamponade
An otherwise healthy medical student gets his annual purified protein derivative (PPD) (tuberculin) as
required by medical school; 48 hours later he goes to have it read and it measures 10 mm. What would you
tell this student about the results?
He has a positive PPD and needs to be treated with 6-9 months of isoniazid (INH).

A 30-year-old female presents with fatigue for several months. She has also had multiple urinary tract
infections (UTIs) over the past year. You order a complete blood count (CBC) with a peripheral smear.
The smear shows Auer rods and 52% myeloblasts. What is the diagnosis?
Acute myelocytic leukemia

A 60-year-old male presents to your office for a physical examination. He has no past medical history, does
not drink or smoke, and currently takes no medications. His physical examination is benign except that he
appears somewhat pale. His CBC shows a hemoglobin of 11 and the mean corpuscular volume (MCV) is
70. He has a low ferritin, low serum iron, and elevated total iron-binding capacity (TIBC). What is your
next step?
Screen for colon cancer—Iron deficiency anemia is colon cancer until proven otherwise.

Your 16-year-old patient comes to your office because his friends told him that he looks “yellow.” He has
no past medical history and is not taking any medications. He denies any recent antibiotic use. He does
mention that he has felt fatigued over the past 2 days. He also says that he tried Indian food for the first
time a few days ago. He had a really tasty bean dish. You order a CBC and his hemoglobin is 8. What is the
diagnosis?
Glucose-6-phosphate dehydrogenase (G6PD) deficiency

An HIV patient with a CD4 count of 198 comes to see you for follow-up on his HIV. What new antibiotic
would you initiate?
Bactrim as prophylaxis against Pneumocystis carinii pneumonia (PCP)

A 55-year-old alcoholic male is brought in to the emergency room (ER) for altered mental status. He is
found to have a pulse oxygen of 85%. A stat CXR is done and the patient is found to have a large right
upper lobe consolidation. He is reported to have a “currant jelly” sputum. What is the most likely
organism?
Klebsiella secondary to an aspiration pneumonia

A 25-year-old female presents to your office complaining of diarrhea, weight loss, and heart palpitations.
What initial test would you order?
Thyroid-stimulating hormone (TSH) and T4 (Think: hyperthyroidism)

A 70-year-old male presents with renal failure. During your history and physical on your review of systems
you discover that he has been having bone pain and weight loss over the past several months. On your
initial laboratory assessment you find that your patient is hypercalcemic, has rouleaux formation, and has
Bence Jones proteins in his urine. A serum protein electrophoresis demonstrates an “M” spike. You order
an x-ray and find he has “punched out lesions.” What is the most likely diagnosis?
Multiple myeloma

A patient presents to the ER with symptoms of nausea, vomiting, and fatigue. He tells you that over the
past few months he has had a significant amount of weight loss. His sister, who has come to the hospital
with him, says that she has noticed that recently his skin has become very tanned as well. You question the
patient about recent sun exposure and he tells you that he has had very little since he is an accountant and
is indoors most of the day. His laboratory tests reveal that he is hyponatremic and hyperkalemic. What
diagnostic test would you order for this patient?
Plasma adrenocorticotropic hormone (ACTH) level to evaluate for Addison disease
A patient with a history of IV drug abuse presents to the hospital with high fever and chills. On physical
examination you hear a new murmur. Blood cultures are drawn and are positive × 2 with Streptococcus
viridans. What is the most likely diagnosis?
Endocarditis. The tricuspid valve is most likely involved.

Unfortunately, your patient has been diagnosed with lung cancer. He has been feeling very weak and
fatigued for the past few days and develops an altered level of consciousness. A CT scan was done and
fortunately there were no metastases to the brain. Electrolytes show that he has a sodium of 125. His
glucose is within normal limits. What test would you order next to confirm your suspected diagnosis?
Urine electrolytes to confirm the most likely diagnosis of SIADH (syndrome of inappropriate antidiuretic
hormone)

A nursing home patient who is alert and oriented presents with severe hyponatremia. Your colleague treats
the patient with hypertonic saline and is able to correct his sodium within 7 hours. Subsequently, the
patient becomes unresponsive and unarousable. Your colleague does not know what happened. What
would you tell her was the cause of her patient’s rapid alteration in mental status?
The patient has central pontine myelinolysis. Hyponatremia should never be corrected too quickly for this
reason.

A patient in renal failure complains of chest pain. Her potassium is 6.5. A stat EKG shows peaked T waves.
What would be the initial treatment that should be given?
Calcium gluconate to protect the heart

Your next patient in clinic is a 75-year-old white male visiting for a routine physical. He mentions that he
has noticed a lesion on the ridge of his ear. You take a look at it and find it is pearly in appearance and has
some telangiectasias. What is the most likely diagnosis?
Basal cell carcinoma

A sexually active 18-year-old male presents with a hot, swollen, severely painful right knee for the past 2
days. He denies any history of trauma to the joint that he can recall. What is the next step in diagnosis?
Arthrocentesis. Most likely organism is Neisseria gonorrhoeae.

A 77-year-old female complains of severe joint pain over the past several years. On her hands you notice
some nodules on her proximal interphalangeal joints (PIP). What are these nodules called?
Bouchard nodes

A 45-year-old woman presents to the ER complaining of dyspnea and chest pain. She just came back from
a cross-country road trip. She also tells you that she had one episode of hemoptysis. The nurse takes her
vitals. They are: Tmax: 37.8°C; BP: 130/90; pulse: 110; respiratory rate: 28; and oxygen saturation of
88%. You examine the patient and find that her left calf is swollen and tender. What is the most likely
diagnosis for this patient’s shortness of breath?
Pulmonary embolism from a deep venous thrombosis (DVT) in her left lower extremity

A 20-year-old patient presents with altered level of consciousness. His parents report that he has been very
thirsty recently. A serum glucose is 849. What test could you order to differentiate between type 1 and type
2 diabetes?
C-peptide. It would be missing in type 1 diabetics.

A 45-year-old obese female presents with a 2-day history of nausea, vomiting, and abdominal pain. On
examination the patient has right upper quadrant pain. You suspect cholecystitis so you order a right
upper quadrant ultrasound. The test is equivocal. What is your next step in management?
This patient needs a hydroxy iminodiacetic acid (HIDA) scan.
A 70-year-old male with a 35-pack-year history of smoking presents with dyspnea on exertion. The patient
has a chronic dry cough and his voice sounds very hoarse. Physical examination demonstrates decreased
breath sounds, a hyper-resonant chest, and distant heart sounds. A CXR reveals flattened diaphragms.
What is the diagnosis?
Chronic obstructive pulmonary disease (COPD)

A 25-year-old male presents with acute right knee pain. The patient denies any history of trauma but does
report fever and chills. He also tells you that over the last week he has had pain in multiple joints as well.
He admits to you that he is sexually active with multiple partners and does not like to use any protection.
On physical examination, the knee is swollen, erythematous, and painful. He has a rash on his palms. You
tap the joint and the fluid demonstrates gram-negative diplococci. What is the diagnosis?
Gonococcal arthritis

An 84-year-old male with a past medical history significant for hypertension, hyperlipidemia, and diabetes
presents with left-sided paralysis. He is admitted to the hospital for further workup. In the next 15 hours,
his symptoms resolve. What is the most likely diagnosis?
Transient ischemie attack (TIA)

A 19-year-old male presents to your clinic complaining of a “rash” on his knees and elbows. He says that he
has used moisturizer on it with no improvement. On physical examination, you find silvery white scaly
patches on his elbows and knees. You also notice that he has pitting of some of his fingernails. What is the
most likely diagnosis?
Psoriasis

A young male presents with a 3-month history of night sweats, fatigue, and 15-lb weight loss. He has
noticed that he has a single, nontender cervical lymph node that does not seem to be resolving. He did
mention that his symptoms seem worse with alcohol consumption. A CBC demonstrates leukocytosis. A
lymph node biopsy demonstrates binucleated giant cells (Reed-Sternberg cells). What is the diagnosis?
Hodgkin lymphoma

A patient presents with altered mental status with petechiae on the lower extremities. The patient has a
temperature of 38.3°C, blood pressure of 110/80. The following are the patient’s labs: CBC: WBC 10,
hemoglobin 10, hematocrit 26, and platelets 50. Electrolytes demonstrates hyperkalemia and blood urea
nitrogen/creatine (BUN/CR) of 40/2.5. The patient has an elevated lactate dehydrogenase (LDH) and
unconjugated bilirubin. What is the diagnosis?
Thrombotic thrombocytopenic purpura (TTP)

An 18-year-old athlete presents with an erythematous, pruritic skin eruption in the intertriginous region. A
potassium hydroxide (KOH) scraping demonstrates hyphae. What is the diagnosis?
Tinea cruris

An HIV patient presents with purple-colored macules and nodules on his skin. It is caused by human
herpesvirus 8 (HHV 8). What is the diagnosis?
Kaposi sarcoma
Bibliography

Andreoli TE, Carpenter CCJ, Griggs RC, et al. Cecil Essentials of Medicine. 7th ed. WB Saunders, 2007.

Fauci AS, Braunwald E, Kasper DL, et al. Harrison’s Principles of Internal Medicine. 17th ed. New York,
NY: McGraw-Hill, 2008.

Humes HD. Kelly’s Textbook of Internal Medicine. 4th ed. Lippincott Williams & Wilkins, 2000.

Barker LR, Burton JR, Zieve PD, et al. Principles of Ambulatory Medicine. 6th ed. Lippincott Williams &
Wilkins, 2002.

Dale D. Rapid Interpretation of EKGs. 6th ed. Cover Publishing Company, 2000.
Index


A-a gradient
AAA. See Abdominal aortic aneurysm (AAA)
Abbreviations, list of important
Abdominal aortic aneurysm (AAA)
preventative screening for
ABGs. See Arterial blood gases (ABGs)
Ablation, radioactive iodine
Abscess(es)
Absence seizures
Absolute risk, defined
Acanthosis nigricans
Accuracy, defined
Achalasia
Acid-base disorders
Acidosis(es)
metabolic
respiratory
Acne
Acne vulgaris
Acromegaly
ACTH. See Adrenocorticotropic hormone (ACTH)
Actinic keratoses
Acute cholecystitis, evaluation of, radiography in
Acute interstitial nephritis (AIN)
Acute lymphoblastic leukemia (ALL)
Acute myelocytic leukemia, vignette
Acute myelogenous leukemia (AML)
vignette
Acute renal failure (ARF)
Acute respiratory distress syndrome (ARDS)
Acute tubular necrosis (ATN)
Addison disease
vignette
ADH. See Antidiuretic hormone (ADH)
Adrenal disorders
Adrenal gland
Adrenal insufficiency
primary
secondary
Adrenocorticotropic hormone (ACTH), action of
African Burkitt lymphoma
Afterload, decrease in
AIDS (acquired immunodeficiency syndrome)
AIN. See Acute interstitial nephritis (AIN)
Albinism
Albuminocytologic dissociation
Aldosterone, function of
Alkalosis(es)
metabolic
respiratory
ALL. See Acute lymphoblastic leukemia (ALL)
Allergy(ies), patient history of
Alpha blockers, for hypertension, side effects of
Alpha-1-Antitrypsin, in lung, function of
Alpha-thalassemia
ALS. See Amyotropic lateral sclerosis (ALS)
Alzheimer dementia
Amantadine, mechanism of
American Burkitt lymphoma
Amino acids, calories from
AML. See Acute myelogenous leukemia (AML)
Ammonium magnesium phosphate stone
Amyotropic lateral sclerosis (ALS)
Anaerobe(s), in pneumonia
Anaplastic cancer
Anemia(s)
aplastic
categories of
causes of
hemolytic
cold autoimmune
iron deficiency
macrocytic
microcytic
normocytic
pernicious
sickle cell
sideroblastic
thalassemias
types of
Aneurysm(s)
aortic, abdominal
preventative screening for
berry
Angina
acute, treatment of
Prinzmetal
stable, defined
treatment of, long-term
unstable, evaluation of
Angiotensin-converting enzyme (ACE) inhibitors, for hypertension
contraindications to
side effects of
Anion gap metabolic acidosis
Ankylosing spondylitis
Anterior cerebral artery (ACA)
Anterior inferior cerebellar artery
Anterior-posterior (AP) film
Anticholinergic drugs, types of
Antidiuretic hormone (ADH), action of
Antral G-cell hyperplasia
Aortic aneurysm(s), abdominal
preventative screening for
Aphasia(s)
Broca
Wernicke
Aplastic anemia
Apraxia
ARBs, for hypertension, side effects of
ARDS. See Acute respiratory distress syndrome (ARDS)
ARF. See Acute renal failure (ARF)
Arrhythmia(s)
Arterial blood gases (ABGs)
in chronic bronchitis
in end-stage emphysema
Arteritis
Takayasu
temporal
Arthritis
gonococcal, vignette
rheumatoid
Arthrocentesis, vignette
Arthropathy(ies)
Ascites
Aspergillus spp.
mucormycosis due to
in pneumonia
Aspiration, diet for patients at risk for
Aspiration pneumonia, vignette
Asterixis
Asthma
Astrocytoma
Atelectasis
Atheroma(s), carotid
ATN. See Acute tubular necrosis (ATN)
Atrial fibrillation
therapeutic INR level for patient on warfarin with
Atrial flutter
Auspitz sign
Azotemia


Bacteria
gram-negative
gram-positive
Bacterial infections
Bacterial peritonitis
spontaneous
Bacterial vaginosis
Bamboo spine
Barrett esophagus
Basal cell carcinoma
vignette
Basal energy expenditure (BEE), in males, calculation of
Base excess, laboratory values for
BEE. See Basal energy expenditure (BEE)
Behçet syndrome
Benign positional vertigo
Benign prostatic hyperplasia (BPH), hypertension with, treatment of
Berry aneurysm
Beta-blockers, for hypertension
contraindications to
side effects of
Beta-thalassemia
Bias(es)
lead time, defined
length time, defined
Bile acid sequestrants, mechanism of
Biliary tract, evaluation of, radiographic study in
Biliary tract disorders
“Blackhead”
Blast crisis
Bleeding. See also Hemorrhage
GI
intercranial, platelet count and
intracranial
variceal
Blistering diseases
Blood, in body mass
Blood pressure, preventative screening for
Blood products
Blood tests
Blood transfusion(s)
infections related to
rejection of
Blood types
universal donor
universal recipient
“Blown” pupil
Bone(s)
Paget disease of
Bone infection
Borrelia burgdorferi
Bouchard nodules, vignette
Boutonniere deformity
Bowel(s), inflammatory
BPH. See Benign prostatic hyperplasia (BPH)
Brain tumors
Breast cancer, preventative screening for
Breslow classification
Brocca aphasia
Bromocriptine, mechanism of
Bronchiectasis
Bronchitis
chronic, defined
vs. emphysema
Brudzinski sign
Buerger disease
Bulla(ae)
Bullous pemphigoid
Burkitt lymphoma
African
American


CABG. See Coronary artery bypass graft (CABG)
CAD. See Coronary artery disease (CAD)
Calcitonin, uses for
Calcium
daily requirement for
normal range for
Calcium channel blockers, short-acting, for hypertension, contraindications to
Calcium gluconate, vignette
Calcium pyrophosphate
Calcium pyrophosphate stone
Calculation, defined
Calorie(ies), from amino acids
Campylobacter jejuni
Cancer(s). See also specific types
anaplastic
breast, preventative screening for
cervical
colon
vignette
colorectal, preventative screening for
follicular
gastric
lung
medullary
ovarian, preventative screening for
papillary
prostate, preventative screening for
skin
Candida spp.
Candidiasis
Captopril stimulation test
Carbamazepine, side effects of
Carbohydrate(s), kilocalories in
Carbon monoxide poisoning
clinical sign of
hypoxia due to
Carbuncle(s)
Carcinoid syndrome, treatment of
Carcinoid tumors, of lung, diagnosis and treatment of
Cardiac enzymes
Cardiac tamponade
vignette
Cardioembolic stroke, treatment of
Cardiology
vignettes
Cardiomyopathy
dilated
Carotid artery stenosis, evaluation of, radiography in
Carotid atheroma
Carotid endarterectomy, indications for
CD4 counts
Celiac sprue
Cellulitis
Central pontine myelinolysis, vignette
Cerebrospinal fluid (CSF), in meningitis
Cerebrovascular accidents (CVAs)
causes of
defined
Cervarix
Cervical cancer
Chest pain, cardiac enzymes tested in
Chest x-ray (CXR)
adequacy of, determination of
in CHF
features seen on
in emphysema
evaluation of
parts of
in pneumonia
positions for
in pulmonary embolism
reading of
Chewing difficulty, diet for patients with
CHF. See Congestive heart failure (CHF)
Chlamydia spp.
C psittaci, in pneumonia
in pneumonia
preventative screening for
Cholangitis
Cholecystitis
acute, evaluation of, radiography in
Cholelithiasis
Cholestasis
vignette
Cholesterol screening
Cholesterol stone
Chromosome(s), Philadelphia
Chronic lymphocytic leukemia (CLL)
Chronic myelogenous leukemia (CML)
Chronic obstructive pulmonary disease (COPD)
defined
radiographic findings in
treatment of
types of
vignette
Chronic renal failure (CRF)
Churg-Strauss disease
Chvostek sign
Cirrhosis
Clarke classification
Clindamycin, Clostridium difficile and
CLL. See Chronic lymphocytic leukemia (CLL)
Closed comedone
Clostridium difficile
Cluster headache
CML. See Chronic myelogenous leukemia (CML)
CMV. See Cytomegalovirus (CMV)
Coagulation, normal
Coagulopathy(ies)
Coccidioides immitis, in pneumonia
Cognitive disorders
Alzheimer dementia
delirium
dementia
Huntington disease
Parkinson disease
Wilson disease
Cold autoimmune hemolytic anemia
Cold nodule
Colitis
pseudomembranous
ulcerative
vs. Crohn disease
Colon. See also specific disorders/conditions
Colon cancer
vignette
Colon cutoff sign
Colorectal cancer, preventative screening for
Colovesicular fistula
Coma
HHNK
myxedema
Comedone(s)
Complex partial seizures
Computed tomography (CT), structures visualized on
Congestive heart failure (CHF)
causes of
CXR of
features seen on
defined
hypertension with, treatment of
treatment of
Conn syndrome
Contraction(s), premature ventricular
COPD. See Chronic obstructive pulmonary disease (COPD)
Coronary artery bypass graft (CABG), indications for
Coronary artery disease (CAD)
defined
diet for patients with
mortality associated with, treatment for decreasing
risk factors for
Coronary artery disease (CAD) equivalents
Corynebacterium spp.
Cough
acute
causes of
defined
chronic
causes of
defined
Coumadin. See Warfarin
Courvoisier sign
Coxiella burnetii, in pneumonia
C-peptide, vignette
Creatinine clearance
CREST syndrome
CRF. See Chronic renal failure (CRF)
Crohn disease
vs. ulcerative colitis
Cryptococcus spp.
in pneumonia
CSF. See Cerebrospinal fluid (CSF)
CT. See Computed tomography (CT)
Cullen sign
Curling ulcer
Cushing disease
Cushing syndrome
Cushing ulcer
CVAs. See Cerebrovascular accidents (CVAs)
CXR. See Chest x-ray (CXR)
Cyclosporine, blood tests in patients taking
Cytomegalovirus (CMV), in pneumonia


Dawn phenomenon
de Quervain thyroiditis
Decerebrate rigidity
Decorticate posturing
Deep vein thrombosis (DVT)
pulmonary embolism due to
risk factors for
therapeutic INR level for patient on warfarin with
vignette
Delirium
vs. dementia
Dementia
Alzheimer
vs. delirium
Demyelinating diseases
ALS
Guillain-Barré syndrome
MS
Dermatology
vignettes
Dermatomyositis
Dexamethasone suppression test
Dextrose, in TPN calories from
Diabetes
complications of
prevention of
diagnosis of
diet for
hypertension with, treatment of
microalbuminuria in, treatment of
mucor in
symptoms of
treatment of, metformin in
type 1
complications of
treatment of
vignette
type 2
complications of
preventative screening for
treatment of
Diabetic ketoacidosis (DKA)
Dialysis
indications for
peritoneal
Diarrhea
Diastolic murmur
DIC. See Disseminated intravascular coagulation (DIC)
Diet(s), types of
Diffuse large cell lymphoma
Digoxin, toxicity of, presentation of
Disseminated intravascular coagulation (DIC)
Diuretic(s)
for hypertension, side effects of
thiazide, for hypertension, contraindications to
Diverticulitis
Diverticulosis
Diverticulum(a)
false
sites in colon
true
Zenker
DKA. See Diabetic ketoacidosis (DKA)
Dressler syndrome
vignette
Duodenal ulcers
posterior, complications of
signs and symptoms of
DVT. See Deep vein thrombosis (DVT)
Dysarthria
Dysphagia


Eaton-Lambert syndrome
Effusion(s), pleural
EKG. See Electrocardiography (EKG)
Elderly, delirium in, UTI and
Electrocardiography (EKG)
in angina patients
in atrial fibrillation
in atrial flutter
in hyperkalemia
in hypocalcemia
in hypomagnesemia
interpretation of
in MAT
in MI
parts of
in pericarditis
in pulmonary embolism
in ventricular fibrillation
in VT
in WPW syndrome
Electrolyte imbalances
hypercalcemia
hyperkalemia
hypermagnesemia
hypernatremia
hyperphosphatemia
hypocalcemia
hypokalemia
hypomagnesemia
hyponatremia
hypophosphatemia
preventative screening for
Embolism, pulmonary. See also Pulmonary embolism
Emphysema
panacinar
vs. bronchitis
Encephalopathy(ies), hepatic
Endarterectomy, carotid, indications for
Endocarditis
defined
diagnosis of
Libman-Sacks, vignette
signs and symptoms of
in SLE
treatment of
types of
valves affected by
vignette
Endocrinology
vignettes
Enzyme(s), cardiac
Ependymoma(s)
Epidural hematoma
Erythema chronicum migrans
Erythema multiforme
Erythema nodosum
Erythrasma
Escherichia coli
Esophageal disorders
Esophageal dysphagia
Esophageal spasm
Esophagus, Barrett
Essential thrombocytosis
Estrogen modulators, examples of
Experimental errors
Expiratory reserve volume, defined
Extracellular fluid, separation of
Extracellular water, percentage of


False diverticulum
False negative, defined
False positive, defined
Family history, described
Fat
kilocalories in
in PTN
Fat-soluble vitamins
FENa
Fever, rheumatic
FFP. See Fresh frozen plasma (FFP)
Fibrate(s), mechanism of
Fistula(s), colovesicular
Fitz-Hugh-Curtis syndrome
Fludrocortisone suppression test
Fluid(s). See also Extracellular water; specific types, e.g., Blood
extracellular, separation of
intravenous, composition of
preventative screening related to
Fluoroquinolone(s), contraindications to
Focal segmental glomerulosclerosis
Focal seizures
Folate deficiency
Folic acid deficiency
Follicle-stimulating hormone (FSH), action of
Follicular cancer
Folliculitis
Food(s), vitamin K in
Formula(s), common
Francisella tularensis, in pneumonia
FRC. See Functional residual capacity (FRC)
Fresh frozen plasma (FFP)
FSH. See Follicle-stimulating hormone (FSH)
Functional residual capacity (FRC), defined
Fungal infections
Furuncle(s)


Gardasil
Gastric cancer
Gastric malignancy, red flags for
Gastric outlet obstruction
Gastric ulcers
signs and symptoms of
Gastritis
Gastroenterology
vignettes
Gastroesophageal reflux disease (GERD)
Gastrointestinal (GI) bleed
Generalized seizures
Genital warts
GERD. See Gastroesophageal reflux disease (GERD)
GFR. See Glomerular filtration rate (GFR)
GH. See Growth hormone (GH)
“Ghon complex”
GI. See Gastrointestinal (GI) Bleed
Gigantism
Glioblastoma multiforme
Glomerular filtration rate (GFR)
Glomerulonephritis
membranous
with pneumonitis
poststreptococcal
rapidly progressive
types of
Glomerulonephropathy(ies)
Glucose-6-phosphate dehydrogenase (G6PD) deficiency
vignette
Goiter, multinodular
Gonococcal arthritis, vignette
Gonorrhea
Gonorrhea spp., preventative screening for
Goodpasture disease
Gout
Gram-negative bacteria
Gram-positive bacteria
Granulomatosis(es), Wegener
Graves disease
Grey Turner sign
Growth hormone (GH)
action of
Guillain-Barré syndrome
Gumma(s)


HAART (highly active antiretroviral therapy)
Haemophilus spp.
H influenzae
in pneumonia
H pylori, peptic ulcers due to
Hairy cell leukemia
Hampton hump
Hashimoto thyroiditis
HCO3, laboratory values for
Headache(s)
Heart beat
S1 sound in
S2 sound in
Heart block, types of
Heart disease
rheumatic
valvular
Heart failure, right
Heart murmurs
Heart valves, mechanical prosthetic, therapeutic INR level for patient on warfarin with
Heinz bodies
Hematocrit, to hemoglobin, formula for conversion of
Hematology-oncology
vignettes
Hematoma(s), epidural
Hemodialysis, vascular access for
Hemoglobin, hematocrit to, formula for conversion of
Hemolytic anemia
cold autoimmune
Hemolytic uremic syndrome (HUS)
Hemophilia(s)
Hemoptysis
Hemorrhage. See also Bleeding
splinterf
subarachnoid
subdural
Henoch-Schönlein purpura
Heparin, coagulation and
Hepatic encephalopathy
Hepatitis
A
B
blood transfusions and
vaccine for
C
blood transfusions and
Hepatorenal syndrome
Herpes simplex virus
Herpes zoster virus
HgA1c
HHNK coma. See Hyperosmolar hyperglycemic nonketotic (HHNK) coma
HIDA scan, vignette
Hidradenitis suppurativa
High altitude, hypoxia due to, treatment of
Highly active antiretroviral therapy (HAART)
Histoplasma spp., in pneumonia
History of present illness (HPI), described
HIV infection
blood transfusions and
preventative screening for
Hodgkin lymphoma
vignette
Holosystolic murmur
“Honeycomb lung”
Horizontal nystagmus
Hormone(s), actions of
Horner syndrome
Hot nodule
HPI. See History of present illness (HPI)
HPV. See Human papillomavirus (HPV)
Human immunodeficiency virus (HIV) infection. See also HIV infection
Human papillomavirus (HPV)
Huntington disease
HUS. See Hemolytic uremic syndrome (HUS)
Hydralazine, for hypertension, side effects of
Hydroxy iminodiacetic acid (HIDA) scan, vignette
Hyperaldosteronism, secondary
Hypercalcemia
Hypercalciuria
Hypercoagulable states, examples of
Hypercortisolism
Hyperkalemia
Hyperlipidemia
Hypermagnesemia
Hypernatremia
Hyperosmolar hyperglycemic nonketotic (HHNK) coma
Hyperparathyroidism
primary
secondary
Hyperphosphatemia
Hyperpigmentation
Hyperplasia(s), antral G-cell
Hypersecretory states
Hypersensitivity reactions
Hypertension
causes of
defined
malignant
signs and symptoms of
treatment of
portal
stroke and
treatment of
comorbidities and
contraindications to
side effects of
Hypertensive emergency, defined
Hypertensive urgency, defined
Hyperthyroidism
vignette
Hyphae
Hypocalcemia
vignette
Hypoglycemic agents, side effects of
Hypokalemia
Hypomagnesemia
Hyponatremia
Hypoparathyroidism
Hypophosphatemia
Hypopituitarism
Hypothesis(es), null, defined
Hypothyroidism
secondary
subclinical
tertiary
Hypovolemia, signs of
Hypoxia
causes of
high altitude and, treatment of
mechanisms of, mnemonic for
signs and symptoms of
treatment of


Idiopathic myelofibrosis
IgA nephropathy
Immune thrombocytopenic purpura (ITP)
Immunization(s). See also Vaccine(s)
adult
Impetigo
Incidence, defined
Infarct(s)
lacunar
watershed
Infection(s). See specific types
Infectious diseases
vignettes
Inflammation, signs of
Inflammatory bowel
Influenza vaccine
INR. See International normalized ratio (INR)
Inspiratory capacity (IC), defined
Inspiratory reserve volume (IRV), defined
Insulin
complications of
types of
Intercranial bleeding, platelet count and
International normalized ratio (INR)
laboratory tests related to
for patient on warfarin
Intracellular water, percentage of
Intracranial bleeding
Intravenous fluids, composition of
Intrinsic factor
Iron, absorption of
Iron deficiency anemia
Isotretinoin
ITP. See Immune thrombocytopenic purpura (ITP)


Janeway lesions


Kaposi sarcoma
vignette
Keratosis(es), actinic
Kernig sign
Ketoacidosis, diabetic
Kidney stone
evaluation of, radiography in
Kilocalories
in carbohydrates
in fat
in protein
Klebsiella spp.
in pneumonia
vignette
Knee(s), ACL of, evaluation of, radiography in
Köbner phenomenon
KUB (kidneys, ureter, and bladder) film


Laboratory studies, in patient history
Laboratory values
Lactase deficiency
Lacunar infarct
Lambert-Eaton syndrome
LDL. See Low-density lipoprotein (LDL)
Lead time bias, defined
Legionella spp., in pneumonia
Length time bias, defined
Lesion(s)
Janeway
skin, terminology related to
Leukemia(s)
hairy cell
lymphoblastic, acute
lymphocytic, chronic
myelocytic, acute, vignette
myelogenous
acute
vignette
chronic
signs and symptoms of
types of
LH. See Luteinizing hormone (LH)
Lhermitte sign
Libman-Sacks endocarditis, vignette
Lichenification
Likelihood ratio, defined
Lipoid nephrosis
Lipoprotein(s), low-density, goal for
Listeria spp., meningitis due to
Liver disorders
diet for patients with
Lou Gehrig disease
Low-density lipoprotein (LDL), goal for
Lung(s), alpha-1-antitrypsin in, function of
Lung cancer
categories of
causes of
diagnosis of
signs and symptoms of
types of
Lung disease, restrictive
Lung volumes
Luteinizing hormone (LH), action of
Lyme disease
Lymphoblastic lymphoma
Lymphogranuloma venereum
Lymphoma(s)
Burkitt
African
American
diffuse large cell
Hodgkin
vignette
lymphoblastic
non-Hodgkin
types of


Macrocytic anemias
Macule
Magnesium, normal range for
Magnetic resonance imaging (MRI), structures visualized on
Maintenance fluids
Malabsorption disorders
Malassezia furfur
Malignancy(ies), gastric, red flags for
Malignant hypertension
signs and symptoms of
treatment of
Mallory-Weiss tear
MAP. See Mean arterial pressure (MAP)
MAT. See Multifocal atrial tachycardia (MAT)
Mean arterial pressure (MAP), calculation of
Measles, mumps, rubella (MMR) vaccine
Mechanical prosthetic heart valve, therapeutic INR level for patient on warfarin with
Medullary cancer
Melanoma
Membranous glomerulonephritis
MEN syndromes
Ménétrier disease
Meniere disease
Meningioma(s)
Meningismus
Meningitis
causes of
CSF findings in
diagnosis of
in neonates
symptoms of
treatment of
Metabolic acidosis
Metabolic alkalosis
Metformin, for diabetics
Methotrexate, blood tests in patients taking
MGUS. See Monoclonal gammopathy of undetermined significance (MGUS)
MI. See Myocardial infarction (MI)
Microalbuminuria, in diabetics, treatment of
Microcytic anemias
Middle cerebral artery (MCA)
Migraine
Minimal change disease
Mitral stenosis
Mitral valve prolapse
MMR (measles, mumps, rubella) vaccine
Monoclonal gammopathy of undetermined significance (MGUS)
MS. See Multiple sclerosis (MS)
Mucor
Multifocal atrial tachycardia (MAT)
Multinodular goiter
Multiple endocrine neoplasia type I (MEN I)
Multiple myeloma
vignette
Multiple sclerosis (MS)
Murmur(s), heart
Murphy sign
Muscle disorders
Myasthenia gravis
Mycobacterium avium, in pneumonia
Mycoplasma spp., in pneumonia
Mycosis fungoides
Myelofibrosis, idiopathic
Myeloma(s), multiple
vignette
Myeloproliferative diseases
Myocardial infarction (MI)
complications of
defined
defining factors for
EKG changes with
emergent treatment of, mnemonic for
symptoms of
Myocarditis
Myxedema coma


Negative predictive value (NPV), defined
Neisseria spp.
N gonorrhoeae, vignette
N meningitidest
Neonate(s), meningitis in
Nephritic syndrome
Nephritis, acute interstitial
Nephrolithiasis
Nephrology
vignettes
Nephropathy, IgA
Nephrosis, lipoid
Nephrotic syndrome
Neurology
vignettes
Nicotinic acid, mechanism of
Nil disease
NNT. See Number needed to treat (NNT)
Nocardia spp., in pneumonia
Node(s), Osler
Nodule(s)
Bouchard, vignette
cold
hot
Non-Hodgkin lymphoma
Normocytic anemia
Nosocomial enteric infection
NPV. See Negative predictive value (NPV)
Null hypothesis, defined
Number needed to treat (NNT), defined
Nutrition
Nutritional status
acute change in, laboratory test in determination of
chronic, laboratory test in determination of
Nystagmus
horizontal
vertical


O2 saturation, laboratory values for
OA. See Osteoarthritis (OA)
Obstructive pulmonary diseases. See also specific diseases
Odds ratio, defined
Odynophagia
Oliguria
Onychomycosis
Open comedone
Ophthalmic artery
Opportunistic infections
Oral thrush
Oropharyngeal dysphagia
Osler nodes
Osteoarthritis (OA)
Osteomalacia
Osteomyelitis
Osteoporosis
hypertension with, treatment of
Ovarian cancer, preventative screening for
Oxygen, increased requirement for, example of


p value, defined
Packed red blood cells (PRBCs)
PaCO2, laboratory values for
Paget disease of bone
Pain, chest, cardiac enzymes tested in
Palpable purpura
PAN. See Polyarteritis nodosa (PAN)
Panacinar emphysema
Pancoast syndrome
Pancoast tumor
Pancreatitis
diet for patients with
vignette
Pancytopenia
Pansystolic murmur
PaO2, laboratory values for
Pap smears
Papillary cancer
Papillary muscle rupture
Papule
Paralysis(es), Todd
Paraneoplastic syndromes
Parathyroid disorders
Parathyroid hormone (PTH), elevation of
Parathyroidectomy
Parkinson disease
Paronychia
Partial parenteral nutrition (PPN)
Partial thromboplastic time (PTT)
Past medical history (PMH), described
Patch(es)
Pathognomonic murmur
Patient(s), admission of, procedure for
Patient history, components of
Pelvic inflammatory disease (PID)
Pemphigoid, bullous
Pemphigus vulgaris
Peptic ulcers
causes of
complications of
diagnosis of
pathology of
perforated, treatment of
risk factors for
signs and symptoms of
treatment of
types of
Pericardial effusion, vignette
Pericardial tamponade
Pericarditis
Peritoneal dialysis
Peritonitis, bacterial
spontaneous
Pernicious anemia
Petechiae
pH, laboratory values for
Phenytoid, side effects of
Pheochromocytoma
Philadelphia chromosome
Phosphate, normal range for
Physical examination, components of
PID. See Pelvic inflammatory disease (PID)
Pigment stone
Pigmentary disorders
Pituitary disorders
Pituitary gland
anterior
hormones secreted from
posterior
Pituitary tumors
Pityriasis rosea
Pityrosporum ovale
Plaque
Plasma cell dyscrasia
Platelet(s), transfusion of, thrombocytopenia and
Platelet count, intercranial bleeding and
Platelet disorders
Platelet levels
Pleural effusion
evaluation of, radiography in
radiographic findings in
Plummer disease
Plummer-Vinson syndrome
PMH. See Past medical history (PMH)
Pneumococcal vaccine
Pneumocystis spp.
P carinii, in pneumonia
vignette
P jiroveci, in pneumonia
Pneumonia(s)
aspiration, vignette
causes of, organisms
community-acquired, treatment of
CXR of
diagnosis of, laboratory tests in
nosocomial, treatment of
physical examination findings in
Pneumocystis carinii
vignette
radiographic findings in
signs and symptoms of
treatment of
types of
Pneumonitis, glomerulonephritis with
Pneumothorax
primary spontaneous
radiographic findings in
tension
Poisoning, carbon monoxide
clinical sign of
hypoxia due to
Polyarteritis nodosa (PAN)
Polycythemia vera
Polymyositis
Portal hypertension
Positive predictive value (PPV), defined
Posterior cerebral artery (PCA)
Posterior inferior cerebellar artery (PICA)
Posterior-anterior (PA) film
Postmyocardial infarction, hypertension with, treatment of
Postnasal drip, causes of
Poststreptococcal glomerulonephritis (PSGN)
Posturing, decorticate
Potassium
for hypertension, contraindications to
normal range for
Pott disease
Power, defined
PPD (purified protein derivative), positive
treatment of
PPN. See Partial parenteral nutrition (PPN)
PPV. See Positive predictive value (PPV)
PRBCs. See Packed red blood cells (PRBCs)
Precision, defined
Preload, decrease in
Premature ventricular contraction (PVC)
Prescription(s), how to write
Prevalence, defined
Preventative medicine
electrolyte evaluation
fluid evaluation
immunizations
preventative screening
Primary adrenal insufficiency
Primary hyperparathyroidism
Primary spontaneous pneumothorax
Prinzmetal angina
Procedure note
Proctitis, ulcerative
Prolactin, action of
Prolactinoma
Propionibacterium acnes
Propylthiouracil (PTU)
Prostate cancer, preventative screening for
Protein(s)
daily requirement for
kilocalories in
Proteus spp.
Prothrombin time (PT)
Providencia spp.
Pseudogout
Pseudohyperkalemia
Pseudohypocalcemia
Pseudohyponatremia
Pseudomembranous colitis
Pseudomonas spp.
in pneumonia
PSGN. See Poststreptococcal glomerulonephritis (PSGN)
Psoriasis
vignette
PT. See Prothrombin time (PT)
PTH. See Parathyroid hormone (PTH)
PTT. See Partial thromboplastic time (PTT)
PTU. See Propylthiouracil (PTU)
Pulmonary diseases, obstructive. See also specific diseases
Pulmonary edema, acute, treatment of
Pulmonary embolism
causes of
CXR findings in
diagnosis off
EKG inf
patient history in
signs and symptoms of
treatment of
vignette
Pulmonology
Pulsus paradoxus
Pupil(s), “blown”
Purified protein derivative (PPD), positive
treatment of
Purpura
Henoch-Schönlein
palpable
thrombocytopenic, thrombotic
vignette
Pustule
PVC. See Premature ventricular contraction (PVC)
Pyelonephritis


RA. See Rheumatoid arthritis (RA)
Radioactive iodine ablation
Radiologic studies. See also specific types, e.g., Chest x-ray (CXR)
CT
CXR
KUB film
MRI
Ramsay Hunt syndrome
Ransom criteria
Rapidly progressive glomerulonephritis
RBCs. See Red blood cells (RBCs)
Red blood cells (RBCs), life span of
Reed-Sternberg cells
Refeeding syndrome
Reiter syndrome
Relative risk, defined
Reliability, defined
Renal artery stenosis
Renal failure
acute
anemia in
chronic
diet for patients with
Residual volume (RV), defined
Respiratory acidosis
Respiratory alkalosis
Respiratory rate
decreased, causes of
increased, causes of
Restrictive lung disease
Reversible ischemic neurologic deficit (RIND)
Review of systems (ROS)
Rheumatic fever
Rheumatic heart disease
Rheumatoid arthritis (RA)
Rheumatology
vignettes
Rickets
Rickettsia rickettsii
Right heart failure
Right-to-left shunt
Rigidity, decerebrate
RIND. See Reversible ischemic neurologic deficit (RIND)
RMSF. See Rocky Mountain spotted fever (RMSF)
Rocky Mountain spotted fever (RMSF)
“Rodent ulcer”
ROS. See Review of systems (ROS)
Roth spots
Rupture(s), papillary muscle


S1 sound
S2 sound
SAAG (serum-ascites albumin gradient)
SAH. See Subarachnoid hemorrhage (SAH)
Salmonella spp.
Sarcoidosis
Sarcoma(s), Kaposi
vignette
SBP. See Spontaneous bacterial peritonitis (SBP)
Scabies
Schatzki ring
Schwannoma(s)
Scleroderma
Sclerosis(es)
amyotropic lateral
multiple
Scrofula
Secondary adrenal insufficiency
Secondary hyperaldosteronism
Secondary hyperparathyroidism
Seizure(s). See also Status epilepticus; specific types, e.g., Generalized seizures
absence
defined
evaluation of
focal
generalized, types of
risk factors for
treatment of
discontinuation criteria
types of
Seizure disorders. See also Seizure(s)
Selective estrogen modulators
Selegiline, mechanism of
Sensitivity, defined
Sentinel loop
Sepsis
Septic shock
Serum-ascites albumin gradient (SAAG)
Sexually transmitted diseases (STDs)
Sézary syndrome
Shock, septic
Shunt(s), right-to-left
SIADH. See Syndrome of inappropriate antidiuretic hormone (SIADH)
Sickle cell anemia
Sideroblastic anemia
Simple partial seizures
Sinemet, mechanism of
Sinusitis
Sipple syndrome
SIRS. See Systemic inflammatory response syndrome (SIRS)
Sjögren syndrome
Skin cancer
Skin lesions, terminology related to
SLE. See Systemic lupus erythematosus (SLE)
Small bowel obstruction, radiographic findings in
Social history, described
Sodium, normal range for
Sodium loading test
Somogyi effect
“Spaghetti and meatballs”
Spasm(s), esophageal
Specificity, defined
Spine, bamboo
Splinter hemorrhage
Spondylitis, ankylosing
Spontaneous bacterial peritonitis (SBP)
Spontaneous pneumothorax, primary
Squamous cell carcinoma
Stable angina. See also Angina
defined
Staphylococcus spp.
S aureus
in pneumonia
S saprophyticus
Statin(s)
mechanism of
side effects of
Statistics, types of
Status epilepticus
STDs. See Sexually transmitted diseases (STDs)
Stenosis(es)
carotid artery, evaluation of, radiography in
mitral
renal artery
Stevens-Johnson syndrome
Streptococcus spp.
group B, meningitis due to
S pneumoniae
in pneumonia
Streptokinase, contraindications to
Stroke
assessment of
cardioembolic, treatment of
defined
evaluation of, radiography in
hypertension and
risk factors for
treatment of
Struvite stones
Study types
Subacute thyroiditis
Subarachnoid hemorrhage (SAH)
Subdural hemorrhage
Sulfonylurea(s), side effects of
Superior vena cava syndrome
Surgical history, described
Swallowing difficulty, diet for patients with
Swan neck deformity
Syndrome of inappropriate antidiuretic hormone (SIADH)
vignette
Syphilis
preventative screening for
Systemic disorders
Systemic inflammatory response syndrome (SIRS)
Systemic lupus erythematosus (SLE)
endocarditis in
Systolic ejection murmur
T


Tabes dorsalis
Tachycardia(s)
multifocal atrial
ventricular
Takayasu arteritis
Tdap vaccine
Teeth, lack of, diet for patients with
Telangiectasia
Temporal arteritis
TEN. See Toxic epidermal necrolysis (TEN)
Tension headache
Tension pneumothorax
Thalassemia(s)
alpha-
beta-
Thiazide diuretics, for hypertension, contraindications to
Thiazolidinedione(s), side effects of
Thrombocytopenia
platelet transfuion and
Thrombocytosis, essential
Thrombolysis, indications for
Thrombolytics, contraindications to
Thrombotic thrombocytopenic purpura (TTP)
vignette
Thrush, oral
Thyroid disease/disorders
Thyroid gland, function of, evaluation of
Thyroid mass
Thyroid storm
Thyroiditis
de Quervain
Hashimoto
subacutet
Thyroid-stimulating hormone (TSH)
action of
vignette
TIA. See Transient ischemic attack (TIA)
Tidal volume (TV), defined
Tinea capitis
Tinea corporis
Tinea cruris, vignette
Tinea versicolor
Tissue plasminogen activator (tPA)
after embolic stroke
contraindications to
Todd paralysis
Tonic-clonic seizures
Torsades de pointes
Total lung capacity, defined
Total parenteral nutrition (TPN)
complications of
components of
fat in
indications for
Toxic epidermal necrolysis (TEN)
tPA. See Tissue plasminogen activator (tPA)
TPN. See Total parenteral nutrition (TPN)
Tram track lung markings
Transfusion(s), blood
Transient ischemic attack (TIA)
vignette
Trichomonas
Trichomonas spp.
Tropical sprue
Trousseau sign
True diverticulum
True negative, defined
True positive, defined
TSH. See Thyroid-stimulating hormone (TSH)
TTP. See Thrombotic thrombocytopenic purpura (TTP)
Tuberculosis
diagnosis of
extrapulmonary
healed primary
latent, detection of
lung affected by
risk factors for
secondary
signs and symptoms of
transmission of
treatment of
Tumor(s). See also specific types
brain
pituitary
Type and cross
Type and screen
Type I error, defined
Type II error, defined
Tzanck smear


UC. See Ulcerative colitis (UC)
Ulcer(s). See also specific types, e.g., Peptic ulcers
Curling
Cushing
duodenal
gastric
peptic
perforated, treatment of
“rodent”
Ulcerative colitis (UC)
vs. Crohn disease
Ulcerative proctitis
Universal donor blood type
Universal recipient blood type
Unstable angina, evaluation of
Uremia
Uremic syndrome
Uric acid stones
Urinary tract infections (UTIs)
delirium in elderly due to
Urine output, normal
UTIs. See Urinary tract infections (UTIs)


Vaccine(s), for adults
HBV
influenza
MMR
pneumococcal
Tdap
varicella
Vaginitis
Vaginosis, bacterial
Validity, defined
Valproic acid, side effects of
Valvular heart disease
Variceal bleed
Varicella vaccine
Vasculitis
Vector-borne diseases
Ventricular fibrillation
Ventricular tachycardia (VT)
Vertical nystagmus
Vertigo
Vesicle
Vital capacity, defined
Vitamin(s)
B12, absorption of
fat-soluble
K, in foods
Vitamin deficiencies
A
B1
B2
B3
B6
B12
C
folic acid
K
zinc
Vitamin K–dependent clotting factors
Vitiligo
Volume(s), lung
Volume status, assessment of
Volvulus
von Willebrand disease, vignette
von Willebrand factor (vWF) deficiency
V/Q mismatch, causes of
VT. See Ventricular tachycardia (VT)
vWF deficiency. See von Willebrand factor (vWF) deficiency


Warfarin
anticoagulation in patient with, reversal of
coagulation and
contraindications to
for pulmonary embolism
therapeutic INR levels for patient on
Wart(s), genital
Water, in body mass
Waterhouse-Friderichsen syndrome
Watershed infarct
Wegener granulomatosis
Wermer syndrome
Wernicke aphasia
Westmark signs
Wheal
Whipple disease
“Whitehead”
Wilson disease
Winter’s formula
Wolff-Parkinson-White (WPW) syndrome
WPW syndrome. See Wolff-Parkinson-White (WPW) syndrome
Writing notes


X-ray(s), chest. See Chest x-ray (CXR)

Z

Zenker diverticulum
Zinc, deficiency of
Zollinger-Ellison syndrome

You might also like